Pediatrics in Review® 2017 01 JANUARY

You might also like

Download as pdf or txt
Download as pdf or txt
You are on page 1of 79

JANUARY 2017

Vol. 38 No. 1
www.pedsinreview.org

Fungal Skin Infections


Gupta, MacLeod, Foley,
Gupta, Friedlander

Tube Feeding in Children


Singhal, Baker, Bojczuk, Baker

Overcoming Challenges
to Care in the Juvenile
Justice System: A Case
Study and Commentary
Savage, Reese, Wallace, Wang,
Jester, Lowe, Hyndman, Durant

ONLINE

Visual Diagnosis: A Child


with Presumed Reactive
Airway Disease, Pectus
Carinatum, and Aortic
Root Dilation
Jeewa, Morris, Dreyer, Adachi,
Denfield, McKenzie
contents

Pediatrics in Review ®
Vol. 38 No. 1 January 2017

COMMENTARY
1 Training to Teach, Teaching to Train
Joseph A. Zenel

3 Plagiarism in Review
Mark Weems

6 Then What?
Hugh D. Allen
ARTICLES
8 Fungal Skin Infections
Aditya K. Gupta, Melissa A. MacLeod, Kelly A. Foley,
Gita Gupta, Sheila Fallon Friedlander

23 Tube Feeding in Children


Sarita Singhal, Susan S. Baker, Georgina A. Bojczuk, Robert D. Baker

35 Overcoming Challenges to Care in the Juvenile Justice System:


A Case Study and Commentary
Rebekah J. Savage, Jasmine M. Reese, Stephenie Wallace, Timothy Wang,
Traci Jester, Robert Lowe, LaKeshia Hyndman, Nefertiti Durant
INDEX OF SUSPICION
44 Case 1: Poor Growth With Presence of a Pituitary Lesion in an
11-year-old Boy
Alexander S. Karageorgiadis, Charalampos Lyssikatos,
Elena Belyavskaya, Georgios Z. Papadakis, Nicholas J. Patronas,
Maya B. Lodish, Constantine A. Stratakis

46 Case 2: Fever and Back Pain in 13-year-old Girl


Reem Shawar, Pisespong Patamasucon, Shawn Rowles

48 Case 3: Recurrent Lesions on Palms of a 12-year-old Girl


Alexander K.C. Leung, Benjamin Barankin

49 Case 4: Worsening Abdominal Distention in a 2-year-old Boy


Sean Indra, Shazia Maqbool

50 Case 5: Fatigue, Polydipsia, and Nocturia in a 17-year-old Boy


S. Amara Ogbonnaya

52 Case 6: Episodic Stiffness in a 30-month-old Girl


Lisa Mitchell, William Adams, Francois Aspesberro
IN BRIEFS
54 Medication Reconciliation
Jenna Merandi, Matthew Sapko, Charline Catt, Jeffrey M Hoffman

56 CHARGE Syndrome
Alexandra Hudson, Carrie-Lee Trider, Kim Blake
ONLINE
e1 Visual Diagnosis: A Child with Presumed Reactive Airway Disease,
Pectus Carinatum, and Aortic Root Dilation
Aamir Jeewa, Shaine A. Morris, William J. Dreyer, Iki Adachi,
Susan W. Denfield, E. Dean McKenzie

Pediatrics in Review® (ISSN 0191-9601) is owned and controlled by the American Academy of Pediatrics. It is published monthly by the American Academy of Pediatrics,
141 Northwest Point Blvd., Elk Grove Village, IL 60007-1098.
Statements and opinions expressed in Pediatrics in Review® are those of the authors and not necessarily those of the American Academy of Pediatrics or its Committees.
Recommendations included in this publication do not indicate an exclusive course of treatment or serve as a standard of medical care.
Subscription price for 2017 for print and online/online only: AAP/CPS Member $210/$160; AAP National Affiliate Member $160/$110; Nonmember $265/$205; AAP
In-training Member $160/$110; Nonmember In-training/Allied Health $190/$130. Institutions call for pricing (866-843-2271). For overseas delivery, add $120. Current single issue
price is $22 domestic, $25 international. Replacement issues must be claimed within 6 months from the date of issue and are limited to three per calendar year.
Periodicals postage paid at ARLINGTON HEIGHTS, ILLINOIS and at additional mailing offices.
© AMERICAN ACADEMY OF PEDIATRICS, 2017. All rights reserved.
Printed in USA. No part may be duplicated or reproduced without permission of the American Academy of Pediatrics.
POSTMASTER: Send address changes to PEDIATRICS IN REVIEW®, American Academy of Pediatrics Customer Service Center, 141 Northwest Point Blvd., Elk Grove Village,
IL 60007-1098.
Pediatrics in Review ®

Editor-in-Chief: Joseph A. Zenel, Sioux Falls, SD Editorial Fellow: Aamir Jeewa, Toronto, ON
Deputy Editor: Hugh D. Allen, Houston, TX Early Career Physician: Heather Campbell, Washington, DC
Associate Editor, Index of Suspicion: Philip R. Fischer, Rochester, MN Editor Emeritus: Lawrence F. Nazarian, Rochester, NY
Associate Editor, Index of Suspicion: Deepak M. Kamat, Detroit, MI Founding Editor: Robert J. Haggerty, Canandaigua, NY
Associate Editor, Visual Diagnosis: Mark F. Weems, Memphis, TN Managing Editor: Luann Zanzola
Associate Editor, In Brief: Henry M. Adam, Bronx, NY Publications Editor: Sara Strand
Associate Editor, In Brief: Janet Serwint, Baltimore, MD Medical Copyediting: Deborah K. Kuhlman, Lisa Cluver
Associate Editor, CME: Rani Gereige, Miami, FL

EDITORIAL BOARD
Robert D. Baker, Buffalo, NY Neal S. LeLeiko, Providence, RI
Peter F. Belamarich, Bronx, NY Michael Macknin, Cleveland, OH
Eyal Ben-Isaac, Los Angeles, CA Susan Massengill, Charlotte, NC
Theresa Auld Bingemann, Rochester, NY Carrie A. Phillipi, Portland, OR
Stephen E. Dolgin, New Hyde Park, NY Peter Pizzutillo, Philadelphia, PA
Lynn Garfunkel, Rochester, NY Mobeen Rathore, Jacksonville, FL
Rani Gereige, Miami, FL Jennifer S. Read, Rockville, MD
Nupur Gupta, Boston, MA E. Steve Roach, Columbus, OH
Gregory A. Hale, St. Petersburg, FL Sarah E. Shea, Halifax, Nova Scotia
Thomas C. Havranek, Bronx, NY Andrew Sirotnak, Denver, CO
Jacob Hen, Bridgeport, CT Miriam Weinstein, Toronto, ON
Jeffrey D. Hord, Akron, OH

PUBLISHER: American Academy of Pediatrics


Mark Grimes, Director, Department of Publishing
Joseph Puskarz, Director, Division of Journal Publishing
Pediatrics in Review® Print Issue Editorial Board Disclosures
The American Academy of Pediatrics (AAP) Policy on Disclosure of Financial Relationships and Resolution of Conflicts of Interest for AAP CME Activities is designed to ensure
quality, objective, balanced, and scientifically rigorous AAP CME activities by identifying and resolving all potential conflicts of interest before the confirmation of service
of those in a position to influence and/or control CME content. All individuals in a position to influence and/or control the content of AAP CME activities are required to
disclose to the AAP and subsequently to learners that the individual either has no relevant financial relationships or any financial relationships with the manufacturer(s)
of any commercial product(s) and/or provider(s) of commercial services discussed in CME activities. Commercial interest is defined as any entity producing, marketing,
reselling or distributing health-care goods or services consumed by, or used on, patients.
Each of the editorial board members, reviewers, question writers, PREP Coordinating Committee members and staff has disclosed, if applicable, that the CME content
he/she edits/writes/reviews may include discussion/reference to generic pharmaceuticals, off-label pharmaceutical use, investigational therapies, brand names, and
manufacturers. None of the editors, board members, reviewers, question writers, PREP Coordinating Committee members, or staff has any relevant financial relationships to
disclose, unless noted below. The AAP has taken steps to resolve any potential conflicts of interest.
Disclosures
• Nupur Gupta, MD, MPH, disclosed she has a financial relationship with Springer US as co-editor for MassGeneral Hospital for Children Handbook of Pediatric Global Health.
• Michael Macknin, MD, FAAP, disclosed that: he and his wife each receive a free cruise for his seven one-hour talks annually on general pediatric subjects for University at
Sea; he receives a grant from the Wendel Family Foundation to fund a randomized study comparing various diets, and one member of the Wendel Family produced the
film “Forks Over Knives,” which describes the virtues of a vegan diet.
• Janet Serwint, MD, FAAP, disclosed she has a clinical research grant from the Centers for Disease Control and Prevention for quality improvement for HPV vaccines.
• Andrew Sirotnak, MD, disclosed that he serves as an expert witness in cases of suspected child abuse.
• Miriam Weinstein, MD, has disclosed she receives research funding (through her hospital’s foundation) from LaRoche-Posay.
The journal extends special thanks to the following question writers and ancillary reviewers who contributed to this issue:
–Denise Bratcher, MD
–Melissa Held, MD
–William Walker, MD
–Courtney Wusthoff, MD
Pediatrics in Review offers 36 CME articles per year. A maximum of one AMA PRA Category 1 CreditTM is earned after achieving a 60% score on each designated quiz.
CME STATEMENTS:
The American Academy of Pediatrics (AAP) is accredited by the Accreditation Council for Continuing Medical Education (ACCME) to provide continuing medical education
for physicians.
The AAP designates this journal-based CME activity for a maximum of 1.00 AMA PRA Category 1 CreditTM. Physicians should claim only the credit commensurate with the
extent of their participation in the activity.
This activity is acceptable for a maximum of 1.00 AAP credit. These credits can be applied toward the AAP CME/CPD* Award available to Fellows and Candidate Members of
the AAP.
The American Academy of Physician Assistants accepts certificates of participation for educational activities certified for AMA PRA Category 1 CreditTM from organizations
accredited by ACCME. Physician assistants may receive a maximum of 1.00 hour of Category 1 credit for completing this program.
This program is accredited for 1.00 NAPNAP CE contact hour; pharmacology (Rx) and psychopharmacology contact hours to be determined per the National Association of
Pediatric Nurse Practitioners (NAPNAP) Continuing Education Guidelines.
It has been established that each CME activity will take the learner approximately 1 hour to complete.
*Continuing Professional Development
How to complete this activity:
Pediatrics in Review can be accessed and reviewed in print or online at http://pedsinreview.aappublications.org. Learners can claim credit monthly online upon completion
of each CME article. The deadline for completing this activity is December 31, 2019. Credit will be recorded in the year in which it is submitted. It is estimated that it will take
approximately 1 hour to complete each CME article. This activity is not considered to have been completed until the learner documents participation in that activity to the
provider via online submission of answers. Course evaluations are online.
Commentary
Training to Teach, Teaching to Train
Although privileged to be Editor-in-Chief of Pediatrics in Review (PIR), I am even
more privileged to be a pediatrics residency program director. Training future
pediatricians similarly to the way pediatricians trained me is an opportunity to
instill the inquisitiveness, compassion, and dedication I witnessed in my teachers,
who I now realize wanted to impart their enjoyment, satisfaction, and sense of
purpose to pediatricians yet-to-be. My attending physicians made sure I knew my
patients and their illnesses well enough that when asked, at any time of day or
night, I could give in my own words a quick, succinct synopsis of a patient’s course
and a coherent rationale for that patient’s therapy. Each attending physician had
a different approach, some nurturing, others curt, but all strove to impress upon
an apprentice their personal interpretation of pediatrics. During my 3 years of
residency, thanks to constant faculty oversight, I grew confident in my skills
and took pride in knowing my patients, being responsible for them, and caring for
them. When I graduated and joined a rural practice, I was certain I was trained to
practice well.
What I did not know was that I also was trained to teach. Very soon in clinical
practice, I found that patients and parents frequently asked about the how and
why of disease and the how and why of treatment. Patient and parental acceptance
of my advice depended on understandable, practical explanations given with a
personal spin. Similarly, the cooperation of fellow nurses and physicians de-
pended on how well I could communicate. As I learned to teach, train, write, and
edit over the years, I found career satisfaction in being able to assess, compre-
hend, and pass on that comprehension. I too was contributing my own personal
interpretation of pediatrics. Through those many long rounds during residency,
my wise attendings had trained me to synthesize what I saw, heard, read, and
reported so I could ascribe meaning to my future patients, colleagues, and now,
residents.
Today, residency training is more standardized and formalized. The Accred-
itation Council of Graduate Medical Education (ACGME) requires residency
programs to train residents to be satisfactorily skilled in 6 competencies upon
graduation: patient care, medical knowledge, communication, professionalism,
practice-based learning and improvement, and systems-based practice. These 6
competencies define the essence of a doctor, a healer, an advocate, and a teacher.
A residency program director is expected to model these very same competen-
cies, and I find as a residency program director that these competencies are
reflected in PIR. In other words, the journal, in concert with the American Board of
AUTHOR DISCLOSURE Dr Zenel has Pediatrics, provides opportunities for pediatricians to maintain medical knowl-
disclosed no financial relationships relevant to edge and to improve good patient care by publishing professional communica-
this commentary. This commentary does not
tions that can be applied to the practice of pediatrics in all systems of care.
contain a discussion of an unapproved/
investigative use of a commercial product/ PIR was established by the American Academy of Pediatrics (AAP) in 1979
device. with the original purpose of providing useful, valuable, and up-to-date information

Vol. 38 No. 1 JANUARY 2017 1


for pediatricians in practice. Founding Editor-in-Chief, Board and national experts believe are important for practic-
Dr Robert Haggerty, basically said the overriding mission of ing pediatricians but are not covered by the ABP MOC speci-
the journal was to synthesize what pediatric experts saw, heard, fications. Another expansion is the number of “Index of
read, and reported and to publish that synthesis in practical Suspicion” cases per issue. This feature is extremely popular.
review articles for general pediatricians. Authors were invited Case submissions are growing in number, as is the backlog
based on their national expertise. Soon, invited experts sought of cases to be published. To accommodate this backlog, PIR
permission to have their trainees serve as coauthors. Over will now publish 6 cases per issue.
time, the journal became a resource for junior faculty to In the spirit of systems-based practice, acknowledgements
author a peer-reviewed case report or review article, giving of various teachers and trainers are in order. Dr Hugh Allen,
them a chance to develop a scholarly career. Meanwhile, Deputy Editor, serves as our scholarly statesman. Associate
thanks to a generous grant, PIR was provided free of charge Editors Drs Deepak Kamat and Philip Fischer of “Index of
to pediatric residents throughout the country, a service that Suspicion” and Associate Editors Dr Henry Adam and
helped residents synthesize what they saw, heard, read, and 2016 AAP Education Award Winner Dr Janet Serwint of
reported. In a sense, the journal developed into a “resi- “In Brief” continue to add educational variety to the
dency” primer for pediatricians-to-be while mentoring med- journal. Dr Mark Weems, having successfully completed
ical authors-to-be. his editorial fellowship, has taken on the role of Associate
Recently, the Executive Board of PIR has witnessed a Editor for “Visual Diagnosis.” Associate Editor Dr Rani
trend of submitted articles containing material that has not Geriege replaces Dr Paula Algranati, who we thank for
been properly cited, thereby creating an impression of false overseeing all those edifying CME questions these past
authorship. PIR relies on authors to synthesize what they years. The PIR Editorial Board suggests authors and topics,
see, hear, read, and have reported. Skipping the synthesis reviews submissions, and advises the Executive Editorial
part short circuits the personal touch that leads to effective Board. Managing Editor Luann Zanzola and Publications
teaching and training. Furthermore, author submissions Editor Sara Strand fine-tune the journal and Director,
that contain unacknowledged sources demean the ACGME Division of Journal Publishing Joseph Puskarz, provides
competencies of professionalism, communication, medical editorial insight.
knowledge, and systems-based practice. Because such sub- Being Editor-in-Chief of PIR helps me train as a resi-
missions may be the result of incomplete training, the PIR dency program director; being a residency program direc-
Executive Board believes some mentoring is due. In accom- tor helps me teach pediatricians as Editor-in-Chief. PIR
panying commentaries, Deputy Editor Dr Hugh Allen and and residency are uniquely tied together. Although the
Associate Editor Dr Mark Weems give their personal spin on grant that provided the printed journal to residents ended a
plagiarism. few years ago, the AAP has continued offering the journal
In the spirit of practice-based learning and improve- online to residents. Thanks to residents and residency
ment, PIR continues to evolve. Although continuing medical program directors continually telling PIR of the value of
education (CME) content is based on the American Board of the printed journal, AAP will resume providing the printed
Pediatrics (ABP) list of specifications for maintenance of journal to residents, starting with this issue. As residents
certification (MOC), the ABP recently streamlined that list, read this issue, they will add 1 more experience in a long-
which gives PIR the opportunity to cover more subjects. One established and enduring process of training to teach and
of the Accreditation Committee on Continuing Medical Edu- teaching to train.
cation (ACCME) missions is to address physicians’ “gaps” in
medical knowledge and practice. Our journal has begun Joseph A. Zenel, MD
publishing more review articles on subjects that our Editorial Editor-in-Chief

2 Pediatrics in Review
Commentary
Plagiarism in Review
For more than 35 years, the editors of Pediatrics in Review (PIR) have turned to
content experts to write review articles that can be used by pediatricians in daily
practice. We also understand that our “Index of Suspicion” and “Visual Diagnosis”
sections provide opportunities for young physicians to build their portfolios, and
we welcome case submissions by junior faculty and residents when supervised by
more experienced pediatricians. When I joined the Editorial Board 2 years ago as
the first PIR editorial fellow, I was shocked by a disturbing trend in submitted
articles: plagiarism.
Plagiarism is not unique to PIR. In fact, it is so prevalent in medical
literature that Miguel Roig’s 2013 PubMed search for “plagiarism” resulted
in 1,086 articles. (1) As of August 24, 2016, that number has risen to 1,461.
Plagiarism threatens the authority of our peer-reviewed journal and puts our
publisher, the American Academy of Pediatrics (AAP), at risk for copyright
infringement. In an effort to prevent plagiarized material from being
published, all manuscripts submitted to PIR are screened with Crossref
Similarity Check, powered by iThenticate (Crossref, Lynnfield, MA, and
Turnitin, LLC, Oakland, CA) and compared against a database containing
more than 60 billion documents. This process is described in our author
instructions, but we continue to receive plagiarized manuscripts. Therefore,
we feel it is important to share some deeper insight into the issue of
plagiarism.
Reading through a small selection of the 1,461 PubMed results, I recognize
that it is far easier to condemn plagiarism than to define it. There is general
agreement that plagiarism always involves theft or deception; that is, reusing
another’s words or ideas without properly acknowledging the original author.
(2) However, there is variation among journal editors as to exactly what is
acceptable. (1) The issue is further complicated by attempts to characterize
different types of plagiarism, depending on the context. Steven Shafer, former
Editor-in-Chief of Anesthesia & Analgesia describes 5 types of plagiarism:
“intellectual theft, intellectual sloth, plagiarism for scientific English, technical
plagiarism, and self-plagiarism.”(3)
Intellectual sloth is the most common type of plagiarism we see at PIR. This is
a function of poor copy-paste habits. I believe that many writers do not recognize
the flaw in this behavior. After all, it seems to be encouraged in so many aspects of
modern medical communication. Each traditional medical school multiple-choice
examination is, in essence, an exercise in copy-paste; the student is expected to
AUTHOR DISCLOSURE Dr Weems has reproduce, verbatim, the lesson material that was presented in the textbook. In
disclosed no financial relationships relevant to the clinical years, bedside teaching reinforces this behavior when higher grades
this commentary. This commentary does not are awarded to those students who can best regurgitate medical facts rather than
contain a discussion of an unapproved/
investigative use of a commercial product/ to those students who ask the most challenging questions. In the world of
device. inpatient medicine, residents become masters of the electronic medical record

Vol. 38 No. 1 JANUARY 2017 3


copy-paste function. This ensures that all progress note data written on the invited topic. This brings up the challenge of
are reproduced daily until the final note can simply be self-plagiarism. Although we do not discourage an author
retitled as a discharge summary. from reusing his or her own words, the appropriate citation
Peer-reviewed medical literature, however, must main- is necessary to prevent copyright violations and to acknowl-
tain a higher standard; there is no role for the copy-paste edge the coauthors and publisher who first produced the
function. When an author employs copy-paste to “use of the material.
words of another author simply to avoid the effort of writing Technical plagiarism, as described by Shafer, is the
new text,”(3) the author deceives the reader by empirically simple omission of quotation marks when a reference
claiming that the written words are those that have come has been cited word-for-word. (3) This may be a result of
from careful review of the literature that is summarized in sloth or accident, but in either case, it is not acceptable and
the author’s language. The copying of entire paragraphs is remains a misrepresentation of one’s work. Intellectual
the most blatant example and can be easily recognized as theft and plagiarism for scientific English rarely occur in
deception. More frequently, we find mosaic plagiarism in submissions to PIR; these are more applicable to primary
which an author copies multiple sentences from another scientific literature.
source and stitches them together by changing 1 or 2 words. Since we began plagiarism screening of all submissions,
This practice is no less deceptive and erodes the trust that we have come to realize that the process is much more
the reader places in the author. Even a small amount of copy- sensitive than specific. Nearly every submission has several
paste plagiarism can cause a reader to question the authen- phrases identified in the review, and each is examined by a
ticity of an entire article or journal. member of the editorial staff. Common phrases such as “a
We commonly find that Wikipedia (Wikimedia Founda- 15-year-old girl” are ignored. We also see that vital signs,
tion, Inc, San Francisco, CA) is the source for plagiarized laboratory values, and simple clinical descriptions are
material in submitted manuscripts. This occurs two ways. frequently flagged; these are also generally ignored. Any
First, the submitting author may use Wikipedia as a source remaining large areas of flagged text are reviewed by an
and reference it appropriately, but the plagiarism screening editor, matched against the source, and evaluated on a
software recognizes the text as having been copied from case-by-case basis.
another source. Such plagiarism may occur without the If the editors of PIR suspect plagiarism after reviewing
author’s knowledge, that is, the Wikipedia content was the plagiarism screening report, we follow the recommen-
copied from the original source. Second, the author may dations of the Committee on Publication Ethics (COPE).
copy Wikipedia text into the manuscript without reference, Generally, this means that we may allow the author to revise
taking advantage of Wikipedia’s open license to freely copy minor copying of short phrases or add quotation marks as
and reuse material. This is an interesting phenomenon appropriate. More substantial plagiarism, however, results
because Wikipedia does encourage readers to use, copy, in a rejection with possible notification to the author’s
and adapt content. Nonetheless, the license requires proper academic institution. The AAP has a plagiarism policy that
attribution. A second stipulation of the license is that is consistent with COPE guidelines and standardized across
content adapted from Wikipedia must retain the same all AAP publications.
copyright license as the original material. This may suggest I share these thoughts with you as readers and aspiring
that PIR articles containing content sourced from Wikipedia authors. As readers, I hope you continue to trust that PIR
must be licensed to the public, which could violate copyright will offer clinically relevant peer-reviewed content to help
agreements made between the submitting author and the guide your pediatric practice. As authors, I hope that you
AAP. Personally, I feel that Wikipedia should never be cited understand that submitting to PIR should be the culmination
as a source of medical literature. The openly editable content of extensive literature review and meticulous manuscript
is not peer-reviewed, and its reliability varies, depending on preparation. I ask that you review COPE resources available
whoever last updated the content. Citing Wikipedia puts the at http://publicationethics.org/ and consider checking your
credibility of PIR at risk, and the open license requirements manuscript with plagiarism screening software before sub-
may put the AAP in legal jeopardy. mission. Several screening programs are available, but you
As previously described, PIR seeks authors who are can use Crossref Similarity Check, powered by iThenticate,
leaders in their field and expects that they have previously by paying a usage fee for each document at http://www.

4 Pediatrics in Review
ithenticate.com/products/crosscheck. This will protect you Mark Weems, MD
from unknowingly submitting plagiarized material and will Associate Editor
ensure that your manuscript moves quickly through plagia-
rism screening so you can share your work with PIR readers References for this article are at http://pedsinreview.aappub-
across the world. lications.org/content/38/1/3.

Vol. 38 No. 1 JANUARY 2017 5


Commentary
Then What?
At all universities, a code of academic integrity is in place that defines academic
misconduct as falsification, fabrication, or plagiarism. Ours is a journal that
concentrates on review articles, interesting cases, or brief commentaries and,
thus, the falsification and fabrication legs of the 3-legged stool are generally not
applicable. Nonetheless, they can occur.
Dr Robert A. Good was a pioneering genius in the field of immunology. He
performed the world’s first bone marrow transplant for severe combined
immunodeficiency disease in 1968. (1) The patient who received that transplant
is still living and is healthy. This miracle treatment opened the pathway for
many applications that today seem commonplace. He received the Lasker award
in 1970 for his work and was in line for a Nobel Prize. His work moved to the
Sloan-Kettering Institute for Cancer Research in New York. There, one of his
fellows, William Summerlin, said he had developed a technique for trans-
planting skin from black mice to white mice and showed that melanocytes
migrated to the white mice, turning their skin to a grey color. Unfortunately,
unbeknownst to Dr Good, this fellow’s success was accomplished by painting
the mouse skin with a marker pen, true research fraud. This is falsification. It
cost Dr Good his job and reputation.
Dr John Darsee (2) was a cardiac researcher at Harvard and Emory universities
who published more than 80 papers that contained fake data. He wrote well and
often but without data to support his hypotheses. This is fabrication. His very
prominent coauthors were mortified and stained by the association.
Let us now progress to the primary problem we are encountering at Pediatrics
in Review: plagiarism. Most high schools, colleges, and universities have an honor
code that specifically prohibits copying the work of others. All medical schools have
a code of academic integrity that prohibits the same. Society expects the same, as
reflected by responses to a recent national convention speech found to contain word-
for-word passages from a previous national convention speech. As a journal, we have
the responsibility to hold a high ethical standard that does not allow copying the work
of others and claiming it to be one’s own. Dr Weems’ commentary nicely addresses
this.
If we detect plagiarism, generally through a software program and occasionally
from reviewers, we are obligated to take certain steps. First, we give the author (via
registered mail) an opportunity to respond. Next we weigh the responses. If, as
a group, the Editorial Board determines that entire paragraphs, tables, and other
items (not bibliography, individual words, standard case report style [see Weems’
commentary]) are plagiarized, the article is rejected. If other articles from the same
AUTHOR DISCLOSURE Dr Allen has disclosed author are also plagiarized, they are publicly retracted. The author and his/her
no financial relationships relevant to this institution are notified, and the author is told not to submit any work to any of the
commentary. This commentary does not
American Academy of Pediatrics journals.
contain a discussion of an unapproved/
investigative use of a commercial product/ Once notified, the author’s school of medicine, which has a compliance office,
device. investigation committee, and due process, can review the situation, usually at the

6 Pediatrics in Review
request of the Dean, and make their own determination, he or she will be hurt. Descent into the cesspool of
which is forwarded to the Dean for determination of out- academic dishonesty might result in drowning or if the
come (which can be dismissal). This not only applies to the author and coauthors emerge from the pool, they will never
lead author but to coauthors as well. There went the Nobel smell like a rosebud.
Prize. DON’T DO IT!
This is serious business. We as editors must be account- Hugh D. Allen, MD
able to our readership. Authors are accountable to the Deputy Editor
integrity of their work. Their institution must be account-
able as well. When an author falls off the 3-legged stool of References for this article are at http://pedsinreview.aappubli-
academic integrity due to 1 or more of the legs being fake, cations.org/content/38/1/6.

Vol. 38 No. 1 JANUARY 2017 7


Fungal Skin Infections
Aditya K. Gupta, MD, PhD, FRCP(C), FAAD,*† Melissa A. MacLeod, MSc,† Kelly A. Foley, PhD,† Gita Gupta, MD,‡
Sheila Fallon Friedlander, MDx
*Division of Dermatology, Department of Medicine, University of Toronto, Toronto, Ontario, Canada

Mediprobe Research, Inc, London, Ontario, Canada

Wayne State University, Detroit, MI
x
Dermatology and Pediatrics, Pediatric Dermatology Training Program, University of California at San Diego School of Medicine, Rady Children’s Hospital,
San Diego, CA

Education Gap
Most pediatricians appear to be familiar with candidal diaper dermatitis,
but there is a lack of knowledge about other, less common fungal
infections in children.

Objectives After completing this article, readers should be able to:

1. Recognize the clinical presentations of different fungal infections in


children.
2. Know the differential diagnosis of various fungal skin infections.
3. Know what diagnostic tests can be used to confirm infection.
4. Be aware of available treatment options and how to manage the
infections appropriately.
AUTHOR DISCLOSURE Dr A. Gupta has
disclosed that he is on the Speakers’ Bureaus
of Valeant, Janssen, Novartis, and Bayer; he is a
consultant for Anacor, Sandoz, and Moberg
INTRODUCTION Pharma; and he is a clinical trials investigator
for Valeant Canada, Nuvolase, Bristol Meyers
Candidal diaper dermatitis is the most common fungal infection of childhood. Squibb, Eli Lilly, Merck, Novartis, Janssen, and
This yeast infection almost always secondarily invades diaper-area skin that has Allergan. Ms MacLeod and Dr Foley have
disclosed that they are employees of
been damaged by an irritant contact dermatitis from maceration, urine, and/or Mediprobe Research, Inc, which conducts
stool. Children in the preschool-age group who no longer wear diapers are clinical trials under the supervision of Dr. A.
more likely to develop tinea infections, particularly tinea capitis. Tinea refers to Gupta. Drs G. Gupta and Fallon Friedlander
have disclosed no financial relationships
dermatophyte infections in the epidermis and areas high in keratin, such as the
relevant to this article. This commentary does
hair and nails. In prepubertal children, tinea capitis and tinea corporis are most contain a discussion of an unapproved/
common; in adolescence, tinea pedis (TP), tinea cruris, and tinea unguium investigative use of a commercial product/
device.
(onychomycosis) are more common. (1) Yeast infections other than candidal
diaper dermatitis, including pityriasis versicolor (PV) (formerly known as tinea ABBREVIATIONS
versicolor) and mucocutaneous candidiasis (MC), may also occur. Chronic MC CMC Chronic mucocutaneous candidiasis
(CMC) is a rare, usually inherited disorder. PV is a common infection in adolescents HIV Human immunodeficiency virus
and adults that usually affects the sebum-prone areas (face, chest, back). Fungal id Dermatophytid
KOH Potassium hydroxide
infections can be a substantial source of morbidity in the pediatric population,
MC Mucocutaneous candidiasis
accounting for about 15% of pediatric outpatient visits in the United States. (2) PCR Polymerase chain reaction
This article reviews the epidemiology and clinical presentations of tinea in- PV Pityriasis versicolor
fections (capitis, corporis, pedis, cruris, unguium), PV, and MC in children. The TP Tinea pedis

8 Pediatrics in Review
differential diagnosis and methods for confirming diagnosis audouinii was the major source of tinea capitis in North
based on clinical presentation are discussed. Recommended America. (6) Subsequently, epidemiology shifted and cur-
treatment options for each type of infection are specified rently about 95% of tinea capitis in North America is caused
(Table 1). Of note, many recommendations are off-label, as the by Trichophyton species (predominantly T tonsurans). Micro-
safety of many agents has not been established for children. sporum species, (7) usually transmitted by pets, causes the
remainder of the cases. However, in central and southern
Europe as well as in developing countries, M canis is the
TINEA CAPITIS
most common causal species. (7) It is important for clini-
Epidemiology cians to be aware of the predominant pathogen in their
Tinea capitis, a communicable fungal infection of the scalp communities because this has implications for optimal
and hair shaft, is the most common fungal infection in treatment choice.
children. (3) The prevalence ranges from less than 1% in
western Europe to as much as 50% in Ethiopia where the Clinical Presentation
infection is endemic. (4) In North America, the prevalence is Tinea capitis may be difficult to diagnose because clinical
estimated to range from 3% to 8% in children. It is unclear signs may be subtle and can vary substantially from child to
whether it is increasing, but immigrant populations, par- child. Symptoms may include scaling, alopecia, broken hair
ticularly those from Africa, are at higher risk. (5) Tinea shafts at the scalp, erythema, pustules, and/or large boggy
capitis most often affects children between ages 3 and 9 scalp masses. Patients may complain of pruritus or tender-
years, those of African heritage, those of low socioeconomic ness, and occipital and posterior cervical adenopathy are often
status, and those residing in urban settings and/or crowded present (Fig 1). A nonspecific, eczematous, pruritic eruption
living conditions. (1)(4) Prior to the 1950s, Microsporum may be noted on the trunk and extremities either before or

TABLE 1. Summary of Treatment Recommendations for Tinea Infections


in Children*
INFECTION FIRST-LINE TREATMENT ALTERNATIVES TERTIARY OPTIONS

Tinea capitis Oral antifungals: terbinafine, griseofulvin Adjunctive agents: selenium sulfide Itraconazole, fluconazole
(Microsporum canis) shampoo, ketoconazole shampoo
Tinea corporis/ Topical antifungals: butenafine, ciclopirox, Oral antifungals (resistant or severe
Tinea cruris clotrimazole, miconazole, terbinafine, infection): terbinafine, griseofulvin,
tolnaftate itraconazole, fluconazole
Tinea pedis Topical antifungals: butenafine, clotrimazole, Topical antifungals: ciclopirox Urea cream
miconazole, terbinafine
Oral antifungals (resistant or recurrent
infection): terbinafine, itraconazole,
fluconazole
Pityriasis versicolor Zinc pyrithione, selenium sulfide, or Oral antifungals (resistant, recurring, serious
ketoconazole shampoos infection): itraconazole, fluconazole
Other topical antifungals: clotrimazole,
ketoconazole, miconazole, terbinafine
Tinea unguium Topical antifungals: ciclopirox, efinaconazole, Oral antifungals (severe infection): Itraconazole, fluconazole
(onychomycosis) tavaborole terbinafine
MC – Oropharyngeal Mild: Clotrimazole troches, miconazole, Fluconazole-resistant infections: Amphotericin B
candidiasis nystatin suspension itraconazole, posaconazole suspension
Moderate-severe: fluconazole
MC – Esophageal Fluconazole Intravenous fluconazole, echinocandin Amphotericin B
candidiasis/CMC (anidulafungin, caspofungin, micafungin)
Fluconazole-resistant infections:
itraconazole, voriconazole, amphotericin
B, or an echinocandin

*Some agents may be used off-label usage in children or be approved for particular ages. See Table 3 for details.
CMC¼chronic mucocutaneous candidiasis, MC¼mucocutaneous candidiasis.

Vol. 38 No. 1 JANUARY 2017 9


during treatment; this is known as an autoeczematization or Diagnostic Tests
dermatophytid (id) eruption. Tinea capitis typically presents Clinical diagnosis should be confirmed via either potassium
as 1 of 6 clinical patterns: gray type, black dot, diffuse scale, hydroxide (KOH) microscopy or culture. Culture is prefer-
pustular type, favus, and kerion. Gray type is characterized able because speciation is provided, allowing determination
by circular patches of alopecia and marked scaling with or of the most appropriate treatment option. Polymerase chain
without erythema. (4)(8) Black dot presents with patches of reaction (PCR) evaluation of dermatophyte infections has
alopecia and is dotted with broken hair stubs (“black dots”). become much more cost effective and “kits” are now avail-
Diffuse scale is characterized by widespread scaling and is able, which is likely to lead to wider availability of this
dandruff-like, with or without erythema. Pustular type pre- exceedingly rapid and sensitive test in the next few years.
sents as alopecia with scattered pustules, scaling, and lymph- At this time, PCR appears more sensitive for nail and skin
adenopathy. Favus has distinctive yellow cup-shaped crusting infections than for hair samples. (12) Wood’s light exami-
around the hair called scutula, along with patchy alopecia nation causes Microsporum species to fluoresce, but most
and generalized scale; this variant is extremely rare and not infections in North America are caused by T tonsurans, which
usually seen in the United States. Kerion is a boggy tumor does not fluoresce. (13) Pathogens that do fluoresce include
with pustules, lymphadenopathy, erythema, and tenderness. Microsporum species and Trichophyton schoenleinii. (7) Under
(4)(8) It is the most severe inflammatory response and can microscopic analysis, an infected hair can present with
be caused by either T tonsurans or M canis. (9) When chil- mycelium (mass of fungal hyphae) on the external surface
dren present without distinguishing characteristics of tinea of the hair shaft (ectothrix) or with mycelium within the hair
capitis such as black dot alopecia or kerion, diagnosis may be shaft (endothrix). (7) A favus infection presents with fungal
simplified by recognizing clusters of symptoms. Hubbard hyphae and characteristic airspaces within the hair shaft. (7)
(10) found that children with adenopathy, alopecia, pruritus, Wood’s light analysis takes minutes to complete compared
and scaling were most likely to have a positive culture. Some with 1 to 4 weeks required for culture results, which are
dermatologists now use scalp dermoscopy (visualization of accompanied by low culture-positive rates, all of which may
scalp and hair shaft with magnification and light) to increase delay treatment and increase the spread of infection. (10)
diagnostic accuracy; corkscrew and comma-shaped hairs are A reasonable course is to start treating children with
frequently seen in fungal scalp infections. (11) typical presentations before culture confirmation, although
a culture should be attempted. Samples may be obtained
either from plucked hairs or cotton swabs that have been
Differential Diagnosis premoistened and rolled over the affected site and are
Because of its broad and varying symptoms, tinea capitis inoculated into transport culture. (14)
has a substantial differential diagnosis. Among the possi- Kerion (abscesses filled with purulent exudate) should be
bilities are alopecia areata, atopic dermatitis, bacterial scalp treated aggressively with systemic antifungal medication
abscess, seborrheic dermatitis, trichotillomania, traction alo- pending laboratory results because if left untreated, perma-
pecia, psoriasis, lichen planopilaris, lupus erythematosus, syph- nent hair loss and scarring may occur. Unfortunately, the
ilis, and Langerhans cell histiocytosis (Table 2). (1)(3)(6) degree of inflammation noted in a kerion is not linked to the
fungal burden, and cultures may sometimes be negative.
However, every attempt should be made to swab the kerion
area as well as other areas of the scalp with a cotton swab.
Tinea capitis infection may spread from the scalp to other
areas of the body (eg, causing tinea corporis) and secondary
bacterial infections (eg, Staphylococcus aureus) may occur. (15)
If children are unlikely to have an infection (eg, no adenop-
athy and scaling), experts recommend confirming infection
via KOH microscopy or a culture before treatment. (4)

Treatment
Systemic treatment is required to penetrate hair shafts.
Traditionally, griseofulvin was considered the treatment of
Figure 1. Tinea capitis. Photo courtesy of Dr Avner Shemer, The Chaim
choice, (16) but a Cochrane collaborative analysis found that
Sheba Medical Center Israel. terbinafine, fluconazole, and itraconazole are as effective as

10 Pediatrics in Review
TABLE 2. Differential Diagnosis of Tinea Capitis (1)(2)(6)
DISORDER DISTINGUISHING CHARACTERISTICS OF CLINICAL PRESENTATION

Alopecia areata Patches of hair loss; total loss of hair; fine miniature hair growth; exclamation point hairs; can involve
eyebrows, eyelashes, beards; possible nail pitting
Uncommon: scaling, crusting, inflammation (consider infection, other diagnoses)
Atopic dermatitis Personal or family history of atopy, may appear on face
Uncommon: alopecia, large posterior occipital or cervical nodes, erythema of scalp usually minimal with
diffuse faint scales common
Bacterial scalp abscess Culture should be used to distinguish from kerion
Seborrheic dermatitis Greasy scaling, typical distribution includes nasolabial folds, hairline, eyebrows, postauricular folds, chest
Uncommon: alopecia and significant lymphadenopathy
Trichotillomania Often involves eyelashes and eyebrows, hairs of varying lengths, scaling uncommon, large geometric
shapes of alopecia present
Traction alopecia Hair loss in areas under tension; folliculitis may also be present
Psoriasis Gray or silver scaling that extends beyond scalp line, nail pitting, family history, involvement of other sites
Lichen planopilaris Often affects skin, mucosa, and nails; no hair follicles seen in areas of hair loss; slowly progressive
Lupus erythematosus Involves skin, especially face and sometimes connective tissue of internal organs; discoid lesions can lead
to scarring
Syphilis Involves other areas of the body, not pruritic, scaling uncommon
Langerhans cell histiocytosis May involve buttocks, liver problems causing jaundice, fluid in the belly, bulging eyes or eye problems

griseofulvin, with shorter periods of treatment with newer Adjunctive therapy with either selenium sulfide sham-
antifungals achieving similar results to griseofulvin. (17) For poo (1% or 2.5%) (41) or ketoconazole shampoo should be
Trichophyton species, terbinafine is preferable, but this agent used to decrease the spread of infection. (1)(42) Because
is not as effective as griseofulvin for Microsporum species. tinea capitis is communicable, children should not at-
(17)(18) When a child presents with a lesion highly suspi- tend school or child care until treatment has started. Once
cious for Microsporum species (eg, infected cat or dog at treatment has begun, the child may return to school but
home, and/or lesion fluoresces under Wood’s lamp), gris- should not share combs, brushes, helmets, or other items
eofulvin should be used. Most experts believe that effective that come in contact with the scalp or play contact sports for
treatment doses of griseofulvin should be higher than 14 days to avoid transmission. (1) Household members
advised in the package insert (Table 3). If griseofulvin is should be queried and clinically examined for signs and
not available or terbinafine is preferred, the duration of symptoms if possible and mycologically tested if these
treatment for Microsporum species may be longer compared exist. The use of selenium sulfide shampoo or ketoconazole
to the duration for Trichophyton species. The duration of shampoo prophylactically (2 times/wk for 2-4 weeks) is
treatment for terbinafine is generally 4 to 6 weeks, and controversial, and no clear evidence-based data support
continuing treatment for 2 weeks after symptoms resolve its use for this purpose, although some experts recommend
may be beneficial. (8) Griseofulvin therapy is generally used this. Some also recommend the same prophylaxis for peo-
for 8 weeks, but many experts reevaluate a child after 4 to 6 ple outside the home in close contact with the child. (1)(4)
weeks of therapy to consider discontinuation. Some systemic Close contacts include other children seen daily, such as in a
antifungals, such as itraconazole and fluconazole, have been classroom or child care. Although this process may seem
successfully used for pediatric tinea capitis, but such use is daunting, families at least should be informed so that chil-
off-label, and a large multinational study investigating flu- dren can be monitored for signs and symptoms and given
conazole reported cure rates below those seen with either the option to engage in prophylactic treatment.
griseofulvin or terbinafine. (40) Nonetheless, fluconazole In some cases, patients may develop an immune re-
has been widely used in children for candidiasis and may be sponse to the fungus triggered by treatment, known as
an option when other agents are either not available or not an id reaction. It often presents as a pruritic, papular, or
covered by the patient’s insurance plan (Table 3). vesicular rash on the face and body and may be alleviated by

Vol. 38 No. 1 JANUARY 2017 11


TABLE 3. Antifungal Agents for Fungal Infections in Children
APPLICABLE MONITORING OTC OR
ANTIFUNGAL DOSAGE DURATION INFECTIONS NOTES GUIDELINES PRESCRIPTION

Topical:
Butenafine 1% cream, twice 1 week Tinea corporis, tinea Children 12þ years OTC
hydrochloride daily (19) pedis, tinea cruris
Ciclopirox 0.77% cream, twice 1 week Tinea corporis, tinea Safety in children <10 OTC
daily (20) pedis, tinea cruris years has not been
established
When topical
clotrimazole and
miconazole fail
8% lacquer, daily 48 weeks Onychomycosis Considered safe for 12þ Prescription
with weekly years
professional
debridement
Clotrimazole 1% cream, twice 4 weeks Tinea corporis, tinea Children 2þ years OTC
daily (21) pedis
1% cream, twice 2 weeks Tinea cruris Children 2þ years OTC
daily (21)
1% cream, twice 1-2 weeks Pityriasis versicolor Children 2þ years OTC
daily (22)
Efinaconazole 10% solution, daily 48 weeks Onychomycosis Safety and efficacy not Prescription
established
Ketoconazole 2% once (23) Once Tinea capitis To decrease spread of OTC
shampoo infection as an
adjunct therapy
Safety in children not
established
2% daily (22) 3-14 days, up to Pityriasis versicolor See above for safety OTC
4 weeks Following treatment, use
monthly for 3 months
to prevent recurrence
Miconazole 2% cream, twice 4 weeks Tinea corporis, tinea Children 2þ years OTC
daily (24) pedis
2% cream, twice 2 weeks Tinea cruris Children 2þ years OTC
daily (24)
2% cream, once or 2 weeks Pityriasis versicolor Children 2þ years OTC
twice daily (22)
Selenium 1% or 2.5% 2 times Duration of oral Tinea capitis To decrease spread of OTC for 1%
sulphide a week (25) treatment infection as an
shampoo adjunct therapy
Safety in children <12 Prescription
years has not been for 2.5%
established for 2.5%
1% (shampoo) or 1-2 weeks, up to Pityriasis versicolor See above for safety OTC
2.5% (lotion) 4 weeks Following treatment,
daily (22) use monthly for 3
months to prevent
recurrence
Tavaborole 5% solution daily 48 weeks Onychomycosis Safety and efficacy not Prescription
established
Terbinafine 1% cream twice 1-2 weeks Tinea corporis, tinea Children 12þ years OTC
daily (26) pedis, tinea cruris
When topical clotrimazole
and miconazole fail
1% cream once or 1-2 weeks Pityriasis versicolor Children 12þ years OTC
twice daily (22)
Continued

12 Pediatrics in Review
TABLE 3. (Continued )

APPLICABLE MONITORING OTC OR


ANTIFUNGAL DOSAGE DURATION INFECTIONS NOTES GUIDELINES PRESCRIPTION

Tolnaftate 1% cream twice 4 weeks Tinea corporis, tinea Children 2þ years OTC
daily (27) pedis, tinea cruris
Urea cream 39% twice daily Not available Tinea pedis Safety in children not OTC
(28) established
When topical terbinafine
and ciclopirox fail
Oral:
Clotrimazole 10 mg 5 times a 2 weeks MC Children 3þ years (need Prescription
troches day (29) to swallow them
properly)
Fluconazole 3-6 mg/kg daily (30) 2-3 weeks, Tinea capitis, For mild-to-severe MC Liver tests, Prescription
(tablet) 12 mg/kg daily for additional 2 tinea corporis, Extensive or resistant exercise
neonates (31) weeks after tinea pedis, tinea infections when caution if liver
Loading dose first symptoms tinea cruris, not tinea capitis dysfunction
day resolve MC
3-6 mg/kg weekly FN: 12 weeks Onychomycosis Not approved for See above Prescription
(32) TN: 26 weeks onychomycosis
Griseofulvin* 10 mg/kg daily (33) 6-8 weeks tinea Tinea capitis, tinea Children 2þ years Prescription
(microsize) up to 20-25 mg/kg capitis, 4-8 corporis, tinea Extensive or resistant
daily weeks tinea pedis, tinea cruris tinea infections when
pedis not tinea capitis
Itraconazole 5 mg/kg daily 4-6 weeks MC, tinea capitis, Safety not established in Liver tests, Prescription
(capsules) (1)(31) tinea corporis, children exercise
tinea pedis, For fluconazole- caution if liver
tinea cruris, resistant MC dysfunction
onychomycosis Extensive or resistant
tinea infections when
not tinea capitis
Pulse (1 week on FN: 2 pulses Efficacy and safety not See above, do Prescription
therapy, 3 weeks TN: 3 pulses established in children not use if
off) congestive
10-20 kg: 50 mg heart failure
every other day,
3x/week; 20-30 kg:
100 mg/day;
30-40 kg:
100 mg/day
alternating
200 mg/day;
40-50 kg: 200
mg/day; >50 kg:
200 mg 2x/day
(32)
Nystatin 100,000 U/mL, 4-6 48 hours after MC Prescription
suspension mL, 4 times a symptoms
day for older resolved
children, 2 mL
for younger
children (34)
Terbinafine 125 mg/day for 6 weeks Tinea capitis, tinea Children 4þ years ALT, AST, do not Prescription
(oral granules) <25 kg corporis, tinea use if liver
187.5 mg/day for pedis, tinea cruris dysfunction
25-35 kg Extensive or resistant
250 mg/day for tinea infections when
>35 kg (35) not tinea capitis
Continued

Vol. 38 No. 1 JANUARY 2017 13


TABLE 3. (Continued )

APPLICABLE MONITORING OTC OR


ANTIFUNGAL DOSAGE DURATION INFECTIONS NOTES GUIDELINES PRESCRIPTION

Terbinafine Same dosage as FN: 6 weeks Onychomycosis Efficacy and safety not See above Prescription
(tablets) above TN: 12 weeks assessed in children
Amphotericin B 0.25-1.5 mg/kg Based on MC Safety in children not Renal, liver, Prescription
daily (36) severity established hematologic
When clotrimazole, tests, monitor
nystatin, fluconazole, electrolytes
itraconazole fail
Anidulafungin 0.75-1.5 mg/kg 2 weeks after MC Safety not established in Prescription
daily (37)(38) last positive children <16 years
culture When clotrimazole,
nystatin, fluconazole,
itraconazole fail
Caspofungin Loading dose of About 2 weeks, MC Safety not established in Prescription
70 mg/m2, followed may be more children <12 months
by 50 mg/m2 When clotrimazole,
daily nystatin, fluconazole,
itraconazole fail
Micafungin 2-3 mg/kg daily for About 2 weeks, MC Safety not established in Prescription
children £ 30 kg may be more children <4 months
2-2.25 mg/kg daily or less When clotrimazole,
for children >30 nystatin, fluconazole,
kg (39) itraconazole fail

ALT¼alanine aminotransferase, AST¼aspartate aminotransferase, FN¼fingernails, MC¼mucocutaneous candidiasis, OTC¼over-the-counter,


TN¼toenails.
*Griseofulvin is no longer available in Canada. Other than griseofulvin, all of the drugs listed have been approved and are available for use in the United
States and Canada, but they are not all indicated for use in children, as specified in the Notes column of the table.
This review is limited to the United States and Canada. Information is based on package inserts obtained from the National Institutes of Health, United
States National Library of Medicine, DailyMed; United States Food and Drug Administration Approved Drug Products Database; and Health Canada’s Drug
Product Database.
Please check the regulatory status of each drug in your jurisdiction. Check for current dosing and monitoring guidelines. Table 3 is presented as a guide only.

topical corticosteroids and systemic antihistamines. How- among wrestlers (tinea corporis gladiatorum), where it is
ever, an id reaction does not necessarily require discontinu- often limited to the neck and arms but may also involve the
ing treatment and may occur before institution of therapy. scalp. (2)
(13)(43) This reaction should be distinguished from a drug
reaction, but id reactions are much more common. (5) Clinical Presentation
Tinea corporis presents as a single or multiple red, scaly
TINEA CORPORIS papules (sometimes follicular) that may spread and com-
bine, forming plaques that tend to be annular and clear in
Epidemiology the center. (2) The plaques generally are limited to a few
Tinea corporis is a dermatophyte infection of the body, often sites on the body and are usually unilateral (Fig 2). (2) Mild
referred to as ringworm. It can be caused by any dermato- erythema, edema, vesicles, pustules, or bulla formation can
phyte that infects humans. Among young children, acute occur, and the sites may be pruritic. (2) The presence of
infections may be caused by M canis from contact with dog pustules and inflammation tends to be more common with
or cat carriers. (2) However, in North America, it is most infections caused by M canis whereas follicular infections
commonly caused by Trichophyton species, especially T are often caused by T rubrum. (2) In addition, follicular
tonsurans and Trichophyton rubrum. (2)(3)(4) Tinea corporis inflammatory reactions are more common among patients
may be spread by close body contact and has been found to who have used topical corticosteroids. (2) Because the use
be more prevalent in warm and moist environments and of topical corticosteroids (eg, when atopic dermatitis is

14 Pediatrics in Review
suspected) may alter the appearance of tinea corporis (tinea Diagnostic Tests
incognito), physicians should use clinical judgement in Diagnosis can be confirmed with KOH microscopy or a
obtaining a sample for KOH microscopy for annular scaly culture, although cultures are usually not needed.

skin lesions, particularly for those lesions that have an


atypical appearance. (4) Treatment
Topical antifungals are generally effective and should be
used for 1 additional week after symptoms resolve. (2) Some
Differential Diagnosis
have suggested that butenafine and terbinafine are more
The differential diagnosis of tinea corporis includes gran-
effective than miconazole and clotrimazole. (3) Topical corti-
uloma annulare, nummular eczema, erythema multiforme, costeroids eventually worsen the infection and should not be
erythema annulare centrifugum, psoriasis, pityriasis rosea, used. When topical treatments fail or infections recur, oral
subacute cutaneous or discoid lupus, atopic dermatitis, antifungals may be needed. This is often the case for those
candidiasis, fixed drug eruption, early Lyme disease, and who have had prolonged pretreatment with topical cortico-
seborrheic dermatitis. (1)(2) These conditions often have steroids, those who have follicular infections, and for indi-
viduals who are immunocompromised because they often
several characteristics that distinguish them from tinea
have extensive and severe infections. Because tinea corporis is
corporis. For example, granuloma annulare is smooth; has
more common in warm and humid environments, the skin
no scaling, vesicles, pustules, or pruritus; and is often nod-
should be kept cool and dry to promote healing. (2)(4)
ular (dermal with no epidermal component) and present
on the dorsum of the hands or feet. (1) Histologically the
TINEA PEDIS
epidermis is not affected; rather, inflammation is in the
dermis. (4) Nummular eczema is less likely to have central Epidemiology
clearing and has more convergent scaling while erythema TP, known as athlete’s foot, is largely caused by T rubrum
and Trichophyton mentagrophytes. Athlete’s foot is most
multiforme is characterized by acute-onset target lesions
common among adolescents and is relatively rare among
(sometimes oral) without scaling. (1) For additional differen-
prepubertal children. Prevalence is estimated to be approx-
tiating characteristics, please refer to Ely et al (1) and Kelly. (4) imately 3% to 9% in children. (44)(45)(46)(47)(48) Because
TP is uncommon among children, it is often misdiagnosed.
(49) This can be problematic because treatment with topical
corticosteroids may alter the clinical appearance, making
subsequent diagnosis difficult. (50)

Clinical Presentation
Symptoms of TP include erythema, scaling, fissures, mac-
eration, and pruritus between the toes extending to the
soles, borders, and sometimes the dorsum of the foot
(Fig 3). Onychomycosis may occur concomitantly. (1) The
3 typical presentations are intertriginous dermatitis (inter-
digital), “moccasin” pattern, and vesicular. Interdigital TP
is the most common presentation and is characterized by
scaling (usually between the fourth and fifth toes (9) because
for anatomic reasons this web space tends to be the most
occluded), maceration, pruritus, and fissuring of the lateral
toe web spaces that may spread to the soles and dorsum of
the foot. (51) This presentation often starts in the toe web
where maceration and moisture are present. (9) Moccasin
TP is typically chronic and is characterized by dry scaling
Figure 2. Tinea corporis. Photo courtesy of Dr Avner Shemer, The Chaim
patches or hyperkeratotic plaques, erythema on the soles
Sheba Medical Center Israel. and border of the foot, and possibly tenderness or pruritus.

Vol. 38 No. 1 JANUARY 2017 15


Treatment
Topical antifungals such as azoles or allylamines are usually
effective in treating TP, with the duration of therapy rang-
ing from 1 to 4 weeks based on the agent (Table 3). First-
line treatment is generally topical clotrimazole, miconazole,
naftifine, or terbinafine. If such therapy is unsuccessful, the
patient should be reevaluated and other topical agents can be
used, including ciclopirox (52) and butenafine hydrochlo-
ride. (3) Upon failure of other topical treatments, high-
potency urea cream is a consideration as adjunctive therapy.
This agent functions as both a keratolytic and humectant
(to keep moist) and is useful in removing scale. (28)(52)
Patients should thoroughly dry their feet before applying
Figure 3. Tinea pedis. Photo courtesy of Dr Avner Shemer, The Chaim
Sheba Medical Center Israel. topical antifungals. Wearing breathable shoes with 100%
cotton socks changed twice daily also is recommended. (51)
Adjunctive treatments such as topical antiperspirants (eg,
(51) This presentation may also involve infection of the nails
aluminum chloride) may be used if there is extreme sweat-
(onychomycosis) and hand (tinea manuum). (9) The vesic- ing. (2) Cool tap water or Burow’s solution soaks may help
ular presentation is characterized by vesicles and pustules relieve discomfort, particularly if secondary bacterial infec-
that are often on the anterior sole or instep but can occur on tion is suspected. (52)
all areas of the foot (51), often accompanied by intertriginous If the infection is extensive or involves the toenails, is
infections. (9) recalcitrant or recurrent, or presents in immunosuppressed
patients or as moccasin TP, systemic therapy (eg, terbina-
fine, fluconazole, itraconazole) may be required. (51)(52)(53)
Differential Diagnosis
TP may appear similar to contact dermatitis, dyshidrotic In moccasin type TP, topical treatment is typically only
useful for reducing the spread of lesions. (9) With extensive
eczema, foot eczema, juvenile plantar dermatosis, and pso-
maceration, bacterial infection (ie, dermatophytosis com-
riasis, although TP is generally more likely to affect the
plex) also may occur, requiring simultaneous antibiotic
intertriginous areas. (3) However, Guenst (51) argues that
therapy. (2)(52) As with tinea capitis, an id reaction may
TP should be included as a differential diagnosis for every
occur with treatment, but the hand is usually the first place
child who presents with a foot rash. Contact dermatitis can
to react in TP. (9)
be distinguished from TP because the distribution gener-
ally matches the footwear. (1) Dyshidrotic eczema uniquely pre-
TINEA CRURIS
sents with vesicles on the lateral aspects of the digits and often
involves the hands. Children with foot eczema may have an Epidemiology
atopic history. Juvenile plantar dermatosis is characterized by Tinea cruris (jock itch) primarily affects adolescent and
shiny, taut skin involving the great toe, ball of the foot, and young adult males; it is less common among children. Risk
heel. Psoriasis usually involves other sites and presents with factors include diabetes, obesity, crowded living conditions,
gray or silver scale, nail pitting or other nail signs, occasionally tight or wet clothing in the groin area, membership in a
arthritis, and possibly a family history. (1) sports team, and a fungal infection on another part of the
body. (54) In North America, jock itch is most often caused
by T rubrum, although Epidermophyton floccosum is also a
Diagnostic Tests common pathogen. (2) It can be spread by fabric such as
In cases with a typical presentation, diagnosis based on clothing, towels, and sheets as well as by direct contact,
appearance may be adequate; when the presentation is especially in warm and humid environments. (2) A person
atypical, KOH microscopy or a culture is recommended. (1) who has tinea cruris may also have tinea pedis because they
KOH microscopy may be difficult with vesicular presenta- can both be caused by T rubrum. (2) Accordingly, the feet
tions because samples should be taken from the root of the should always be examined in a child presenting with tinea
vesicle. (9) cruris. (54)

16 Pediatrics in Review
Clinical Presentation They should be applied as directed on the affected area and
Tinea cruris occurs on the upper-medial thighs with bor- 2 to 3 cm beyond. (54) Oral itraconazole, terbinafine, or
ders that generally spread outwards, possibly extending to fluconazole may be needed to treat extensive or resistant
the buttocks and above the waistline. It usually spares the infection. Griseofulvin is not recommended because it ad-
scrotum (in contrast to candidal infections) and is charac- heres poorly to keratinocytes in the stratum corneum. (54)
terized by edema, erythema, skin-colored (hyperpigmented) Combined treatment with corticosteroids and oral ketocona-
scaling, papules or plaques that can become vesicles or pus- zole is also not recommended. (55) Throughout treatment,
tules, and often maceration. (2) Compared to tinea corporis, the affected area should be kept dry and patients should wear
there is usually less central clearing and inflammation. (2) loose-fitting clothing and use a separate towel for the groin
Tinea cruris is typically very itchy, and scratching of the skin area after bathing. (54)
may create lichenification as well as thickening of the scrotal
skin. This may falsely create the appearance of an infection
TINEA UNGUIUM (ONYCHOMYCOSIS)
involving the scrotum. (2) However, the scrotum may be
involved if candidiasis is also present. Clinical presentation Epidemiology
can be further complicated if the use of topical medications Tinea unguium, widely referred to as onychomycosis to
results in allergic contact dermatitis. (2) account for fungal infection due to dermatophytes, non-
dermatophyte molds, and/or yeasts, is a therapeutically
challenging condition that usually requires systemic ther-
Differential Diagnosis
apy. The pooled prevalence of culture-confirmed onycho-
The differential diagnosis of tinea cruris includes candidal
mycosis in childhood has been estimated at 0.14%. (56) Risk
intertrigo, erythrasma, inverse psoriasis, and seborrheic
factors include concomitant TP, family members with TP
dermatitis. Tinea cruris is generally the only one of these
and/or onychomycosis, and frequent wearing of occlusive
conditions that does not affect the scrotum and penis. (1)
footwear (as in organized sports). The most common caus-
Distinguishing features of candidal intertrigo include in-
ative organisms are the dermatophytes T rubrum and Tri-
volvement of the scrotum, satellite lesions, and uniform
chophyton mentagrophytes. Toenails are more commonly
redness. (1) Erythrasma is typically red-brown and does not
affected, although fingernail onychomycosis caused by yeast
have a border. It can be distinguished from tinea cruris
may be seen in children younger than age 6 years. (57)
under Wood’s light as it fluoresces coral-red. (1)(2) Inverse
psoriasis is red, clearly distinguished by boundaries, and
Clinical Presentation
may involve nail pitting. Seborrheic dermatitis involves
Features of tinea unguium may include thickened and
greasy scaling in the nasolabial folds, hairline, eyebrows,
dystrophic nails, discoloration ranging from whitish to brown,
postauricular folds, and chest; lesions are generally not
onycholysis, and/or destruction of all or part of the nail plate
annular. (1) Tinea cruris may also appear similar to contact
(Fig 4). Distal lateral subungual onychomycosis is the most
dermatitis, lichen simplex chronicus, pityriasis versicolor,
common subtype in all groups, but superficial white ony-
Darier disease, Majocchi granuloma, Langerhans cell his-
chomycosis, which is characterized by white lesions, is more
tiocytosis, and pemphigus vegetans. (54)
common in children. Proximal subungual onychomycosis
may indicate immunocompromise. Concomitant TP, 2 or
Diagnostic Tests more affected nails on the same foot, or unilateral dystrophy
KOH microscopy of skin scrapings from the periphery of of the first and fifth nails may be predictive. (58) When only
the lesion should be used to confirm diagnosis. Hyphae in 1 digit is involved, the possibility of a subungual exostosis
the middle of healthy superficial stratum corneum (known (bony projection) should be considered, and radiologic
as the “sandwich sign”) as well as a deeper parakeratotic or evaluation should be performed, particularly if the patient
hyperkeratotic stratum corneum may be visible if biopsy does not respond to antifungal therapy.
is performed. (54) A culture can also be used to confirm
diagnosis. PCR may be the diagnostic option of choice in Differential Diagnosis
the future. A number of conditions have similar presentations to ony-
chomycosis, including nail trauma, paronychia, psoriasis,
Treatment subungual exostosis, and contact/atopic dermatitis. Wait-
Treatment with topical antifungals, including clotrimazole, ing a short period may lead to resolution of symptoms (ie,
miconazole, tolnaftate, and butenafine, is usually adequate. from trauma) because children have thinner nail plates

Vol. 38 No. 1 JANUARY 2017 17


efficacy of lasers in treating onychomycosis to recommend
their use.
Similar to adults, systemic antifungal medications are
recommended for children with established severe ony-
chomycosis. (62) Terbinafine, itraconazole, and flucona-
zole are prescribed off-label and are effective (Table 3). (32)
Immunocompromised patients should not be started on
systemic antifungals until their other health care clini-
cians are involved in the decision-making because the
benefits of their use for onychomycosis may not outweigh
the risks (eg, drug-drug interactions, hepatic/renal com-
plications, rare marrow effects). Topical antifungal solu-
tions and topical urea creams may be a safer option for
Figure 4. Onychomycosis. Photo courtesy of Dr Avner Shemer, The such patients.
Chaim Sheba Medical Center Israel.
Medications have varying degrees of effectiveness in
treating onychomycosis, with systemic antifungals more
and faster growing nails. Onychomycosis must be con- effective at eliminating fungus (mycologic cure) than topical
firmed with mycologic testing before initiating systemic antifungals. Concomitant TP should be treated with topical
therapy. antifungals. Treating family members for TP and/or ony-
chomycosis and advising patients and families to replace old
footwear, keep feet clean and dry, and launder socks in hot
Diagnostic Tests
water to kill fungal spores can help to prevent recurrent
A positive KOH or periodic acid-Schiff stain and positive
disease.
fungal culture are required for diagnosis. Topical therapy
may be started before diagnosis, but identification of the
infecting organism allows for selection of the most appro- PITYRIASIS VERSICOLOR
priate systemic therapy. PCR, if available, may also be used
Epidemiology
and provides results more quickly than culture.
PV (formerly tinea versicolor) is a superficial fungal infec-
tion of the skin caused by overgrowth of Malassezia species
Treatment of yeast in the stratum corneum. Malassezia globosa is the
In adults, mild-to-moderate disease (£50% nail involve- most frequent causative organism worldwide, with Malas-
ment) with no matrix involvement allows for consideration sezia sympodialis and Malassezia furfur also common. (63)
of the use of topical antifungals in lieu of systemic drugs. PV is not contagious because Malassezia species are part
Until recently, ciclopirox was the only topical antifungal of the normal skin flora. These yeasts are more likely to flour-
available in North America, prescribed off-label (amorolfine ish in hot and humid environments. PV is, therefore, more
is available in the European Union). One small clinical trial common in the summer months or in tropical climates.
has demonstrated the efficacy of ciclopirox in children. (59) PV is more common in adolescents and young adults than
Efinaconazole and tavaborole received US Food and Drug in young children because increased sebum production
Administration approval in 2014. The safety and efficacy of may facilitate fungus growth. (64)
these topical antifungals have not been established in chil-
dren. However, in a child with mild-to-moderate onycho- Clinical Presentation
mycosis, particularly if there is no involvement of the nail PV is characterized by hyper- or hypopigmented round or
matrix, and perhaps even in more severe disease, the topical oval macules with fine scale and a flaking appearance. It is
solutions efinaconazole and tavaborole may be considered often diagnosed based on clinical appearance. Lesions are
(off-label use). Efinaconazole and tavaborole may also be an generally confined to the trunk, neck, and upper arms, all of
attractive option for adolescents who are concerned about which are areas with a high density of sebaceous glands. The
the appearance of their nails as there is evidence that these face, particularly the temples, is also commonly affected
new antifungals can penetrate the nail plate in the presence in children. Lesions are generally asymptomatic, although
of a coat of nail polish. (60)(61) Pediatric studies using these some patients experience mild pruritus. (64) Fine scale
agents are ongoing. There is not enough information on the may be difficult to discern and the “evoked scale sign,” a

18 Pediatrics in Review
stretching or scraping of a lesion, produces a visible fine Treatment
scale that can aid in diagnosis. (65) Hyper- and hypopig- A systematic review concluded that topical treatments
mentation do not necessarily resolve with drug therapy; are effective and well tolerated, with longer durations of
many months may be required for skin to recover a normal treatment more likely to produce favorable outcomes. (69)
appearance. Shampoos and lotions are applied once or twice daily
for 7 to 14 days. One study using ketoconazole shampoo
documented good response with daily application for 3
Differential Diagnosis
consecutive days. (70) Zinc pyrithione, selenium sul-
Differential diagnoses include postinflammatory hyper
fide shampoo/lotion, and ketoconazole shampoo are
(hypo)pigmentation, pityriasis rosea, pityriasis alba, sebor-
applied to affected areas for 5 to 10 minutes and then
rheic dermatitis, vitiligo, and mycosis fungoides. Only PV
washed off in the shower. Selenium sulfide may be more
and seborrheic dermatitis are fungal infections. However,
likely to cause dry skin. Monthly applications of ketoco-
cultures may be negative in both of these disorders, and
nazole, zinc pyrithione, or selenium sulfide shampoo for
culturing requires special agents (eg, olive oil) added to the 3 months may prevent recurrence. Effective topical anti-
media to support yeast growth. Seborrheic dermatitis af- fungal agents include clotrimazole, miconazole, ketocona-
fects the head, neck, chest, and intertriginous folds, and ery- zole, and terbinafine.
thematous scaling plaques are often symmetric and favor Systemic treatment with oral antifungals should only be
sebum-rich areas. Pityriasis alba usually affects the face, considered in cases of recalcitrant or recurring disease or if
with mild erythema preceding hypopigmented lesions and large areas are affected. Itraconazole (off-label) and fluco-
limited to no fine scaling. A history of inflammatory rash and nazole are appropriate, but oral terbinafine is ineffective for
a lack of scale characterize postinflammatory pigmentation. PV. In adults, itraconazole 200 mg daily for 5 to 7 days
Pityriasis rosea usually lasts 1 to 3 months; often occurs in or fluconazole 300 mg weekly for 2 to 4 weeks is admin-
the spring and fall; and presents with a pinkish, erythem- istered. Relapse following topical or oral treatment is com-
atous, scaly herald patch followed by the appearance of mon (60%-80%), and repeated or maintenance therapy
multiple other plaques, often on the trunk. (66) This is may be necessary. (71) Oral ketoconazole should no longer
in contrast to PV, in which lesions have finer scale and are be prescribed for any dermatomycosis in children or adults
hyper- or hypopigmented. An absence of scale in depig- due to risk of hepatotoxicity. (72)
mented macules and accentuation of the depigmentation
under Wood’s light differentiates vitiligo from PV. (67)
MUCOCUTANEOUS CANDIDIASIS
Mycosis fungoides, although rare, is the most common
primary cutaneous lymphoma in children and presents as Epidemiology
numerous scaly patches on the trunk and extremities. These MC encompasses any infection of the body where mucous
lesions are often hypopigmented, particularly in patients membrane meets skin that involves a Candida species, often
of color, and a biopsy is necessary for diagnosis. (68) Diag- Candida albicans. However, candidiasis may also occur in
nosis is often delayed when pityriasis alba, rosea, or PV is folds of skin (intertrigo) such as the groin and corners of the
suspected instead. mouth. Due to pediatrician familiarity with candidal diaper
dermatitis, this condition will not be discussed further.
Relatively few children present with infections in the mouth
Diagnostic Tests (oropharyngeal candidiasis) and throat (esophageal candi-
PV is confirmed via microscopic examination with a positive diasis), known as thrush. Thrush is most common among
KOH. Wood’s light examination may be clinically helpful babies younger than age 1 month and people with compro-
but not necessarily conclusive because not all Malassezia mised immune systems (73) that allow the naturally present
species fluoresce bright yellow or gold. Skin scrapings Candida species to multiply. An estimated 5% to 7% of
should be taken from the edge of lesions, and the trans- babies younger than age 1 month (73) and about 39%
parent tape method may be used if it is difficult to obtain of children with human immunodeficiency virus (HIV)
skin scrapings. Under the microscope, Malassezia species develop an infection. (74) Other risk factors include old
appear as “spaghetti and meatballs,” a combination of short age, endocrine dysfunction, malnutrition, trauma, prema-
hyphae and round budding yeast cells. A clinical diagnosis turity, and use of antibiotics and other medical interven-
should be microscopically confirmed before systemic treat- tions. (2) Chronic mucocutaneous candidiasis (CMC) is
ment, but topical treatment can be instituted empirically. even rarer and may involve a genetic component that is

Vol. 38 No. 1 JANUARY 2017 19


also associated with T-cell immunodeficiency. (75) Patients than 1.2 mg/dL (106 mmol/L) for more than 3 consecutive
with CMC have persistent or recurrent infections that can doses, the dose interval may be decreased to once every 48
involve the nails and require long-term therapy. (37)(76) hours until the serum creatinine measures less than 1.2
mg/dL (106 mmol/L). (31) For fluconazole-resistant infec-
Clinical Presentation tions, itraconazole or posaconazole suspension is recom-
The most common symptoms of thrush are white patches mended. (37)(79) Amphotericin B is recommended when
or plaques on the mucous membranes, redness or soreness other treatment options fail. (37)
in the infected area, difficulty swallowing, and cracking Esophageal candidiasis must always be treated system-
at the corners of the mouth. (73)(77) CMC is often more
ically and oral fluconazole is recommended. (37) For
widespread, occurring in the mucous membranes, skin,
patients who cannot complete oral therapy, intravenous
and nails (onychomycosis). (76)
fluconazole or an echinocandin (eg, micafungin, caspofun-
gin, anidulafungin) are options, with amphotericin B less
Differential Diagnosis
preferred. For fluconazole-resistant infections, itraconazole,
The differential diagnosis of MC depends on the location
voriconazole, amphotericin B, or an echinocandin may be
of infection. If the patient has oral candidiasis, the differ-
ential diagnosis includes leukoplakia, oral hairy leukoplakia, used. (37) In recurrent infections or CMC, suppressive
lichen planus, bullous pemphigoid, pemphigus vulgaris, therapy with fluconazole has been found to be effective.
erythema multiforme, herpes simplex, and aphthous sto- Children who have HIV infection also should be treated
matitis. (78) However, none of these diagnoses is commonly with highly active antiretroviral therapy to reduce recurrent
found in early childhood. (2) For cases of CMC or intertrigo, infections. (31) Proper treatment is important because un-
the differential diagnosis includes erythrasma, seborrheic treated MC may lead to a more severe, invasive infection.
dermatitis, dermatitis enteropathica, syphilis, Gram-negative
folliculitis, familial benign chronic pemphigus (Hailey-
Hailey disease), and bacterial intertrigo. (78)
Summary
Diagnostic Tests • On the basis of strong evidence, tinea capitis is the most common
Because Candida species naturally occur in the body, myco- fungal skin infection in children. (6)(7)
logic tests are of minimal value; the presence of Candida • On the basis of strong evidence, treatment for tinea capitis must
species does not imply that there is an infection. (2) There- be systemic to penetrate the hair shafts. (17)
fore, diagnosis of MC is primarily based on clinical pre- • On the basis of consensus, diagnosis of fungal skin infections
sentation. However, the presence of Candida species may be often can be confirmed through potassium hydroxide
microscopy or a culture, although cultures are of limited use
confirmed through KOH microscopy or a culture. (77) The
for tinea corporis, pityriasis versicolor, and mucocutaneous
presence of many pseudohyphae supports the diagnosis of candidiasis.
a candidal infection.
• On the basis of consensus, topical corticosteroids eventually
worsen tinea corporis infections and should not be used.
Treatment • On the basis of consensus, the feet should be examined for tinea
Therapy for oropharyngeal candidiasis primarily involves pedis and possibly onychomycosis in a child or adolescent
azole antifungals, either topically or systemically; systemic presenting with tinea cruris. (54)
antifungal therapy is required for esophageal candidiasis • On the basis of evidence and consensus, waiting a short period of
and CMC. (37) The following recommendations are based time may allow resolution of symptoms resembling
on the most up-to-date clinical practice guidelines provided onychomycosis (ie, from trauma) because children have thinner
nail plates and faster growing nails.
by the Infectious Diseases Society of America. (37)
Mild cases of oropharyngeal candidiasis can be treated
with clotrimazole troches (small tablet or lozenge), mico-
nazole mucoadhesive buccal tablet, or nystatin suspension.
(37) Moderate-to-severe cases should be treated with oral References for this article are at http://pedsinreview.aappublications.
fluconazole. (37) In neonates, if creatinine levels are greater org/content/38/1/8.

20 Pediatrics in Review
PIR Quiz
There are two ways to access the journal CME quizzes:
1. Individual CME quizzes are available via a handy blue CME link under the article title in the Table of Contents of any issue.
2. To access all CME articles, click “Journal CME” from Gateway’s orange main menu or go directly to: http://www.aappublications.
org/content/journal-cme.

1. A 2-year-old African-American boy presents to your office with a 1-month history of an REQUIREMENTS: Learners
enlarging lesion on his head. On physical examination, you note a 3-cm in diameter lesion can take Pediatrics in
that is boggy and has overlying pustules in the occipital area. There is some shotty Review quizzes and claim
posterior cervical and occipital lymphadenopathy that is not tender. His mother denies the credit online only at:
child having any fevers but notes that he seems uncomfortable when the area on his head http://pedsinreview.org.
is touched. You suspect a kerion. What is your next step in management?
A. Begin treatment with a systemic antifungal medication pending kerion culture To successfully complete
results. 2017 Pediatrics in Review
B. Observe the skin lesion for 1 month. articles for AMA PRA
C. Obtain a skin biopsy of the skin lesion before beginning any treatment. Category 1 CreditTM,
D. Perform a Wood’s light examination of the lesion to see if the fungus fluoresces. learners must
E. Send a skin scraping for polymerase chain reaction. demonstrate a minimum
2. You diagnose tinea capitis in a 14-year-old boy who presented to your clinic with a small performance level of 60%
area of alopecia with associated scaling and erythema. Your diagnosis is confirmed with or higher on this
potassium hydroxide microscopy and culture. You give the parents a prescription for the assessment, which
appropriate systemic antifungal medication. The parents ask about whether this condition measures achievement of
is contagious to other children. Of the following, what would the best response be? the educational purpose
A. A follow-up skin culture must be performed in 2 weeks and if it does not show and/or objectives of this
fungus, the child may then return to school. activity. If you score less
B. All children in the household plus any other children who are close contacts should than 60% on the
begin prophylactic antifungal therapy. assessment, you will be
C. Once treatment is begun, the child may return to school but should not share given additional
combs or helmets or play contact sports for 14 days to avoid transmission. opportunities to answer
D. The child should avoid contact with infants younger than age 1 year during therapy. questions until an overall
E. The child should not be allowed to return to school until treatment is complete. 60% or greater score is
achieved.
3. A father brings his 5-year-old daughter to your office with concerns about an itchy rash
that developed over her face and body a few days after beginning therapy for tinea capitis.
Her vital signs show temperature of 98.4°F (36.9°C), heart rate of 98 beats/min, respiratory This journal-based CME
rate of 26 breaths/min, and blood pressure of 100/55 mm Hg. Over her cheeks and trunk, activity is available
you note a papulovesicular rash that seems to be slightly pruritic. You also note a small area through Dec. 31, 2019,
of tinea capitis on her left temporal area. There are some broken hair shafts visible with however, credit will be
mild scaling and erythema of the skin but no discharge. Of the following, what is the most recorded in the year in
likely explanation for her new symptoms? which the learner
A. A dermatophytid reaction. completes the quiz.
B. A viral exanthem.
C. Drug reaction to the antifungal.
D. Pityriasis rosea.
E. Tinea corporis.
4. You are examining a 15-year-old boy who is a varsity wrestler at his high school. He is
complaining of pruritus and erythema in his groin area for 2 weeks. On physical
examination, his vital signs are all within normal limits. He has large areas of scaling with
scattered overlying papules and areas of maceration on his upper medial thighs bilaterally.
You are concerned about tinea cruris. Of the following, which additional condition is often
associated with tinea cruris?
A. Inverse psoriasis.
B. Nummular eczema.
C. Onychomycosis.
D. Systemic candidiasis.
E. Tinea pedis.

Vol. 38 No. 1 JANUARY 2017 21


5. An 11-year-old girl presents to your office with a chief complaint of white patches inside
her mouth and some difficulty swallowing. Her mother reports that the child just finished 5
weeks of antibiotic therapy for osteomyelitis of her right tibia. Her vital signs are within
normal ranges. Mouth examination shows several patches of white plaques on her inner
cheeks with underlying erythema. There are similar lesions in her posterior oropharynx
extending inferiorly toward her esophagus. What is your best initial choice for treatment at
this time?
A. Amphotericin B.
B. Clotrimazole troches.
C. Fluconazole.
D. Griseofulvin.
E. Terbinafine.

Parent Resources from the AAP at HealthyChildren.org


• Tips for Treating Viruses, Fungi, and Parasites: https://www.healthychildren.org/English/health-issues/conditions/treatments/Pages/Tips-
For-Treating-Viruses-Fungi-and-Parasites.aspx
• Thrush and Other Candida Infections: https://www.healthychildren.org/English/health-issues/conditions/infections/Pages/Thrush-and-
Other-Candida-Infections.aspx
• Fungal Diseases: https://www.healthychildren.org/English/health-issues/conditions/infections/Pages/Fungal-Diseases.aspx
• Tinea Infections (Ringworm, Athlete’s Foot, Jock Itch): https://www.healthychildren.org/English/health-issues/conditions/skin/Pages/
Tinea-Infections-Ringworm-Athletes-Foot-Jock-Itch.aspx

22 Pediatrics in Review
Tube Feeding in Children
Sarita Singhal, MD,*† Susan S. Baker, MD, PhD,*† Georgina A. Bojczuk, RD, CSP,† Robert D. Baker, MD, PhD*†
*Department of Pediatrics, Jacobs School of Medicine and Biomedical Sciences, University at Buffalo, State University of New York, Buffalo, NY

Digestive Diseases and Nutrition Center, Women and Children’s Hospital of Buffalo, Kaleida Health, Buffalo, NY

Education Gap
Enteral tube feeding (TF) is an important part of the care of acutely ill
children as well as an essential technique to deliver nutrition to
children who have chronic conditions. However, the techniques of TF
are neither a part of medical school courses nor specifically taught in
pediatric residency programs. Thus, learning how to initiate,
monitor, moderate, and transition TF is often learned by trial and
error.

Objectives After completing this article, readers should be able to:

1. Understand the need for nutritional assessment and nutrition support.


2. Recognize indications and contraindications for tube feeding (TF).
3. Plan the evaluation of a patient who requires TF.
4. Recognize the factors that are important to make the proper selection
of enteral access for TF.
5. Plan the initiation and administration of TF.
6. Recognize the potential complications of TF and learn troubleshooting
methods.
7. Coordinate care for home nutrition support and assist in the transition
to oral feedings.
8. Understand the social needs of patients who have TF.

INTRODUCTION

Tube feeding (TF) is a mode of providing enteral nutrition when oral feeding is
not possible or not sufficient. TF is delivered through a medical device that can
AUTHOR DISCLOSURE Drs Singhal, S. Baker, be placed into the stomach, duodenum, or jejunum via either the nose, mouth, or
and R. Baker and Ms Bojczuk have disclosed
the percutaneous route. This review focuses on TF in children beyond the neonatal
no financial relationships relevant to this
article. This commentary does not contain a period.
discussion of an unapproved/investigative use Nutritional support can be either enteral or parenteral. Enteral nutrition (EN)
of a commercial product/device. Dr Singhal’s
refers to any method of feeding that uses the gastrointestinal (GI) tract to deliver
present affiliation is: Department of Pediatrics,
Division of Pediatric Gastroenterology, Baystate part or all of a child’s nutritional requirements. It can include a normal oral diet or
Children’s Hospital, Spingfield, MA. feeding via tube. Parenteral nutrition (PN) refers to the delivery of nutrients by

Vol. 38 No. 1 JANUARY 2017 23


vein. Studies have shown that EN is preferred over PN be-
cause it leads to earlier gut function, fewer infections, lo- TABLE 1. Indications for Tube Feeding
wer cost, and shorter hospital stay. (1)(2)(3) In addition, EN
1. Insufficient oral intake
delivers nutrients directly to the GI tract, avoiding delete-
- Anorexia
rious changes in the normal gut physiology and the aberrant
physiology that ensues when nutrients are infused directly -Food aversion
into the systemic circulation without the benefit of the -Malabsorption (cystic fibrosis, short bowel syndrome, pancreatic
modifying functions of the GI tract and liver. (4) PN is insufficiency)

reserved for patients who have GI tract dysfunction that -Increased needs (congenital heart disease, bronchopulmonary
dysplasia)
prohibits adequate nutrient absorption.
Nutrition is important for normal growth and develop- 2. As a primary therapy
ment in children. Nutritional assessment, therefore, should -Metabolic disease
be an integral part of the care for every pediatric patient. -Intolerance to fasting
Complete nutritional assessment includes a medical his-
-Inflammatory bowel disease
tory, nutritional history that includes dietary intake, physical
examination, anthropometrics, pubertal staging, skeletal ma- 3. Oral motor dysfunction
turity staging, and biochemical tests of nutritional status. -Prematurity
(5)(6)(7)(8) Most healthy children have the ability to ingest -Neuromuscular disease
enough nutrients to meet their needs, but children with
-Neurologic disease
chronic medical conditions or prolonged illnesses may not
4. Abnormal gastrointestinal tract
be able to meet their nutritional goals. Approximately 10%
to 15% of children in the United States have special health -Congenital malformations
care needs. Most of these children are at risk for poor growth. -Esophageal stenosis
It is important for clinicians to identify these children, as- -Intestinal pseudo-obstruction
sess their nutritional needs, and provide adequate nutri-
5. Injury/critical illness
tional support for their growth and development.
-Burn
-Trauma
INDICATIONS FOR TF
-Surgery
TF is used for children who have inadequate or unsafe oral
-Sepsis
intake and a functioning GI tract (Table 1). If the GI tract is
able to absorb some but not all of the nutritional needs, Adapted with permission from: Baker SS, Baker RD, Davis AM. Pediatric
partial EN should be attempted. If the patient has normal Nutrition Support. 2007; Jones & Bartlett Learning. Burlington, MA. www.
jblearning.com. Copyright 2007.
swallowing function, oral supplements may be added to
achieve the complete nutrition intake and to maintain oral skills.
dependent on various factors, including the patient’s GI
tract anatomy and function, indication for feedings, ex-
CONTRAINDICATIONS FOR TF
pected duration of feedings (short term of weeks to months
When oral feeding is not possible or is inadequate, TF is the or long term of months to years), and the risk of aspiration.
method of choice because it is more physiologic than PN. An upper GI radiographic series should be performed to
The only absolute contraindication for TF is a nonfunction- rule out anatomic barriers to TF such as malrotation or
ing GI tract, such as GI obstruction or severe intestinal gastric outlet obstruction. A careful history can identify
ischemia. Conditions such as intestinal fistulae and severe signs and symptoms suggestive of aspiration. In some
pancreatitis are no longer considered contraindications. cases, a swallow study under fluoroscopy may be helpful to
evaluate swallowing function and the risk of aspiration. A
number of options for enteral access are available (Table 2).
CHOICE OF ENTERAL ACCESS FOR TF
Orogastric (OG) feeding is used most frequently in pre-
Once it is decided that TF is necessary, the clinician must term infants before they develop a gag reflex (34 weeks’
determine the most appropriate enteral access based on the gestation). Preterm infants are obligate nose breathers, so
individual clinical situation. The choice of enteral access is OG feeding avoids obstruction of the nares. OG feedings are

24 Pediatrics in Review
TABLE 2. Choice of Tube
DURATION OF
TYPE OF TUBE FEEDING PLACEMENT COMMENT USE

Orogastric Short-term Bedside • Used in preterm infants up • Feeding


to 34 weeks’ gestation • Medication
• Safe with basilar skull fracture • Hydration

Nasogastric Short-term Bedside • With or without stylet • Feeding


• Medication
• Hydration

Nasointestinal (including any nasal Short-term Fluoroscopic • Displaces easily • Feeding


tube that extends beyond the • Weighted or unweighted • Hydration
pylorus)
Gastrostomy Long-term Surgical • Not ready for immediate use • Feeding
• Medication
• Hydration

Percutaneous endoscopic Long-term Endoscopic or radiologic • Can be used 4 hours from • Feeding
gastrostomy (PEG) techniques placement • Medication
• Hydration

Low-profile device Long-term Initial endoscopic • More convenient and easier • Feeding
to care for than gastrostomy • Medication
tube • Hydration
Internal balloon or internal Thereafter at • Aesthetically more pleasing
“mushroom” bedside (home)
Gastrointestinal (including any gastric Long-term Endoscopic, • Possible to access both stomach • Limits reflux and
tube that extends beyond the radiologic, or both and small bowel aspiration
pylorus) gastrojejunostomy or • Easily dislodged • Feeding
percutaneous endoscopic • Hydration
jejunostomy
Jejunostomy Long-term Surgical or endoscopic • No access to stomach • Feeding
• Hydration
• Limits reflux and
aspiration
Low-profile jejunal device Long-term Endoscopic • More convenient • Feeding
Internal mushroom bolster or • Aesthetically more pleasing • Hydration
fluid-filled balloon • No access to stomach • Some medications

preferred over nasogastric (NG) feedings in the presence of of aspiration of swallowed secretions, an issue with many
a basilar skull fracture. OG feedings are also used when the neurologically impaired children. An NG tube can be placed
nares are obstructed (eg, cystic fibrosis). at the bedside, but proper position should be verified ra-
NG feeding is the method of choice in children with diologically or by aspiration of acidic stomach contents. Aus-
normal gastric function and a low risk of aspiration who cultation over the stomach is not recommended. (9)
require short-term nutrition support. It is the simplest, least Gastrostomy tubes (G-tubes) are preferred for intragas-
expensive method and requires no invasive procedures. tric feeding that is expected to last longer than 3 months.
Frequently, it is used for patients being evaluated for more G-tubes can be placed surgically, endoscopically, or radiolog-
permanent tubes to ensure tolerance for intragastric feed- ically. Surgically placed tubes have the advantage of creating
ings. NG tubes have been used for long-term nutrition a formal attachment between the abdominal wall and the
support in patients who have learned to place the tube each stomach wall, thus reducing the possibility that the tube
night, infuse feeding slowly overnight, and remove the tube becomes dislodged, potentially contaminating the perito-
in the morning. Even for patients at some risk for aspiration, neal cavity. For the other 2 methods of placement, 2 months
NG feeding is not contraindicated. Infusing feedings over a are required for an attachment to form (tract maturation).
long period of time can result in less emesis by limiting the A disadvantage of a surgically placed G-tube is that it cannot
gastric contents at any given time. It is important to recog- be used immediately. Some surgeons do not use a surgically
nize that TF by any method does not decrease the likelihood placed tube for several days, in contrast to endoscopically or

Vol. 38 No. 1 JANUARY 2017 25


radiologically placed tubes, which can be used within a few pass into the duodenum and jejunum by peristalsis. Neither
hours of insertion. erythromycin, metoclopramide, nor weighting has been asso-
If a simultaneous antireflux procedure is planned, sur- ciated with any benefit in promoting passage of the tube by
gical placement makes sense. However, there is no indica- peristalsis. This method is time-consuming and not always
tion for a prophylactic fundoplication in a patient without a successful. Nasointestinal tubes can be placed endoscopically
history of reflux. Even in the presence of prior reflux, a using a number of techniques. The endoscopic techniques
G-tube does not necessarily worsen reflux and, at times, may are invasive, require anesthesia, and are not always success-
improve it. ful. The radiologic placement of nasointestinal tubes has
G-tube placement carries a risk of infection (intraabdo- proven to be the fastest and most reliable approach. Nasoin-
minal abscess or peritonitis) or, in endoscopically or radio- testinal tubes are easily dislodged and may require frequent
logically placed tubes, colonic perforation. Not infrequently, replacement.
air is introduced into the peritoneal cavity during the A previously placed G-tube can be converted into a GJ
course of either radiologic or endoscopic G-tube placement. tube by replacing the G-tube with a device that looks similar
This is not a substantial complication because the air re- to the G-tube but has an extension that traverses the pylo-
sorbs without treatment. However, it can be alarming if rus and extends into the small intestine. The extension is
an abdominal radiograph is obtained because the air in the usually placed into the jejunum by a radiologist, although
cavity mimics the radiologic findings of bowel perforation. there are methods to carry the extension into the jejunum
All G-tubes are prone to local irritation, with formation of with an endoscope. Usually the tract must be mature (ie, the
granulation tissue, local skin irritation, and skin infection. G-tube must have been in place for 2 months) to make this
After the tract has matured, the G-tube can be replaced with conversion. One commercial kit allows for a small tube to be
a “low-profile” device, which is aesthetically more pleasing placed through a newly inserted G-tube and then past the
and easier to maintain. Low-profile devices cause less irri- pylorus into the small intestine. The smaller tube has an
tation. A kit is available that allows initial placement of a adapter that fits the tube snugly into the PEG tube, thereby
low-profile device. The standard percutaneous endoscopic eliminating leakage.
gastrostomy (PEG) kit contains a PEG tube with an external Jejunostomy feeding tubes can be placed directly through
bolster, safety trocar cannula, safety-shielded scalpel, retrieval the skin and into the jejunum either surgically or endo-
snare, Y-port, and C-clamp. Low-profile devices have either a scopically. These two techniques are not used frequently
mushroom-shaped bolster or a fluid-filled balloon that keeps because both methods eliminate access to the stomach,
the tube in the lumen of the stomach. The advantage of the which is often necessary for venting.
mushroom is that it is difficult to dislodge, and the advantage
of the fluid-filled balloon is that the fluid can be removed and
PREPYLORIC VERSUS POSTPYLORIC FEEDINGS
the tube replaced. Accidental loss of fluid from the balloon
can result in the tube coming out unintentionally. Prepyloric feeding is more physiologic than postpyloric
G-tubes should be changed periodically. Some centers feeding because feeding into the stomach allows a more
recommend changing G-tubes every 3 months. Other cen- normal digestive process. Most patients tolerate prepyloric
ters do not advocate a routine, but change the tubes when feeding if the stomach is functionally and structurally
they appear worn or become nonfunctional. Parents can be normal. The stomach acts as a reservoir and can tolerate
taught to change the low-profile devices that have an internal larger volumes and higher osmotic loads than the small
balloon in the stomach. Because G-tubes do not prohibit intestine. Prepyloric feedings allow for the use of bolus
simultaneous oral feeding, they are excellent for transition- feeding, creating a more flexible feeding schedule that is
ing from TF to oral feeding. When TF is no longer needed, closer to a natural pattern of eating. Bolus feedings may lead
the G-tube can simply be removed. Most often the stoma to better gastric contractility because they are cyclical and
seals and heals. Occasionally, it remains open, requiring associated with peaks and troughs of insulin secretion.
either endoscopic closure using internal clips or formal Prepyloric feedings also provide a protective effect against
surgical closure. hyperosmolar formulas because duodenal osmoreceptors
Nasointestinal feeding tubes can be used for the short- regulate gastric emptying and retain the formula in the
term delivery of feedings beyond the stomach. These tubes stomach until ingested formula is isosmotic. This decreases
can be placed by a number of methods. A weighted or the risk of dumping. Dumping syndrome is the result of
unweighted tube can be passed into the stomach with extra high-volume or high-osmolar fluid entering the duodenum.
length of the tube allowed to remain in the stomach and to This induces inappropriate gut hormone release and vasomotor

26 Pediatrics in Review
GI symptoms such as abdominal pain, bloating, and diarrhea. Bolus feedings are given to patients who receive intra-
One of the major concerns with the prepyloric feeding is the gastric feedings because the stomach can tolerate large vol-
potential risk of pneumonia from aspiration of formula from umes. Once the feeding volume required for a 24-hour period
the stomach. (10)(11) Therefore, clinicians should be cautious is determined, it is divided into 4 to 6 bolus feedings that can
in administering prepyloric feedings to patients with severe be administered by syringe, gravity, or with a pump. Bolus
gastroparesis and a history of aspiration. feedings are more physiologic and allow patient mobility.
Postpyloric feeding is defined as feeding beyond the They are the preferred method of delivery for ambulatory
pylorus, either into the duodenum or into the jejunum patients. They also are less expensive and easier to administer
distal to the ligament of Treitz. Postpyloric feeding can than continuous feedings. However, bolus feedings may not
be delivered via nasoduodenal tube, gastrojejunal tube, or be successfully used in patients with reflux or gastroparesis or
surgical jejunostomy. This is useful for patients who are those who require large feeding volumes. Patients might
at risk for aspiration pneumonia or who have intolerance to experience nausea, vomiting, diarrhea, or abdominal disten-
gastric feeding, recurrent emesis, or severe gastroesopha- sion with bolus feeding. (8)(12)
geal reflux. (11)(12) Postpyloric feedings bypass the stomach, Continuous feedings are delivered via an infusion pump.
thus decreasing but not eliminating the risk of reflux and They can be administered through jejunal, gastrostomy,
aspiration. Especially in neurologically impaired children or nasointestinal tubes. Continuous feedings are admin-
who are unable to protect their airways, postpyloric feeding istered at a constant hourly rate over an 8- to 24-hour period,
can be helpful. Patients with gastroparesis or with emesis as depending on the patient’s nutritional requirements. Contin-
a result of chemotherapy also can benefit from postpyloric uous feedings are beneficial for critically ill patients, mal-
feedings. Most postpyloric tubes have a “gastric port” that nourished patients, and those who have malabsorption due
allows venting of the stomach and administration of med- to intestinal diseases. They are also used for patients who
ications that need to be given into the stomach. have intolerance to gastric bolus feedings. Nocturnal feed-
Postpyloric feedings bypass the digestive and antibacte- ings are useful for providing extra energy to ambulatory
rial functions of the stomach. Because the small intestine patients. Continuous feedings are more expensive due to
cannot tolerate hyperosmolar feedings, postpyloric feed- equipment requirements and supplies and they restrict
ings must be administered slowly over most of the day, ambulation. (8)(12)
thus precluding bolus feedings. The major disadvantages of
postpyloric feeding are the difficulty in placement of tubes
INITIATION OF TF
and the risk of tube migration back into the stomach. The
tubes are long and have a small caliber, which creates the After a thorough evaluation by a multidisciplinary team that
chance of occlusion. Placement into the proximal jejunum consists of variable combinations of a gastroenterologist,
is preferable because it decreases the likelihood of feedings nurse, dietitian, speech-language pathologist, occupational
refluxing into the stomach and esophagus, with aspiration therapist, psychologist, surgeon, and radiologist, the deci-
into the airway. Also, tubes ending in the jejunum are less sion to initiate TF is taken.
likely to become displaced, a problem with all postpyloric Initiating and administering a TF regimen requires care-
tubes. ful assessment of the nutritional status of each patient to
ensure that the best regimen is prescribed, monitored, and
maintained. Assessment takes into account each patient’s
BOLUS VERSUS CONTINUOUS FEEDING
age, medical status, and GI tract function as well as type of
TF can be administered continuously or intermittently (bo- feeding tube, feeding goals, and desired feeding schedule. If
luses). The 2 methods are often combined, such as contin- results of this assessment suggest that TF is indicated, the
uous feeding during the night and bolus feeding during the first step is to select an appropriate formula (Tables 3 and 4).
day. This combination is useful for ambulatory patients who Once a formula is selected, TF may be administered by
require large volumes to meet their nutritional requirements. bolus, continuous, or a combination of these methods. For
The method of delivering nutrition is dependent on various most patients there is no need to dilute formula; full-
factors, such as the type of tube (prepyloric or postpyloric), strength formula is started and volume gradually increased
medical condition of the patient (age, ambulatory status, as tolerance is demonstrated.
underlying diseases), expected tolerance to feedings, nutri- Initiation and advancement of TFs can vary greatly
tional requirements, and other factors (availability of parental among clinicians and individual medical facilities, but most
support, acceptance, availability of equipment, cost). medically stable patients can tolerate fairly rapid progression

Vol. 38 No. 1 JANUARY 2017 27


TABLE 3. Choice of Formula for Infants
MACRONUTRIENT SOURCE
FORMULA EXAMPLES/PRODUCT (VARIES BY INDIVIDUAL
CATEGORY INDICATIONS MANUFACTURER PRODUCT) COMMENTS

Human Milk Preferred nutrition source for PRO: whey-predominant, • Nonhomogenized


virtually all infants* when whey:casein ratio varies • Special care and technique
mother’s milk and/or FAT: human milk needed when feeding via
donor milk is available CHO: lactose tube
• 20 kcal/oz, crematocrit can
be performed to measure
kcal concentration
Postdischarge Posthospital discharge Enfacare Lipil,a Similac PRO: whey, casein, cow milk • Standard concentration is
Formulas for formulas for former NeoSureb protein 22 kcal/oz
Prematurity preterm infants FAT: high-oleic, soy, coconut • High in protein, vitamin D,
oils, MCT, DHA, ARA calcium, and phosphorus
CHO: corn syrup solids, • 250-310 mOsm/kg water
lactose, maltodextrin
Standard Infant Normal gastrointestinal tract Enfamil Premium,a PRO: cow milk protein, whey • Standard concentration is
Formulas Enfamil Gentlease,a protein concentrate 20 kcal/oz
Gerber Good Start (Gentlease and Good Start • Similac concentration is
Gentle,c Similac contain partially hydro- 19 kcal/oz
Advanceb lyzed protein) • 230-310 mOsm/kg water
FAT: palm olein, soy, coco-
nut, high-oleic sunflower,
DHA, ARA
CHO: lactose,
galactooligosaccharides,
polydextrose, corn syrup
solids
Soy-based Infant Galactosemia, primary or Enfamil Prosobee,a Gerber PRO: soy protein isolate and • Lactose-free
Formulas secondary lactose Good Start Soy,c Similac L-methionine • 20 kcal/oz standard
intolerance, Soy Isomil FAT: palm olein, soy, coco- concentration
families preferring vegan nut, high-oleic safflower/ • Similac concentration is
formula option sunflower, DHA, ARA 19 kcal/oz
CHO: corn syrup solids, • 180-200 mOsm/kg water
sucrose
Extensively Food protein intolerance, Similac Alimentum,b PRO: cow milk protein • Hypoallergenic
Hydrolyzed Infant malabsorption, Nutramigen Lipil,a hydrolysate • Lactose-free
Formulas steatorrhea, intractable Pregestimila FAT: long-chain fat, variable • 20 kcal/oz standard
diarrhea MCT (0%-55% of fat), DHA, concentration
ARA • 320-370 mOsm/kg water
CHO: corn syrup solids,
modified corn starch,
dextrose, sucrose
Elemental/Free Severe protein allergy, Alfamino Infant,d Elecare PRO: free amino acids • Hypoallergenic
Amino Acid Infant eosinophilic Infant,b Neocate Infant,e FAT: high-oleic safflower oil, • No cow milk protein, soy,
Formulas gastrointestinal disorders, PurAminoTMa soy, coconut, sunflower fructose, galactose, or
malabsorptive conditions, oil, variable MCT (33%-43% lactose
short bowel syndrome of fat), DHA, ARA • 330-350 mOsm/kg water
CHO: corn syrup solids

*Contraindicated in some instances, such as galactosemia, maternal human immunodeficiency virus/AIDS, active tuberculosis, and some inborn errors of
metabolism.
a
Mead Johnson Nutrition, Chicago, IL.
b
Abbott Laboratories, Abbott Park, IL.
c
Nestlé Infant Nutrition, Florham Park, NJ.
d
Nestlé Health Science, Florham Park, NJ.
e
Nutricia North America, Gaithersburg, MD.
ARA¼arachidonic acid, CHO¼carbohydrate, DHA¼docosahexaenoic acid, FAT¼fat, MCT¼medium-chain triglyceride, PRO¼protein.
Formula content and specifications as per product manufacturers’ websites.

28 Pediatrics in Review
TABLE 4. Choice of Formula for Children Older Than Age 1 Year
MACRONUTRIENT SOURCE
FORMULA EXAMPLES/PRODUCT (VARIES BY INDIVIDUAL
CATEGORY INDICATIONS MANUFACTURER PRODUCT) COMMENTS

Standard Pediatric Normal GI tract requiring a Boost Kid Essentials TM,a
PRO: cow milk protein con- • 1 kcal/mL, 30 cal/oz
Enteral Formulas complete or supplemental Compleat Pediatric,a centrate (Compleat con- • Lactose-free
source of energy from tube Nutren Junior,a tains food ingredients: • Meets/exceeds 100% of the
feeding PediaSure Enteral 1.0 Cal, chicken, peas, carrots, DRIs for protein, vitamins/
PediaSureb tomatoes, and cranberry minerals for children 1-8
Adolescents >13 years: juice) years, 9-13 years in 1,000 mL
Jevity 1 Cal,b Nutren FAT: high-oleic sunflower oil, and 1,500 mL, respectively
1.0,a Osmolite 1.0b • For oral or tube feeding use
soybean, safflower, canola
oil, variable MCT (0%-20% of • Fiber content varies
fat) • 300-550 mOsm/kg water
CHO: maltodextrin, sucrose,
corn syrup
Calorie-dense Normal GI tract requiring Children 1-13 years: PRO: cow milk and whey • 1.2-2.0 kcal/mL
Pediatric increased energy needs, Boost Kid EssentialsTM protein concentrate, • Lactose-free
a  b
Formulas shortened feeding 1.5, PediaSure 1.5 sodium and calcium • Nutritionally complete in
schedules, fluid restriction, Adolescents >13 years: caseinate, soy protein varying volumes/patient
 b
or have volume intolerance Isosource 1.5, Jevity 1.2, isolate age-dependent
 b 
Jevity 1.5 Cal, Nutren FAT: high-oleic sunflower/saf- • 370-780 mOsm/kg water
1.5,a Nutren 2.0,a Osmolite flower, soy oil, canola oil, • 69%-81% free water; while
1.2,b Osmolite 1.5b
variable MCT (0%-20% of fat) using these, be sure ade-
CHO: sucrose, corn syrup quate free water flushes are
solids, maltodextrin provided to meet hydration
needs of patient
Reduced-calorie Age 1-13 years with decreased Compleat Pediatric Reduced PRO: cow milk and whey • 0.6-0.63 kcal/mL
Pediatric Enteral energy needs requiring a Calorie,a PediaSure protein concentrate, soy • Lactose-free
Formulas lower-energy complete SideKicks 0.63 kal/mLa protein isolate, sodium • Beneficial to address
feeding caseinate, chicken, pea disproportionate weight
protein isolate gain often associated with
FAT: canola oil, soy oil, vari- developmental disabilities
able MCT (0%-20% of fat) • Fiber content varies
CHO: corn syrup, sucrose • 300-420 mOsm/kg water
Hydrolyzed Impaired GI tract function PediaSure Peptide 1.0,b PRO: enzymatically hydro- • 1.0-1.5 kcal/mL
Pediatric requiring peptide-based PediaSure Peptide 1.5,b lyzed whey protein, • Flavored and unflavored
Formulas complete nutrition formula; Peptamen Junior 1.0,a hydrolyzed sodium • Lactose-free
may be beneficial for use in Peptamen Junior 1.5a caseinate • Fiber content varies
malabsorption, short bowel FAT: canola oil, soy oil, vari- • 260-450 mOsm/kg water
syndrome, chronic diarrhea, able MCT (50%-60% of fat)
delayed gastric emptying, or CHO: maltodextrin, sucrose,
for previous intolerance
cornstarch
issues with intact protein
formulas

Free Amino Acid For children with impaired GI Alfamino Junior,c Elecare PRO: free amino acids • 0.8-1.0 kcal/mL
Pediatric tract function requiring a Junior,b Neocate Junior,d FAT: high-oleic safflower oil, • Flavored and unflavored
Formulas hypoallergenic, amino acid- Neocate Splash,d Vivonex soy oil, variable MCT (33%- • Lactose-free
a
based formula; may be Pediatric 70% of fat) • Available in powder or ready
beneficial for use in patients CHO: corn syrup solids, to feed (Neocate Splash
with multiple food allergies, potato starch, modified manufactured as ready
eosinophilic GI disorders, corn starch to feed)
malabsorptive conditions, • 360-590 mOsm/kg water
short bowel syndrome, and
other GI tract impairments

a
Nestlé Health Science, Florham Park, NJ.
b
Abbott Laboratories, Abbott Park, IL.
c
Nestlé Health Science, Florham Park, NJ.
d
Nutricia North America, Gaithersburg, MD.
CHO¼carbohydrate, DRI¼dietary reference intake, GI¼gastrointestinal, MCT¼medium-chain triglyceride, PRO¼protein.
Formula content and specifications as per product manufacturers’ website.

Vol. 38 No. 1 JANUARY 2017 29


to reach their established goal within 1 to 2 days of initi- interfere with care at the worst. Table 6 lists some of these
ation (Table 5). Medically fragile patients (particularly those complications, possible causes, and how to troubleshoot
at risk for refeeding syndrome) or patients with compro- them. Table 6 does not discuss gastric residuals because
mised GI tracts often require slower progression of feed- gastric residual volumes do not correlate with risk of aspi-
ings. Patients should be monitored closely for the initial ration in critically ill patients and cessation of feedings based
3 to 5 days to ensure adequate nutrient delivery and for on gastric residual volumes underfeeds ill patients and does
potential complications. The family should be involved not prevent aspiration.
in the process of designing and implementing a TF reg-
imen, especially if the patient is to be discharged on home
HOME NUTRITION SUPPORT
TF. Ultimately, the family is responsible for administering
the daily feeding, and it is imperative that the schedule Once the TF is established and the patient shows a clinical
work well for all involved. response, discharge planning should begin. The important
components of discharge planning are patient/caregiver
training and arranging for supplies and necessary equip-
COMPLICATIONS
ment, home nursing care, and follow-up evaluations with
Fortunately, most complications associated with TF are clinicians. Home care involves a team consisting of physician,
more of a nuisance than they are causative of morbidity. nurse, dietitian, case manager, discharge planner, vendor for
However, serious complications can occur and can be a supplies, and home nursing services.
manifestation of the underlying disease or the enteral The patient/caregiver should be educated about the
feeding itself. For example, vomiting or retching is often disease process and the need for TF, handling of equipment
seen in children with developmental delays or due to an (using feeding pump), managing feedings (feeding reg-
intervention such as a fundoplication. Complications can be imen, preparation and administration of formula), and
attributed to the mechanics of the TF, infections, metabolic methods for troubleshooting common problems and mon-
issues, intolerance, noncompliance, lack of patient or parent itoring for complications. It is also crucial to explain to the
satisfaction, and failure to reach nutritional goals. family when to call the health care team. The family should
Complications seen immediately after G-tube insertion be provided with both routine and emergency telephone
include misplacement, bleeding, and infection and should numbers. Arranging for supplies and services at home is
be referred to the service that placed the tube: gastroenter- handled either by the home care team of the hospital or a
ology, interventional radiology, or surgery. The proceduralist commercial home care company. The home care company
who placed the tube usually addresses these complications. provides numerous services, including the equipment, home
Later problems can occur that are vexing at the least and nursing, and delivery of nutrient supplies.

TABLE 5. Initiation and Advancement of Tube Feeding


TYPE PATIENT AGE INITIAL INFUSION RATE ADVANCEMENT GOAL

Continuous 0-12 months 1-2 mL/kg/h 1-2 mL/kg every 8 hours 5-6 mL/kg/h
1-3 years 1 mL/kg/h 1 mL/kg every 8 hours 4-5 mL/kg/h
4-10 years 20-30 mL/h 20-30 mL every 8 hours 3-4 mL/kg/h
11-18 years 30-60 mL/h 30 mL/h every 8 hours 100-150 mL/h
Bolus 0-12 months 30-60 mL every 2-3 hours 15-60 mL/feeding 150 mL every 4-5 hours
1-3 years 30-90 mL every 2-3 hours 60 mL/feeding 180 mL every 4-5 hours
4-10 years 75-90 mL every 3 hours 60 mL/feeding 210 mL every 4-5 hours
11-18 years 90-120 mL every 3 hours 60 mL/feeding 240 mL every 4-5 hours
Cyclic 0-12 months 1-2 mL/kg/h 1-2 mL/kg/2 h 75 mL/h x 12-18 h per day
1-3 years 1 mL/kg/h 1 mL/kg/2 h 90 mL/h x 8-16 h per day
4-10 years 25 mL/h 25 mL every 2 hours 120 mL/h x 8-16 h per day
11-18 years 30 mL/h 30 mL every 2 hours 150 mL/h x 12 h per day

Adapted with permission from: Baker SS, Baker RD, Davis AM. Pediatric Nutrition Support. 2007; Jones & Bartlett Learning. Burlington, MA. www.jblearning.
com. Copyright 2007.

30 Pediatrics in Review
TABLE 6. Complications of Tube Feeding and Troubleshooting
COMPLICATION POSSIBLE CAUSE TROUBLESHOOTING

Mechanical
Tube breaks Manufacturer problem, mishandling, Replace tube or part
worn out
Cannot rotate tube, dimples skin, sometimes Tube too tight Resize and change to appropriate size; if
skin is irritated balloon present, check volume and be sure
not overfilled
Buried bumper, cannot rotate tube but feeds Tube too tight and becomes imbedded into the Contact team that placed tube to assess and
flow freely into the stomach gastric wall; caused by too small a tube or replace tube
mechanical traction on the tube
Clogged tube Failure to rinse tube after feedings, delivery of Replace tube; attempts to force fluids
“crushed” medications through tube through a clogged tube often result in
tube rupture
To prevent future clogs, change
medications to liquid form if available,
educate on proper flushing protocols
Dislodged Handling by patient Reposition
Tube hangs out onto abdominal wall Tube too long or internal balloon issue Resize and replace with appropriate-sized
tube; if balloon present, be sure it is
intact and correct volume of water
is in place
Leaking at stoma site Assess tube size; assess for infection and Place correct-sized tube, treat infection, check
underfilled balloon fluid volume in balloon
Granulation tissue Caused by repeated mechanical trauma Cauterize with silver nitrite, assess tube for size,
educate about handling of tube
Metabolic
Dehydration Too little free water, hyperosmolar or Increase free water, reassess formula
high-protein formula
Hyperglycemia Diabetic with changed insulin requirement Monitor blood glucose, reduce carbohydrate
content, adjust insulin dose
Hyperkalemia High-potassium formula, renal insufficiency, Change formula; give potassium binder,
intravenous potassium, acidosis insulin, glucose; stop or decrease
intravenous potassium; correct acidosis
Hyperphosphatemia Renal insufficiency Change formula to a renal-specific
formulation; give phosphate binder,
calcium supplements
Hypokalemia Malnutrition, diarrhea, insulin administration Monitor electrolytes, fluid and electrolyte
replacement, assess insulin dose
Hypophosphatemia Refeeding syndrome, insulin administration Phosphorus supplements; hold feedings if
phosphorus is £1.0 mg/dL (£0.32 mmol/L)
until correction begins, assess insulin dose
Hyponatremia Overhydration Adjust fluids
Acute rapid weight gain Fluid overload Adjust fluids
Rapid excessive weight gain Too many calories Reassess prescription for enteral feeding:
formula concentration, rate, length of
feeding
Inadequate weight gain Not enough calories Reassess prescription for enteral feeding:
formula concentration, rate, length of
feeding
Continued

Vol. 38 No. 1 JANUARY 2017 31


TABLE 6. (Continued )

COMPLICATION POSSIBLE CAUSE TROUBLESHOOTING

Gastrointestinal
Diarrhea, general Osmotic load, infection, contamination of Assess formula composition; assess method of
feeding administration, storage, handling; review all
medications, especially liquid ones, for
osmotic load (sorbitol content); reduce rate
of nutrient delivery; assess for infection
Diarrhea, gastrointestinal tract Flat villus lesion, short bowel, pancreatic Define gastrointestinal problem and treat
abnormalities insufficiency accordingly: pancreatic supplements,
bulking agents; slow rate of administration
Malabsorption Define cause Treat identified cause; consider increased use
of medium-chain triglycerides, elemental or
semi-elemental formula
Vomiting Gastrointestinal obstruction, tube Assess and treat accordingly; reposition or
malpositioned (if using a Foley catheter, change type of tube; do not administer
the balloon occludes the pylorus), medications with feedings; reposition
medications administered with feeding, patient; slow feeding rate or change to
patient positioning, delayed gastric continuous feedings; consider
emptying, after a fundoplication erythromycin trial; if after fundoplication,
trial of nasojejunal feedings
Constipation Inadequate fluids, inadequate fiber, inactivity, Assess fluids, consider formula containing
obstruction, fecal impaction fiber, disimpact as necessary, consider
osmotic stool softener
Skin is erythematous, warm, or turning Culture Treat with broad-spectrum antibiotic, re-
tube is painful; pus is visible educate on proper tube care and hygiene of
tube site
Abdominal distention Gastrointestinal obstruction, constipation, Treat obstruction; treat constipation; if not
aerophagia, bacterial overgrowth obstructed or constipated, vent gastric port
or consider a Farrell Valve Bag*

*Farrell Valve Bag System (Corpak Medsystems, Buffalo Grove, IL) is a device that allows continuous venting of tube feedings and provides reservoir for
formula.

It is important to realize and relay to families that TF is new food. Posttraumatic feeding disorder is caused by a
a dynamic process that can and should be adjusted fre- traumatic oropharyngeal event such as intubation or pro-
quently to best meet the needs of the child. This includes longed tube feeding. (13) Treatment involves a multidis-
taking into account changes in medical status (which may ciplinary team consisting of a physician, speech therapist,
affect TF tolerance), variations in activity levels or nutrient occupational therapist, dietitian, and child psychologist.
requirements, decreases or increases in oral intake, and growth. The relationship between the child and the caregiver also
plays an important role. Behavioral treatments usually
involve positive reinforcement, modeling (observation
TRANSITION FROM TF TO ORAL FEEDINGS
of other person performing goal behavior), and shaping
The decision to start weaning the child from TF depends on (goal behavior is broken down into components with the
his or her nutritional status, oral motor skills, swallowing final stage of completing the behavior). Other important
ability, and concurrent diseases as well as the readiness of components of treatment are oral motor stimulation,
the patient/caregivers. For most children, TF has a sub- supplementation, and appetite stimulants. (8)(13)(14)(15)
stantial behavioral component such as absence of sucking, (16)(17)(18)
withdrawal from offered food, tongue chewing, gagging,
and food aversion with the sight or smell of food. This
SOCIAL CONSIDERATIONS
makes the process more complicated and challenging.
Sensory food aversions increase the child’s sensitivity to It is important for the health care team to understand that
the sight or smell of food and leads to the fear of trying initiating TF is a difficult decision for the patient and family.

32 Pediatrics in Review
Studies have shown that parents’ decisions about tube inser- thoughtful planning and dedication to deliver safe nutri-
tion are complex. (19) Parents might have a feeling of guilt or tion support. A team approach is necessary. The team
see it as a failure on their part to adequately feed their child. should always include the patient, the patient’s family,
Brotherton et al (20) reported that the key issues based on and caregivers.
parents’ views about the impact of feeding on daily lives of
both the children and their families, included delayed and
disturbed sleep, restricted ability to go out, difficulties in
finding a place to feed, child care problems, negative attitudes Summary
of others toward feeding, and family divisions.
• Based on overwhelming observational studies as well as
Pederson et al (21) reported that factors associated with consensus, tube feeding (TF) is a means of delivering complete
the stress reported by parents of children with an enteral nutrition to those who would otherwise not receive it, thus
feeding tube were severity of their child’s illness/disability, supporting the best possible growth and development. (Evidence
the constant caretaking demands placed on the parents, and quality B, C, D). (4)
the level of support provided by the parents’ social network. • A successful TF program requires input from many clinicians and
It is, therefore, important that health care professionals thoughtful planning and dedication to deliver safe nutrition
support. Based primarily on consensus with some observational
prepare the child and the family for the challenges of TF
and cohort studies, a team approach is necessary (Evidence
and provide them support. quality C and D). (19)(20)
• Based on consensus with the support of some observational
studies, the team should always include the patient, the patient’s
CONCLUSION
family, and caregivers (Evidence quality C and D). (19)(20)
TF is a means of delivering complete nutrition to those who
would otherwise not receive it, thus supporting the best
possible growth and development. A successful TF program References for this article are at http://pedsinreview.aappubli-
requires input from many health care professionals and cations.org/content/38/1/23.

Parent Resources from the AAP at HealthyChildren.org


• Caring for a Premature Baby: What Parents Need to Know: https://www.healthychildren.org/English/ages-stages/baby/preemie/Pages/
Caring-For-A-Premature-Baby.aspx
• Challenges Faced by Parents of Children with Congenital Heart Disease: https://www.healthychildren.org/English/health-issues/
conditions/heart/Pages/Challenges-Faced-by-Parents-of-Children-with-Congenital-Heart-Disease.aspx

Vol. 38 No. 1 JANUARY 2017 33


PIR Quiz
There are two ways to access the journal CME quizzes:
1. Individual CME quizzes are available via a handy blue CME link under the article title in the Table of Contents of any issue.
2. To access all CME articles, click “Journal CME” from Gateway’s orange main menu or go directly to: http://www.aappublications.
org/content/journal-cme.

REQUIREMENTS: Learners
1. An 8-month-old infant is admitted to the hospital for failure to gain weight despite home- can take Pediatrics in
based efforts to provide adequate energy orally. You discuss with the parents the feeding Review quizzes and claim
options of enteral nutrition (EN) and parenteral nutrition (PN). As compared to PN, which of credit online only at:
the following EN features makes it the preferred feeding method in this patient? http://pedsinreview.org.
A. Ability to promote earlier gut function.
B. Delivery cost is similar to PN. To successfully complete
C. Effectiveness is independent of liver function. 2017 Pediatrics in Review
D. Hospital length of stay is similar to that for PN. articles for AMA PRA
E. Infection risk is similar to that for PN. Category 1 CreditTM,
learners must
2. You are asked to lead an orientation program for new nurses at your hospital on oral feeding demonstrate a minimum
alternatives. In which of the following clinical scenarios would an orogastric tube be the performance level of 60%
preferred feeding method? or higher on this
A. Preterm infant (now 32 weeks corrected age) with persistent oxygen requirement. assessment, which
B. Term infant being treated with therapeutic cooling for hypoxic ischemic measures achievement of
encephalopathy. the educational purpose
C. Term infant with 15q11.2-q13 deletion (Angelman syndrome). and/or objectives of this
D. Toddler with a temporal skull fracture and altered mental status. activity. If you score less
E. Toddler with Pierre Robin sequence who requires long-term nutrition support. than 60% on the
assessment, you will be
given additional
3. You are placing a nasointestinal tube in an infant with severe oral aversion who has not
opportunities to answer
tolerated gastric feeding. Which of the following is the most reliable method to confirm the
questions until an overall
successful placement of the tube in this patient?
60% or greater score is
A. Aspiration of bilious stomach contents. achieved.
B. Auscultation over the abdomen.
C. Clinical impression.
D. Radiologic confirmation. This journal-based CME
E. Using a tube/body length nomogram. activity is available
through Dec. 31, 2019,
however, credit will be
4. A 4–year-old girl with dystonic cerebral palsy (Gross Motor Function Classification System V)
recorded in the year in
requires enteral nutrition support because of frequent aspiration pneumonias and poor gut
which the learner
motility. Postpyloric feeding is recommended as the preferred method of enteral feeding. Which
completes the quiz.
of the following statements represents the greatest advantage of postpyloric feeding in this child?
A. Eliminates the risk for reflux aspiration.
B. Has a lower incidence of tube occlusion.
C. Is better tolerated in patients with gastroparesis.
D. Minimizes symptoms of dumping syndrome.
E. Permits a more flexible feeding schedule.

5. You are meeting with your multidisciplinary team to implement a patient’s feeding plan.
You discuss with the team the 2 considerations of bolus feeding versus continuous feeding.
In deciding to choose between the 2 methods, which of the following factors favors bolus
feedings over continuous feedings?
A. Gastroparesis.
B. Intestinal malabsorption.
C. Patient mobility level.
D. Poor nutritional status.
E. Volume of oral intake.

34 Pediatrics in Review
Overcoming Challenges to Care in the Juvenile
Justice System: A Case Study and Commentary
Rebekah J. Savage, MD, MPH,*† Jasmine M. Reese, MD, MPH,*‡ Stephenie Wallace, MD, MSPH,* Timothy Wang, MD,x
Traci Jester, MD, RD,{ Robert Lowe, MD, PhD,** LaKeshia Hyndman,†† Nefertiti Durant, MD, MPH*
*Department of Pediatrics, Division of General Pediatrics and Adolescent Medicine; †Department of Family and Community Medicine, Division of Student
Affairs; xDepartment of Pediatrics; {Department of Pediatrics, Division of Pediatric Gastroenterology, Hepatology, and Nutrition; **Department of Pediatrics,
Division of Pediatric Rheumatology, University of Alabama at Birmingham, Birmingham, AL

Physicians’ Primary Care of Southwest Florida, Fort Myers, FL
††
University of Alabama at Birmingham School of Medicine, Birmingham, AL

Practice Gaps
1. Vulnerable youth populations, such as those in the justice system, have
been found to have disproportionately higher unmet medical needs
that sometimes require innovative approaches to diagnosis and
management. (1)(2)
2. Pediatric clinicians require education about the social and systems-
level challenges to care and the strategies to overcome these
challenges to effectively serve as treating and advocating clinicians for
these patients. (1)

Objectives After completing this article, readers should be able to:

1. Discuss demographic and health care needs of delinquent youth in the


juvenile justice setting.
2. Recognize the role of clinician biases in delivering care to delinquent youth.
3. Discuss challenges to proper diagnosis, management, and treatment of
health conditions in a justice setting and how these factors affect
AUTHOR DISCLOSURE Drs Savage, Reese, patient adherence.
Wallace, Wang, Jester, Lowe, and Durant and
Ms Hyndman have disclosed no financial 4. Discuss how to improve adherence in a vulnerable population,
relationships relevant to this article. This
including family involvement and connection to community resources.
commentary does not contain a discussion of
an unapproved/investigative use of a 5. Recognize the social and systems-level barriers associated with caring
commercial product/device.
for youth with chronic medical conditions, including family stressors,
ABBREVIATIONS transportation, and poor communication by clinicians.
AAP American Academy of Pediatrics
ED Emergency department
EMR Electronic medical record
IBD Inflammatory bowel disease Abstract
JDO Juvenile detention officer
NCCHC National Commission on Youth in the criminal justice system commonly suffer from multiple
Correctional Health Care medical and psychological health problems. Because they likely live in
NSAID Nonsteroidal anti-inflammatory
lower socioeconomic environments, the medical care they receive
drug

Vol. 38 No. 1 JANUARY 2017 35


through the justice system might be their only recent medical care and
can result in the discovery of health problems or chronic illnesses that
must be managed while in the system and beyond. We describe the case
of an adolescent diagnosed with a serious chronic disease during his time
in an urban detention center to illustrate how health workers and justice
staff must use a team approach in caring for this vulnerable population of
children. Barriers to appropriate care, including social and systems-level
challenges, are discussed. The lessons learned in this case can be applied
more broadly to other vulnerable youth populations, including those in
foster care and impoverished communities where the primary care
pediatrician (or other assigned pediatric specialist) is both the leader of
the medical team and an advocate for quality care.

INTRODUCTION juvenile justice health care. (1)(13) According to the NCCHC


guidelines, all youth should undergo health screening upon
Since the Juvenile Court Act of 1899, youth have been
arrival at a detention center. (12) The initial screening rec-
separated from adults in the criminal justice system. (3)
ommended by NCCHC is to assess for urgent medical
Currently in the United States, more than 61,000 youths
problems, contagious diseases, and behaviors such as sui-
younger than age 21 years are incarcerated, detained, or living
cidal ideation. Per NCCHC guidelines, youth should also
in residential homes. There are almost 200 individuals in the
complete a comprehensive health assessment, including a
juvenile justice system for every 100,000 juveniles. (4) Place-
physical examination by a physician, physician assistant, or
ment options for juvenile offenders include residential facil-
nurse practitioner within 7 days of admission. The goal of
ities that house incarcerated adolescents after adjudication and
the NCCHC standards is to address medical and mental
detention centers where adolescents typically spend a short
period of time before adjudication or while awaiting disposi- health needs at the beginning of detainment and provide
tion or transfer to another facility. (5) In recent years, cases continual quality of care throughout detainment. (12)
handled by juvenile courts have been decreasing, with slightly Although urgent medical concerns are appropriately the
more than 1 million cases in 2013 resulting in more than focus of juvenile justice health care, unmet chronic medical
200,000 adolescents being placed in detention centers. (6) and psychological needs are frequently encountered. (1)(9)
Poverty and race are associated with youth criminal (14) In a seminal study on juvenile justice health by Hein
involvement. One out of every 5 youths younger than age 18 and colleagues, (14) almost 50% of detainees had diagnos-
years in the United States lives in poverty, a factor linked to able medical problems. Many of these were chronic condi-
juvenile justice involvement. (7) There are proportionately more tions not yet diagnosed for various reasons, including lack of
delinquency cases for African American youth compared to health care access. For example, Goodwin and colleagues
national census data. (7) Minority status and poverty are both (15) found that 20% of youth in the justice system had a
associated with poor health. (8) Thus, it follows that the health diagnosable mood disorder and almost 20% had asthma
needs of the juvenile justice population are greater, with youth when assessed upon entrance to a detention center. In a
in the justice system disproportionately affected by multiple study by Feinstein et al, (9) at least 10% of youth in a
medical and mental health problems, including sexually trans- detention center had a medical diagnosis requiring fol-
mitted infections, dental problems, substance use disorders, low-up evaluation after discharge, more than 16% had been
trauma histories, and suicidal ideation. (1)(2)(9)(10)(11) treated for mental illness before intake, and only 33%
To address the complexity of health concerns consis- reported having a medical home.
tently, the National Commission on Correctional Health Biases may exist among health workers caring for incar-
Care (NCCHC) has published standards of care for detained cerated or detained youth. (16)(17) Although the medical
and incarcerated youth. (12) In addition, the American team may not intentionally exhibit prejudice or “explicit”
Academy of Pediatrics (AAP) and the Society for Adolescent bias based on race, ethnicity, or institutional status, research
Health and Medicine have published policy statements on shows that clinical decision-making of physicians may be

36 Pediatrics in Review
affected by implicit bias (due to race, ethnicity, gender, and which are read as normal. The patient is diagnosed with
other factors). (17) For youth in the justice setting, implicit muscular sprain and returned to the detention center with a
bias may be related to suspicion for secondary gain (or a plan to continue NSAID therapy and rest.
change in a social situation that results from a certain Two days later the patient develops new left knee swell-
complaint or physical symptom). (18) Such bias is uncon- ing. Knee radiographs obtained as an outpatient are read as
scious or “automatic” (17) and may play a role in delaying normal. On reevaluation, the detention center physician
medical diagnosis and treatment. Of the articles written to notes worsening of left knee swelling, with increased pain
date on detained youth, none have addressed the possibility and limited range of motion. The adolescent has no other
of implicit or explicit clinician bias. systemic symptoms such as fever. Results of laboratory
Quality care for the juvenile justice population is chal- studies obtained by the detention center nurse are concern-
lenging due to the higher prevalence of health concerns and ing for inflammation or infection: elevated white blood cell
social issues in the context of a constrained system of care. count of 12,850/µL (12.85  109/L), erythrocyte sedimen-
To explore these challenges further, we report the case of an tation rate of 68 mm/h (normal 0-15 mm/h), and C-reactive
adolescent detained at a juvenile detention center in an protein of 250 mg/L (2,381 nmol/L) (normal <10 mg/L
urban southeastern United States city who was diagnosed [95.24 nmol/L]). Due to the concern for acute infection of
with a complicated chronic medical condition during incar- the knee joint, he is transported immediately to the ED,
ceration. We describe the challenges of meeting the health where he is admitted to the hospital and orthopedic surgery
care demands of detained youth and discuss the care of is consulted. Throughout his short hospitalization, joint
chronic illness among vulnerable youth populations. By fluid cultures remain negative, and parenteral antibiotics
sharing obstacles and lessons learned, we hope to foster a are discontinued after 2 days. He improves without antibi-
conversation among pediatric clinicians that leads to impro- otics and is discharged to the detention center.
ved care for this sometimes-overlooked patient population. Two days after discharge, the patient has a stable left
Informed consent for the sharing of deidentified informa- knee examination but continues to have multiple joint com-
tion was obtained from the patient and his mother. plaints with refusal to ambulate. An appointment at the local
children’s hospital adolescent clinic is coordinated by the
detention center physician, where rheumatology is con-
CASE PRESENTATION
sulted to assist in his evaluation. In the clinic, he complains
A 15-year-old African American adolescent presents to the of worsening left knee swelling; continued jaw pain precluding
county’s youth detention facility, where an initial health normal food intake; and new-onset pain in multiple joints,
screening reveals a history of asthma and normal findings including the left elbow, right shoulder, right wrist, and right
on physical examination. After 1 to 2 weeks at the detention hand. The patient’s right hand is swollen and tender to palpation.
facility, the patient complains of right hip pain, which is For the first time, the patient reports an unclear number of
evaluated by the physician working at the detention center. episodes of nonbloody diarrhea for the past 2 weeks. Physicians
After reassuring findings on the evaluation (negative for note weight loss since the previous hospitalization. Given his
trauma or history of arthritis), the physician advises rest and deterioration, he is admitted again to the local children’s hos-
nonsteroidal anti-inflammatory drugs (NSAIDs). In sub- pital. Laboratory tests and imaging of the involved joints show
sequent weeks, the adolescent develops multiple other joint evidence of inflammation and arthritis of his right wrist.
complaints, including hip and jaw pain in the context of After ruling out infectious causes, the patient is given
worsening behavior and altercations with other detainees corticosteroids to address a working diagnosis of inflam-
and detention officers. NSAIDs are administered for 1 week matory arthropathy. Because of the history of diarrhea, weight
with no improvement in the jaw pain. Detention officers and loss, a perianal skin tag, and stool studies positive for blood,
health care staff express concern that the patient is attend- gastroenterology is consulted about possible inflammatory
ing medical visits for secondary gain (ie, missing required bowel disease (IBD). Laboratory results to investigate IBD are
activities with a possibility to travel outside of the facility for pending at the time of discharge. He is discharged with
medical evaluation). The patient eventually develops difficulty instructions to follow up with multiple specialists, including
eating solid food due to jaw pain, refusing many of his meals. rheumatology, physical therapy, and gastroenterology, de-
He also becomes unable to ambulate due to worsening hip pending on stool laboratory results. Discharge medications
pain. Because of this change in status, the physician sends the included high-dose NSAIDs and corticosteroids.
patient to the emergency department (ED) at the local chil- Because of the complexity of his illness and specialty
dren’s hospital. He is evaluated with radiographs of the hips, needs after discharge from the hospital, the patient is

Vol. 38 No. 1 JANUARY 2017 37


granted a conditional medical probation to be released to his mother discuss the diagnosis with both gastroenterology
home early (before adjudication). Two weeks after discharge and rheumatology physicians during a coordinated joint
from the hospital, stool laboratory results suggest possible IBD. clinic visit. Because of the details of his charge, the patient’s
The inpatient team’s social worker contacts both the patient’s sentencing is delayed many times, and, therefore, he re-
probation officer and his family regarding the importance of mains in the detention center for multiple months. He is
returning to appointments to discuss these results and the initially treated with methotrexate (an injectable antineo-
need for further evaluations. However, the patient and family plastic agent) with little improvement. After a few weeks,
do not appear for rheumatology or gastroenterology appoint- an injectable tumor necrosis factor-a inhibitor medication
ments and do not return phone calls. (adalimumab) is added, although another delay in treatment
Because of the lack of attendance at medical appoint- results from pharmacy and shipping confusion. At the end
ments, the physician and inpatient social worker contact the of his time in short-term detention, the patient is transferred
department of human resources to obtain family assistance. to a youth prison facility for incarceration and receives
Finally, 2 months after hospital discharge and weeks after transportation to the same hospital for subspecialist care.
the return of the concerning laboratory results, the patient is
ordered by the judge to reenter the short-term detention DISCUSSION
facility due to violation of his medical probation.
Challenges to the care of detained adolescents with complex
On reentry, he is evaluated medically at the detention
chronic medical conditions have been infrequently dis-
center and found to have problems (eg, weight gain) due to the
cussed in the literature on juvenile justice health. The case
long-term use of high-dose corticosteroid medication. Detention
presented in this article illustrates numerous challenges and
staff and physicians are unable to contact his guardians regard-
lessons learned (Table 1).
ing the immediate need for home medications as well as further
diagnostic evaluations. Because of the inability to make contact
with his guardians, the patient’s medications are obtained from Challenge 1: Communication
pharmacies by juvenile detention officers (JDOs) and from Communication across systems and disciplines is key to
nursing staff. Detention center physicians consult with gastro- provision of appropriate care. Challenges to communication
enterology and rheumatology about tapering corticosteroids occurred at several levels: 1) patient-clinician, 2) system-
while monitoring blood pressure and symptoms closely at least system, and 3) clinician-family.
3 times per week. With the help of the courts, permission from The first challenge in communication occurred between
his guardians is finally granted to complete his diagnostic the adolescent himself and clinicians. Health screenings and
evaluation while at the detention center. assessments are the standard of care (12) but rely on the self-
Biopsies from gastroenterology procedures verify the report of the detainee and, thus, can be problematic. (9) Health
diagnosis of ileocolonic Crohn disease. The patient and complaints during detainment were addressed quickly but were

TABLE 1. Improving Health Care for Youth in Vulnerable Settings:


Challenges and Proposed Improvements
CHALLENGE PROPOSED IMPROVEMENTS EXAMPLE ACTIVITIES FROM CASE

Communication Assign a team coordinator Coordinator connects clinicians via email correspondence
and asks for feedback and advice regarding medical
details.
Assign a family advocate Advocate works with social and legal resources to maintain
family communication.
Clinician biases Increase objective measures of health status Clinicians document vital signs, including weight, at every
medical visit.
Improve awareness of bias Detention center staff and health workers meet together for
training and communication purposes.
Family and social barriers Involve community resources early in process Team leader connects family with human resources worker.
Use team approach to engagement Medical, legal, law enforcement, and community agency
staff work together to meet needs of family.
Resource and time constraints Educate nonmedical staff on health care needs Medical staff meets with management of detention center
to reallocate resources to meet needs of patient.

38 Pediatrics in Review
difficult to assess completely due to lack of appropriate com- and social situation led to improvement in both communi-
munication with the patient regarding the clinicians’ medical cation and quality of care throughout his medical course.
concerns. In addition, patient confidentiality was restricted,
Challenge 2: Provider Biases
given the presence of detention officers and constant shackling
Medical complaints in the context of poor behavior may
during medical visits outside of the facility, leading to a lack of
have led to biases by medical and detention staff. Behavior
information sharing between patient and clinician.
exhibited by the patient, including altercations with other
The second communication barrier occurred between
detainees and refusal to eat solid foods, led to concerns that
systems of care: the justice system and the hospital-based
some of the patient’s medical complaints may have arisen to
system. Clinicians documenting information at the deten-
avoid consequences or required activities (ie, meals). These
tion center used only paper records, but hospital records
examples display the potential dangers of health care biases,
were electronic. Electronic medical records (EMRs) are an
both explicit and implicit. In the patient’s case, clinician
essential component of the health care reforms of the
biases due to the patient’s detained status, gender, or race
Affordable Care Act because of improved efficiency and
may have negatively affected or delayed appropriate care.
quality of care. (19) The AAP recommends use of EMRs by
Research on this important phenomenon is lacking.
correctional health care facilities. (1) In this case, the ED and
Clinician awareness of implicit bias based on detention
inpatient attending physicians could not review the patient’s
status and behavior can decrease the disparity in care that
medical chart via EMR. The paper chart from the justice facil-
may result. (17) Strategies to combat such bias include
ity was not transported to the hospital with the patient. This led
treating all patients individually without comparing them
to a delay in hospital documentation of details such as diffi-
to other detainees. (17) In this case, to overcome bias, the
culty eating and weight loss, which were documented only in
medical team used objective measures of medical status (eg,
the paper chart. Similarly, different JDOs accompanied the
physical examination) and improved upon their documen-
patient to each medical visit, none of whom consistently
tation of his observed behaviors. Through consistent mon-
understood his medical course. This led to miscommunication
itoring of agreed-upon parameters, clinicians and staff
between medical and justice system providers that affected the
improved clinical assessments. These assessments led to his
patient’s timely diagnosis and treatment.
hospital admission and disease diagnosis. Developing stand-
Third, communication between the medical team and the
ard objective clinical measures to monitor such patients may be
patient’s family was limited due to several factors, including
helpful. Clinicians and detention center staff should confront
transportation and work scheduling. During his second hospital
their own implicit biases for all patients through training
stay, discharge planning included subspecialty appointments
programs and enhanced communication about this important
and treatment of arthritis with potentially dangerous medica-
topic to increase awareness and improve care for all youth,
tions. The medical social worker assisted the team in commu-
especially those in juvenile justice settings.
nicating with the family briefly before discharge. However, as in
many rheumatologic illnesses, diagnosis takes a prolonged Challenge 3: Family and Social Barriers
period of time. The diagnostic process and treatment plan Family engagement is essential to the care of detained
can be confusing to clinicians, patients, and family members. adolescents. Research indicates that 50% of parents of
Similarly, no clinicians phoned his documented primary care detainees who required follow-up evaluation and treatment
provider to discuss his course and future care needs. for chronic conditions were not responsive to staff phone
In summary, communication can be a great pitfall to calls and did not make efforts to assist in this process. (9)
caring for chronic diseases in the juvenile justice system. In Our patient was placed on probation after his second
this case, the pediatrician working in the justice system took hospitalization to allow for the family to take responsibility
responsibility for the patient’s care needs, which led to for his care. His family did not adhere to medical recommen-
improved clinician and family communication. She made dations. After his return to the center, he required multiple
phone calls to his parents on multiple occasions before and medication changes and was difficult to manage from an
after hospitalizations. When he failed to present at subspe- arthritis standpoint, requiring escalation of therapy and a
cialty appointments, she coordinated the legal and social prolonged corticosteroid taper. The ensuing weeks involved
resources necessary for him to receive care (eg, contacting coordination of specialty appointments and attempts at com-
the county’s human resource department). She began an munication with the patient’s mother. She initially did not
email correspondence among all involved in his care, includ- return phone calls. Appointments, procedures, and medication
ing the detention center nurse who was at the center every changes were delayed because of the necessity of her involve-
weekday. Coordination of care in this complicated medical ment as legal guardian.

Vol. 38 No. 1 JANUARY 2017 39


TABLE 2. Members of the Care Team for Youth in the Justice System
TEAM MEMBER ROLE

Patient Focus of the care team


Patient’s family Advocates for patient
Provides resources to patient
Communicates resource needs
Justice center physician/clinician Leads and coordinates care of patient
Promotes communication among team members
Justice center nursing staff Provides direct care and monitors progress
Acts as liaison among patient, family, medical staff, and nonmedical staff
Medical or other health social worker Assists nursing and medical staff with patient care needs
Engages family in care
Subspecialty physician/clinician Provides specialty expertise on illness
Law enforcement officer (eg, juvenile detention officer) Monitors and reports on patient’s behavior
Directs disciplinary actions to promote health and positive development
Judicial official (eg, probation officer) Represents and serves the patient within the legal system
Understands family dynamics and social concerns
Community agency (eg, human resources worker) Assesses need for assistance
Connects patient and family with needed resources
Medical home physician/clinician Maintains relationship with patient and family
Manages health care and quality of life over time

Despite these challenges, the detention center medical administer his own medications (injections) during his time
team continued to attempt to engage the family. The center’s in detention, monitoring him with each administration and
nurse phoned the adolescent’s mother on a daily basis to empowering him to take responsibility in the absence of his
attempt communication of medical updates and reported family.
needs, including medications to be filled or brought in to the Young men and women in the justice system are more
center. Medical staff and judicial officials (eg, probation likely to have low social support, which hinders chronic care
officers) communicated frequently. Medical visits to explain management. (1) Such social barriers at baseline (eg, trans-
Crohn disease and arthritis were coordinated between sub- portation) may escalate when a chronic disease is diagnosed
specialty providers and the patient and his mother to edu- during detention or incarceration. Even without chronic
cate them on the details of the diagnosis. With the help of disease, detained youth have poor follow-up care after release
all members of the team and some persistence, his mother from detention. (9) One challenge to accessing this follow-up
became more engaged in his care and eventually could be is that most detainees have health insurance, especially
relied upon for medication refills and other needs. Im- Medicaid, suspended during imprisonment, which can cause
portantly, the detention center nurse taught the patient to confusion about medical care coverage after release. (20)(21)

TABLE 3. Selected Resources for the Clinician


NAME WEBSITE

National Commission on Correctional Health Care (NCCHC) www.ncchc.org


Society for Adolescent Health and Medicine www.adolescenthealth.org
Office of Justice Programs, US Department of Justice www.ojp.gov/programs/juvjustice.htm
Substance Abuse and Mental Health Services Administration (GAINS Center for Behavioral www.samhsa.gov
Health and Justice Transformation) www.samhsa.gov/gains-center
American Academy of Pediatrics (Advocacy & Policy) www.aap.org/en-us/advocacy-and-policy
Individual state departments of public health and associated mental health divisions

40 Pediatrics in Review
Our patient’s lack of follow-up was likely due to poor and socioeconomic factors such as poverty. Thus, many
communication with his family about his illness, leading to challenges arise when a detained youth is diagnosed with
poor understanding of his health care needs in the context of a chronic illness. Poor behavior on the part of the adolescent
broader social and economic challenges. We attempted to can lead to clinician biases; lack of family engagement and
overcome this challenge through involvement of multiple understanding can result in nonadherence; poor communi-
team members, including detention staff, probation offi- cation can exist between systems of care; and many systems
cers, judges, and medical providers (Table 2). Future cases and individuals must deal with resource deprivation.
may benefit from early linkage to community agencies, The primary care pediatrician (or other assigned pediat-
including human resources and a community primary care ric specialist) leads the medical team through coordination
provider or a “medical home” before discharge or release. In of care, consistent communication, and patient advocacy.
most cases of youth involved in the justice system, the With persistence in family engagement, creative resource
probation officer is an appropriate starting point for pedi- allocation, and awareness of biases, quality of care can
atric clinicians who need to communicate with a legal improve for all patients. Pediatric clinicians are the optimal
representative to provide appropriate care. advocates for vulnerable youth populations such as those
in the justice system. Such public health advocates are
Challenge 4: Resource and Time Constraints uniquely positioned to educate health care administration
Although health care standards for detained youth exist, on appropriate health care delivery. Youth health providers
resources such as staffing are not always readily available. For and policymakers must ensure that patients receive devel-
example, detention center staff members are required to opmentally appropriate care that is no less than care for
transport youth to specialty medical appointments, laboratory patients outside of the justice setting. Detained youth should
evaluations, and other studies. With complex medical diseases, retain their insurance coverage and have equal access to
these appointments may require a full day, using a paid staff primary and specialty care. Pediatric clinicians and public
member for 1 detainee. In a resource-deprived setting, staff health practitioners have an opportunity to engage and edu-
must be educated on how to appropriately address the medical cate patients, their families, and communities to improve the
needs of the young detainee with a complex medical illness. care of all vulnerable youth populations, especially those
A team approach assisted in delivery of care via commu- within the juvenile justice system.
nication among the medical staff, detention officers, and
management personnel of the center itself. For example, the
center’s director and other staff received education on the
patient’s illness and its serious nature, including the daily Summary
medication schedule and possible complications. Resource • Youth involved in the juvenile justice system are a vulnerable
allocation was adjusted to meet transportation needs, such as pediatric population who require special attention due to
visits to pharmacies by the center’s nurse or the probation disproportionate rates of undiagnosed and untreated medical
officer to ensure that the patient obtained his medication in a illnesses that can lead to health disparities and poor outcomes.
timely fashion. Overall, to provide the necessary resources in • Challenges to care for justice-involved youth include social
a unique situation, the medical team and staff had to take an factors such as family involvement, communication with
clinicians, and transportation.
individualized and unique approach. Clinicians may obtain
• Addressing systems-level barriers to care, such as lack of
more information on the care of justice-involved and other
resources in justice facilities and poor access to care for
vulnerable youth at several online resources (Table 3).
impoverished youth, can offer opportunities for improvement
through policy changes and effective collaboration.

CONCLUSION • Quality pediatric primary care is best implemented with a team


approach to appropriately evaluate and manage the acute and
Our patient’s story illustrates the complexity of diagnosing chronic health problems encountered in this setting. This
and caring for chronically ill youth in a justice setting. The includes linkage to community resources and a medical home
after release from the justice system.
lessons learned can be applied more broadly to other vulner-
able youth populations, such as those in foster care and
impoverished communities. Detained youth have vulnerabil-
ities on multiple socioecological levels, including individual References for this article are at http://pedsinreview.aappubli-
high-risk behaviors and mental health needs, as well as family cations.org/content/38/1/35.

Vol. 38 No. 1 JANUARY 2017 41


PIR Quiz
There are two ways to access the journal CME quizzes:
1. Individual CME quizzes are available via a handy blue CME link under the article title in the Table of Contents of any issue.
2. To access all CME articles, click “Journal CME” from Gateway’s orange main menu or go directly to: http://www.aappublications.
org/content/journal-cme.

REQUIREMENTS: Learners
1. A 14-year-old boy arrives at a detention center where he will be housed while he can take Pediatrics in
awaits adjudication. He undergoes health screening upon arrival. In addition to Review quizzes and claim
assessing for urgent medical problems and behaviors such as suicidal ideation, credit online only at:
which of the following conditions must be included in the routine health http://pedsinreview.org.
screen?
A. Asthma. To successfully complete
B. Contagious diseases. 2017 Pediatrics in Review
C. Mood disorder. articles for AMA PRA
D. Rheumatologic disease. Category 1 CreditTM,
E. Weight loss. learners must
demonstrate a minimum
2. A 16-year-old boy undergoes a health screening upon arrival at a detention center. The performance level of 60%
screening reveals no urgent medical problems. In the context of this initial admission or higher on this
screening, a comprehensive health assessment including a physical examination should be assessment, which
performed within what time period from admission? measures achievement of
A. 48 hours. the educational purpose
B. 7 days. and/or objectives of this
C. 14 days. activity. If you score less
D. 30 days. than 60% on the
E. 60 days. assessment, you will be
given additional
opportunities to answer
3. A medical director of a juvenile detention center conducts monthly meetings with the
questions until an overall
detention center health care team and staff to discuss operational and training issues.
60% or greater score is
Which of the following is the most effective measure to adopt to avoid disparity in care that
achieved.
may result from implicit biases?
A. Giving context by comparing patients to other detainees.
B. Implicit bias toward secondary gain as a motivator for symptom reporting. This journal-based CME
C. Provider awareness of implicit bias based on behavior. activity is available
D. Reliance upon patient reporting of medical conditions. through Dec. 31, 2019,
E. Use of subjective measures of medical status. however, credit will be
recorded in the year in
which the learner
4. A 13-year-old girl receives a provisional diagnosis of lupus while at a detention center.
completes the quiz.
Medical follow-up is required after additional test results are received and to monitor her
response to initial therapy. The hospital clinician attempts to call the patient’s family
regarding appointments but is unable to reach a parent or guardian. What percentage of
parents of detainees requiring follow-up for chronic conditions is not responsive to staff
phone calls?
A. 15%.
B. 30%.
C. 50%.
D. 70%.
E. 85%.

5. A 17-year-old boy receives a diagnosis of epilepsy while detained. After release, the patient
fails to attend multiple pediatric and neurology appointments, with no response to multiple
phone calls. Which of the following might be the most appropriate starting point of contact
for pediatric clinicians who need to communicate with a legal representative to provide
appropriate care?

42 Pediatrics in Review
A. Judge.
B. Juvenile detention officer.
C. Juvenile dependency lawyer.
D. Probation officer.
E. Public defender.

Parent Resources from the AAP at HealthyChildren.org


• Out-of-Control Teens: PINS Petitions & the Juvenile Justice System: https://www.healthychildren.org/English/health-issues/conditions/
emotional-problems/Pages/When-a-Teenager-is-Out-of-Control.aspx
• Foster Parents: FAQs: https://www.healthychildren.org/English/family-life/family-dynamics/adoption-and-foster-care/Pages/Foster-
Parents-FAQs.aspx

Vol. 38 No. 1 JANUARY 2017 43


Poor Growth With Presence of a Pituitary
1 Lesion in an 11-year-old Boy

Alexander S. Karageorgiadis, MD,*† Charalampos Lyssikatos, MD,*


Elena Belyavskaya, MD,* Georgios Z. Papadakis, MD,‡
Nicholas J. Patronas, MD,‡ Maya B. Lodish, MD,*
Constantine A. Stratakis, MD, DSc*
*Section on Endocrinology and Genetics, Eunice Kennedy Shriver National Institute of Child Health
and Human Development, National Institutes of Health, Bethesda, MD

Department of Pediatrics, Georgetown University Hospital, Washington, DC

Department of Radiology and Imaging Sciences, Clinical Center, National Institutes of Health,
Bethesda, MD

EDITOR’S NOTE
Beginning with our January 2017 issue, Pediatrics in Review will publish three additional
Index of Suspicion cases each month. All cases will be available in their entirety online, but
only the Presentation for each case will be published in the print edition of the journal. We
anticipate the move to online will encourage authors to use more images and video in their
case submissions.

PRESENTATION

An 11-year-old boy presents to his primary pediatrician due to poor growth. His
EDITOR’S NOTE
We invite readers to contribute Index of height was at the 25th percentile between ages 6 and 10 years and now has fallen
Suspicion cases through the PIR manuscript to the 10th percentile. He also complains of fatigue and has a 1-month history
submission system at: https://mc. of polyuria, polydipsia, and nocturia. Review of symptoms is negative for weight
manuscriptcentral.com/pir. loss, headaches, vision changes, gynecomastia, breast discharge, nausea, vomit-
ing, and cold and heat intolerance. His past medical and family histories are not
AUTHOR DISCLOSURE Drs Karageorgiadis,
Lyssikatos, Belyavskaya, Papadakis, Patronas, contributory and his development has been normal.
Lodish, and Stratakis have disclosed no On physical examination, his temperature is 96.8°F (36ºC), blood pressure is
financial relationships relevant to this article. 86/52 mm Hg, heart rate is 59 beats/min, respiratory rate is 18 breaths/min, and
This commentary does not contain a
discussion of an unapproved/investigative oxygen saturation is 100% in room air. His standing height is 138.23 cm (10th per-
use of a commercial product/device. centile), weight is 31.5 kg (10th percentile), and body mass index is 16.5 (10th-50th
percentile). His neurologic examination yields unremarkable results. His pubic hair
is at Sexual Maturity Rating stage 2 and testicular volume of both testes is 10 mL.
Laboratory evaluation shows free thyroxine (T4) of 0.76 ng/dL (9.78 pmol/L)
(normal range 0.93-1.60 ng/dL [11.97-20.59 pmol/L]), total T4 of 4.3 mg/dL (55.35
nmol/L) (normal range 4.5-12 mg/dL [57.92-154.45 nmol/L]), thyrotropin (TSH)
of 1.97 mIU/L (normal range 0.45-4.5 mIU/L), and insulinlike growth factor
(IGF-1) of 103 ng/mL (13.49 nmol/L) (normal range for an 11-year-old male:
112-454 ng/mL [14.67-59.47 nmol/L]). His complete blood cell count, urinalysis
results, and electrolyte values are within normal limits, and his evaluation for
celiac disease is negative. Due to these results, the pediatrician orders mag-
netic resonance imaging (MRI) of the pituitary, which shows a lesion within the

44 Pediatrics in Review
Figure. A and B. Initial magnetic resonance
imaging of the brain showing a pituitary mass
within the sella and suprasellar cistern. C. Pituitary
mass 3 months after therapy.

sella and suprasellar cistern. The greatest vertical, anterior- The Case Discussion and Suggested Readings appear with the
posterior, and transverse diameters of this lesion measure online version of this article at http://pedsinreview.aappublica-
19, 21, and 25 mm, respectively (Fig). tions.org/content/38/1/44.

Vol. 38 No. 1 JANUARY 2017 45


DISCUSSION In girls with prolactinomas, microadenomas are encoun-
tered more frequently, and their clinical presentation is
The patient underwent imaging immediately after initial
associated with galactorrhea and hypogonadotrophic hypo-
results became available, without further referrals or addi-
gonadism. Interestingly, males have a greater incidence of
tional laboratory evaluation. Clinicians could have consid-
macroadenomas, which can cause neuro-ophthalmologic
ered referral to pediatric endocrinology before proceeding
signs and symptoms. Prolactinomas in adult males have
with imaging to evaluate pituitary function with further
been associated with hypopituitarism, with the most com-
tests. However, the presence of low free T4 with normal
mon form being growth hormone deficiency and hypogonad-
TSH and low IGF-1 values suggested central hypothyroid-
ism; hypocortisolism is less common. The diameter of the
ism and growth hormone deficiency, which justify the
adenoma and its prolactin secretion have been shown to affect
decision for pituitary imaging, especially taking into con-
the degree of both hypogonadism and hypothyroidism.
sideration the patient’s symptomatology.
Diagnosis of a prolactinoma requires both the laboratory
The patient was subsequently referred to neurosurgery for
results of persistent hyperprolactinemia and imaging stud-
trans-sphenoidal resection of the pituitary mass that was
ies showing an adenoma. Prolactin values can be related
suspected to be a craniopharyngioma. Before the scheduled
to the size of the tumor. Most patients with prolactin
surgery, the patient’s family asked for a second opinion. Al-
concentrations greater than 150 mg/L (6,521.70 pmol/L)
though a prolactinoma was considered as a possibility for his
(normal range 2-25 mg/L [86.96-1,086.95 pmol/L]) have a
pituitary tumor, his prolactin values had never been measured.
prolactinoma, but clinicians cannot rule out prolactinoma
Therefore, the biochemical evaluation was repeated, and the
if the prolactin values are only moderately increased. The
prolactin measurement was added. His prolactin was found to
abnormal expression of genes such as the pituitary tumor-
be substantially elevated at 996 mg/L (43,304.09 pmol/L)
transforming gene (PTTG) and high mobility group A2 gene
(normal range 2-25 mg/L [86.96-1,086.95 pmol/L]), which
(HMGA2) has been associated with the development of
led to the diagnosis of a prolactin-secreting macroadenoma.
pituitary tumors, including prolactinomas.

Differential Diagnosis Management


Craniopharyngiomas are the most common pituitary tumors Prolactinomas are treated conservatively with dopaminer-
in children, accounting for approximately 90% of all pituitary gic agonists as the initial therapy of choice to shrink the pitui-
neoplasms and 6% to 13% of all intracranial tumors. Pituitary tary mass. Such treatment offers the best chance for normal
adenomas are rare and account for less than 3% of all supra- pubertal development, gonadal function, and future fertility.
tentorial tumors, with prolactinomas being the most common Surgery (usually with a trans-sphenoidal approach) is only
(48%-52%). Other conditions that could present with similar performed in cases of drug therapy failure or for neurosurgical
symptoms and lesions on imaging studies are cystic lesions, emergencies such as apoplexy. Radiotherapy is very rarely used
such as Rathke cleft cysts, teratomas, or ependymomas, and and only after failed medication therapy and/or surgery.
even more rarely blastomas, germ cell tumors (50% of which
are usually germinomas), and metastases from other primary Patient Course
tumor locations (a relatively common event in adult patients The patient was started on 0.5 mg cabergoline, a dopamine
with pituitary tumors but very rare in children). receptor agonist that blocks prolactin secretion from the
Taking into consideration the fact that our patient did not pituitary gland, twice a week and 25 mg levothyroxine
present with galactorrhea or any neuro-ophthalmologic signs, once daily. After 1 month of cabergoline treatment, his
which may occur in a boy with a prolactinoma, craniopharyn- prolactin concentrations had substantially decreased to
gioma or another cystic lesion was considered as the most li- 15.1 mg/L (656.52 pmol/L). During his follow-up evaluation
kely initial diagnosis. In addition, the noncontrast MRI showed 3 months later, the pituitary MRI showed a decrease in the
a nonhomogeneous appearance of the tumor with an enhanc- size of the adenoma (1.6  1.7 cm) (Fig 1C), the prolactin
ing spot, which is more characteristic of a craniopharyngioma. values dropped to 5.5 mg/L (239.12 pmol/L), and his thyroid
However, most probably this was due to hemorrhage because it hormones and IGF-1 values returned to normal range. He
completely resolved in subsequent imaging studies. had grown 1.2 cm during those 4 months, and his puberty
had progressed, with more pubic hair and increase in testic-
The Condition ular size. Use of cabergoline, especially long-term, has been
Prolactinomas are the most common hormone-secreting associated with mild-to-moderate tricuspid regurgitation
pituitary tumors and have been reported to occur at all ages. and subtle changes in cardiac valves such as calcifications,
thickening, and increased mitral tenting. Because of this con- Fideleff HL, Boquete HR, Suárez MG, Azaretzky M.
Prolactinoma in children and adolescents. Horm Res.
cern for potential cabergoline-associated valvulopathy, baseline
2009;72(4):197–205
echocardiography was also performed and results were normal. Jagannathan J, Kanter AS, Sheehan JP, Jane JA Jr, Laws ER Jr. Benign
brain tumors: sellar/parasellar tumors. Neurol Clin. 2007;25
(4):1231–1249, xi
Lessons for the Clinician Kars M, Pereira AM, Bax JJ, Romijn JA. Cabergoline and cardiac valve
• Patients with prolactinomas do not always present with disease in prolactinoma patients: additional studies during
characteristic clinical manifestations. long-term treatment are required. Eur J Endocrinol. 2008;159
(4):363–367
• Prolactin must be measured in all patients suspected to
Peng J, Qiu M, Qi S, Li D, Peng Y. Hypopituitarism patterns among
have a pituitary mass, even if the patient does not present adult males with prolactinomas. Clin Neurol Neurosurg.
with typical clinical manifestations of prolactinoma. 2016;144:112–118
• A proper diagnosis of pituitary mass lesions can avoid un- Stanley T. Diagnosis of growth hormone deficiency in
childhood. Curr Opin Endocrinol Diabetes Obes. 2012;19
necessary surgery and/or radiotherapy and, thus, reduce (1):47–52
morbidity and mortality. Yamada M, Mori M. Mechanisms related to the pathophysiology and
management of central hypothyroidism. Nat Clin Pract Endocrinol
Suggested Readings Metab. 2008;4(12):683–694
Yang I, Sughrue ME, Rutkowski MJ, et al. Craniopharyngioma: a
Delman BN. Imaging of pediatric pituitary abnormalities. Endocrinol comparison of tumor control with various treatment strategies.
Metab Clin North Am. 2009;38(4):673–698 Neurosurg Focus. 2010;28(4):E5
2 Fever and Back Pain in 13-year-old Girl

Reem Shawar, MD,* Pisespong Patamasucon, MD,* Shawn Rowles, MD*


*Pediatrics, University of Nevada School of Medicine, Las Vegas, NV

PRESENTATION

A previously healthy 13-year-old girl who plays hockey presents with fever of 6
AUTHOR DISCLOSURE Drs Shawar,
Patamasucon, and Rowles have disclosed no days’ duration accompanied by lower back pain and generalized myalgia. She
financial relationships relevant to this article. initially went to an urgent care facility, where she had normal findings on
This commentary does not contain a
urinalysis and a negative flu antigen test. She was diagnosed with a viral illness,
discussion of an unapproved/investigative
use of a commercial product/device. but her symptoms persisted. Today she is experiencing a severe throbbing frontal
headache and nausea. She has no history of furuncles, intravenous drug use, or
recent blunt spinal trauma. Review of symptoms is negative.
Initial vital signs show temperature of 99.2°F (37.3°C), heart rate of 130 beats/
min, and blood pressure of 126/67 mm Hg. Physical examination of the girl, who
is sitting uncomfortably in bed, reveals localized tenderness to palpation in the
lower lumbar region. A nonradiating grade 2/6 systolic ejection murmur is best
heard at the apex. There are no skin lesions or rashes. The patient is alert and
oriented to person, place, and time, with intact short- and long-term memory.
Cranial nerves II through XII are intact. Strength is 5/5 in all extremities, with
normal tone and sensations to pain, temperature, and light touch. She has normal
gait with intact finger-to-nose touch and aligned ankle-over-tibia. Her deep tendon
reflexes are 2þ throughout and Brudzinski and Kernig signs are both negative.
Laboratory evaluation reveals a white blood cell (WBC) count of 10,200/mL
(10.2  109/L) with 79.9% granulocytes, 10.7% lymphocytes, 9.1% monocytes,
0.1% eosinophils, and 0.2% basophils. Her erythrocyte sedimentation rate is 36
mm/h and C-reactive protein measures 281.5 mg/L (2,681.01 nmol/L).

Figure 1. T1-weighted magnetic resonance imaging of lumbar spine after contrast.

46 Pediatrics in Review
During her stay in the emergency department, the pa- evaluation with magnetic resonance imaging (MRI) of the
tient begins to experience neck pain. A lumbar puncture lumbar spine to confirm the diagnosis (Fig 1).
is performed after results of computed tomography scan of
the head are within normal limits, with no space-occupying The Case Discussion and Suggested Readings appear with the
lesion. The fluid drawn from the lumbar puncture is de- online version of this article at http://pedsinreview.aappublica-
scribed as purulent and mucoid, which leads to further tions.org/content/38/1/46.

Vol. 38 No. 1 JANUARY 2017 47


DISCUSSION clinicians placed a peripherally inserted central catheter,
which allowed her to complete her antibiotic course at
The purulent fluid drawn from the lumbar puncture was
home.
sent for Gram stain, bacterial culture, and cell count. The
Gram stain showed many WBCs and Gram-positive cocci in
The Condition
clusters, and the cell count revealed more than 10,000/mm3
An epidural abscess is a rare but serious condition that
of WBCs with undetectable protein or glucose. The Gram
requires emergent diagnosis and treatment. It is a localized
stain from the blood culture drawn on admission also
collection of pus between the dura mater and the overlying
showed many WBCs and gram-positive cocci in clusters.
skull or vertebral column that can extend longitudinally over
MRI of the lumbar spine with contrast was performed
many segments of the spinal cord because there are no
(Fig 2). The lumbar puncture results and MRI findings
resisting structures. It can occur either by contiguous spread
confirmed the diagnosis of a spinal epidural abscess with
or via hematogenous dissemination from furuncles or
paraspinal musculature abscess.
urinary tract or wound infections. It may also result as a
Upon admission of the girl to the general pediatric floor,
complication from spinal surgery or, rarely, lumbar punc-
the neurosurgeon and pediatric infectious disease specialist
ture. Minor trauma to the spine, which was a possibility for
and pediatric cardiologist were consulted. The patient was
this young hockey player, may also be a cause of epidural
given a 24-hour course of dexamethasone and treated with
abscess. Multiple studies, primarily in adult populations,
vancomycin and ceftriaxone. The neurosurgeon did not rec-
have shown that predisposing conditions include diabetes
ommend any surgical intervention because the abscess
mellitus, alcoholism, infection with human immunodefi-
was partially drained during the lumbar puncture. The
ciency virus, and any spinal abnormality or intervention.
pediatric cardiologist performed echocardiography within
The most common organism isolated is S aureus, with an
24 hours of admission, which did not show any evidence of
increasing prevalence of the methicillin-resistant strain.
endocarditis. During her hospital stay, the culture obtained
Other infecting organisms include aerobic and anaerobic
from the lumbar puncture showed heavy growth of oxacillin-
streptococci as well as aerobic gram-negative bacilli, espe-
sensitive, clindamycin-resistant coagulase-positive Staphy-
cially Escherichia coli and Pseudomonas aeruginosa.
lococcus aureus. The blood culture from admission also
Back pain, fever, and headache are the usual complaints,
showed oxacillin-sensitive, clindamycin-resistant coagulase-
although many patients have nonspecific symptoms, which
positive S aureus. The girl continued to receive oxacillin for
can delay diagnosis. The clinical presentation is usually
a total of 6 weeks to treat the spinal epidural abscess,
insidious because the abscess enlarges too slowly to produce
paraspinal musculature abscess, bacteremia, and secondary
the sudden onset of major neurologic deficits. The pre-
bacterial meningitis. Because her inflammatory markers
sentation of spinal epidural abscess can be divided into 4
continued to trend down while she was receiving oxacillin,
phases: back pain at the level of the affected spine (phase 1);
nerve root pain radiating from the affected spinal area
(phase 2); motor weakness, sensory deficit, and bladder
and bowel dysfunction (phase 3); and paralysis (phase 4).
The progression from 1 phase to another can vary from
hours to days, although after stage 3, the progression to ir-
reversible paralysis can be rapid.
The diagnosis of spinal epidural abscess is based on
clinical suspicion, elevated inflammatory markers, and neu-
rologic imaging. MRI is the imaging modality of choice in
cases of suspected spinal epidural abscess because it can
properly show the spinal cord and epidural space. It can
also identify osteomyelitis, adjacent abscesses, soft-tissue
infections, and intramedullary spinal lesions. Diagnosis
Figure 2. In the T1-weighted magnetic resonance imaging, a fluid
collection (red arrow) is seen in the dorsal epidural space with no spinal of spinal epidural abscess can only be confirmed by drain-
canal or foraminal narrowing. There are also fluid collections (white age of the purulent fluid.
arrows) adjacent to the L4 spinous process along with overlying reactive
myositis within the paraspinous musculature. These findings are highly Treatment should not be delayed once spinal epidural
suspicious for abscess centered at the L4 spinous process level with
abscess has been diagnosed because disease progression
extension into the left dorsal posterior epidural space. The abscess tracks
caudally to at least the level of the S2 vertebral level, which is not shown. can be rapid and unpredictable. Treatment consists of two
principles: decreasing the size of the mass and eradicating Lessons for the Clinician
the infecting organism. Broad-spectrum antibiotic treat- • Physicians should be aware of spinal epidural abscess, a
ment should be initiated promptly until the infecting organ- rare but serious diagnosis, to avoid delay in treatment and
ism’s specificity and sensitivity is identified. Due to variable permanent neurologic deficits.
causes, identifying the microorganism through drainage fol- • The clinical presentation of the epidural abscess can be
lowed by long-term intravenous antibiotics is the mainstay for divided into 4 stages: spinal ache, root pain, motor and
treating extra-axial central nervous system infections. Typically, sensory deficits, and lastly paralysis.
treatment should be continued for 3 to 4 weeks but should be • Epidural abscess is managed both surgically and medi-
prolonged if the patient has osteomyelitis. It is important to cally to reduce the size of the abscess and eradicate the
note that data about management in the pediatric population infecting organism.
are limited, and there is no reported consensus.
The key clinical feature in this case was recognizing that Suggested Readings
this patient could have spinal epidural abscess, which is
Darouiche RO. Spinal epidural abscess. N Engl J Med. 2006;355
a serious diagnosis that can progress rapidly. The differen- (19):2012–2020
tial diagnoses include compressive and inflammatory Hawkins M, Bolton M. Pediatric spinal epidural abscess: a 9-year
processes involving the spinal cord such as transverse mye- institutional review and review of the literature. Pediatrics. 2013;132
litis, metastatic tumor, and herniation of an intervertebral (6):e1680–e1685

disc. Other infectious processes that must be excluded are Sáez-Llorens X, Guervera JN. Parameningeal infections. In: Cherry JD,
Demmler-Harrision GJ, Kaplan SL, Hotez PJ, Steinbach WJ, eds.
bacterial meningitis, osteomyelitis, endocarditis, and disc Feigin and Cherry’s Textbook of Pediatric Infectious Disease. 7th ed.
space infection. Philadelphia, PA: Elsevier Saunders; 2014:462–472
Recurrent Lesions on Palms of a 12-year-old
3 Girl

Alexander K.C. Leung, MBBS,* Benjamin Barankin, MD†


*University of Calgary, Pediatrics, Calgary, Alberta, Canada

AUTHOR DISCLOSURE Drs Leung and Toronto Dermatology Centre, Toronto, Ontario, Canada
Barankin have disclosed no financial
relationships relevant to this article. This
commentary does not contain a discussion
PRESENTATION
of an unapproved/investigative use of a
A 12-year-old girl presents with a 1-year history of whitish papules and plaques on
commercial product/device.
her palms after brief exposure to water, such as handwashing or taking a shower
or bath. She is greatly bothered by the appearance and texture of her hands. She
denies having similar lesions on her soles or other cutaneous areas when exposed
to water. There is no family history of similar skin disorders or cystic fibrosis.
The child has atopic dermatitis and asthma. She does not have allergic rhinitis,
abnormal scalp hair, hyperhidrosis, or other skin or systemic disease. Of partic-
ular note, she has no history of chronic lung disease or gastrointestinal disease.
She is not taking any medication and is otherwise healthy.
Physical examination of the palms reveals normal findings. However, follow-
ing immersion of the hands in water at room temperature for 5 minutes,
numerous yellowish-white, nonscaling, smooth-surfaced, flat-topped papules,
some of which have coalesced to form plaques, appear on both palms (Fig).
After drying, the skin normalizes within 30 minutes. The dorsal surfaces of the
hands and other body sites are unaffected with water immersion. The remainder
of the examination is unremarkable. On specific examination, the girl has no
sparse or thin hair.

Figure. White, smooth-surfaced, flat-topped papules and plaques appear on both palms after
immersion in room temperature water for 5 minutes.

The Case Discussion, References, and Suggested Readings appear with the online version
of this article at http://pedsinreview.aappublications.org/content/38/1/48.

48 Pediatrics in Review
DISCUSSION present in 31 of 54 (53.4%) patients with cystic fibrosis, 2
of 23 (8.7%) carriers, and no patients in the control group
Based on the history and physical findings, clinicians diag-
after immersion of the hands in room temperature tap water
nose aquagenic palmar keratoderma. Genetic analysis did
for 3 minutes. (1) In another study, aquagenic palmar kerato-
not find any mutation in the cystic fibrosis transmembrane
derma was present in 11 of 27 (41%) patients with cystic fibrosis
conductance regulator (CFTR) gene.
after immersion of the palms in water for 2 to 3 minutes. (2)
Aquagenic palmar keratoderma is characterized by the
Except for patients with cystic fibrosis, the occurrence
transient appearance of translucent white papules or
is usually sporadic. The condition has a predilection for
plaques on the palms shortly after brief exposure to water
adolescents and females. In most cases, the cause is not
that disappear after drying within minutes to 1 hour. The
known. Aquagenic palmar keratoderma occurs in otherwise
primary differential diagnosis is hereditary papulotranslu-
healthy individuals and more commonly in patients with
cent acrokeratoderma. The latter condition has an autoso-
atopy, hyperhidrosis, and marasmus as well as patients taking
mal dominant mode of inheritance and presents with
angiotensin-converting enzyme inhibitors (eg, enalapril, rami-
bilateral, symmetric, asymptomatic, yellow-white translu-
pril), angiotensin-receptor blockers (eg, valsartan, losartan),
cent papules, coalescing into plaques. It appears soon after
selective cyclo-oxygenase inhibitors (eg, celecoxib, rofecoxib),
puberty and persists throughout life. The lesions occur
nonsteroidal anti-inflammatory drugs (eg, acetylsalicylic
primarily along the margins and pressure points of the
acid), and certain antibiotics (eg, aminoglycosides).
hands and feet. Although papules and plaques appear without
The exact pathogenesis is not known. An abnormality
water exposure, typically the papules gain increased promi-
of the sweat duct with dilation of the eccrine ostia, hyper-
nence with water exposure. Hereditary papulotranslucent
keratosis, barrier dysfunction of the stratum corneum, and
acrokeratoderma is associated with fine-textured scalp hair
defective chloride channels has been implicated.
and an atopic diathesis. Aquagenic palmar keratoderma also
Histologic examination of a classic lesion shows hyper-
should be differentiated from punctate keratosis of palmar
keratosis of the stratum corneum, dilation of intraepidermal
creases, a condition that primarily affects African American
eccrine sweat ducts, dilation of eccrine sweat duct ostia
patients in which the lesions are confined to palmar creases.
(acrosyringium), and hyperplasia of the eccrine sweat glands.

The Condition Diagnosis


Clinically, aquagenic palmar keratoderma is characterized The diagnosis is primarily clinical, based on the character-
by translucent whitish papules coalescing into plaques istic physical findings after immersion of the hands in water.
and wrinkling of the edematous palms after contact with The “hand-in-the bucket” sign is pathognomonic. Dermo-
water. The skin changes usually resolve spontaneously scopy of the lesion typically shows marked enlargement of
within minutes to 1 hour after the affected area has been the sweat duct puncta compared with unaffected palmar
dried. The diagnostic “hand-in-the bucket” sign refers to the areas. Histologic investigations are usually not necessary.
fact that the skin changes are not or barely visible until the Because aquagenic palmar keratoderma can occur both in
hand is submerged in water. Warm or hot water is more patients with cystic fibrosis and in mutation carriers, affected
rapidly effective than cold water in inducing the pheno- patients should undergo sweat chloride testing and CFTR
menon. Prolongation of water exposure may intensify the mutation analysis.
lesions. Some patients react only to tap water but not salt-
water; others react to both. The condition is typically bilateral Management
and symmetric. This keratoderma is usually asymptomatic, If present, the underlying cause should be removed if
although some patients complain of pruritus, tingling, burn- possible. For example, offending medications should be
ing, tightening sensation, or pain in the affected area. discontinued. Because the condition is benign and usually
A less common variant presents with persistent translucent asymptomatic, treatment may not be necessary. Despite not
whitish papules and plaques, edema, and wrinkling of the knowing the exact pathogenesis, the cosmetic appearance
palms that worsens after submersion of the hands in water. and texture can be improved. For patients who prefer to have
Aquagenic palmar keratoderma is an uncommon con- treatment, topical 15% to 20% aluminum chloride hexahy-
dition, except in patients with homozygous and, less com- drate in anhydrous ethyl alcohol, 5% to 20% salicylic acid
monly, heterozygous cystic fibrosis. We suspect that the ointment, 10% urea cream or ointment, 12% ammonium
condition is much more common than is generally ap- lactate cream, petroleum-based barrier creams, and 3%
preciated. In 1 study, aquagenic palmar keratoderma was formalin in anhydrous ethyl alcohol can be used alone or
in combination. Based on the fact that an abnormality of the • Affected patients should undergo sweat chloride testing
sweat duct with dilation of the stratum corneum has been and CFTR mutation analysis.
implicated in the pathogenesis, dermal botulinum toxin
injections and iontophoresis should be considered for cases
not responding to topical treatment, especially in those
References
with associated hyperhidrosis. 1. Chinazzo C, De Alessandri A, Menoni S, Romanisio G, Rebora A,
Rongioletti F. Aquagenic wrinkling of the palms and cystic fibrosis:
This patient was sufficiently bothered by the cosmetic
an Italian study with controls and genotype-phenotype correlations.
appearance and texture of her hands that she was embar- Dermatology. 2014;228(1):60–65
rassed to shake or show her hands. Accordingly, she was 2. Garçon-Michel N, Roguedas-Contios AM, Rault G, et al. Frequency
treated with nightly applications of 15% aluminum chloride of aquagenic palmoplantar keratoderma in cystic fibrosis: a new
sign of cystic fibrosis? Br J Dermatol. 2010;163(1):162–166
hexahydrate with a hydroalcoholic salicylic acid gel base,
which was prescribed based on previous success with this
agent by the authors and a previous literature search. She Suggested Readings
reported satisfactory control with this treatment. Dermal Ertürk-Özdemir E, Özcan D, Seҫkin D. Acquired aquagenic syringeal
botulinum toxin injections were offered and may be con- acrokeratoderma: a case series of 10 patients. Australas J Dermatol.
2013;56(2):e43-e45. doi: 10.111/ajd.12122
sidered in the future.
Ibusuki C, Oka M, Fukunaga A, Kunisada M, Nishigori C. Unilateral
aquagenic wrinkling of the palms with a peculiar clinical course.
Lessons for the Clinician Eur J Dermatol. 2012;22(5):679–680
• Aquagenic palmar keratoderma is characterized by the Kent JB, Statuta SM, Greer KE, MacKnight JM. Watersport hands. Sports
transient appearance of translucent white papules or Health. 2014;6(4):360–362
plaques on the palms shortly after brief exposure to water. Rongioletti F, Tomasini C, Crovato F, Marchesi L. Aquagenic (pseudo)
• The skin changes usually resolve spontaneously within keratoderma: a clinical series with new pathological insights.
Br J Dermatol. 2012;167(3):575–582
minutes to 1 hour after the affected area has been dried.
Sezer E, Erkek E, Duman D, Sahin S, Cetin E. Dermatoscopy as an
• It is an uncommon condition, except in patients with homo- adjunctive diagnostic tool in aquagenic syringeal acrokeratoderma.
zygous and, less commonly, heterozygous cystic fibrosis. Dermatology. 2012;225(2):97–99
Worsening Abdominal Distention in a
4 2-year-old Boy

Sean Indra, MD,* Shazia Maqbool, MD*


*Children’s Hospital of Michigan, Detroit, MI

PRESENTATION

AUTHOR DISCLOSURE Drs Indra and A previously healthy 2-year-old boy presents to the emergency department with
Maqbool have disclosed no financial a history of abdominal distention that began 1 week ago and has since been
relationships relevant to this article. This
commentary does not contain a discussion of
worsening. He is toilet-trained and his family mentions that he has been having
an unapproved/investigative use of a intermittently decreased urination for the past month. Some days he only urinates
commercial product/device. once or twice. He has had no fever, emesis, joint pain, anorexia, weight loss, or
decreased activity. The boy’s medical history and family history are unremarkable.
On physical examination, he has normal vital signs. He appears well and has
normal physical examination findings, with the exception of his gastrointestinal
evaluation. His abdomen is soft but with prominent swelling in the right upper
quadrant and epigastric regions. In the area of swelling, there is a smooth, nontender,
palpable mass without overlying erythema. A liver edge is not palpable because the
mass is overlying that area, but a spleen tip is noted at the left costal margin.
Initial laboratory results include:
• White blood cell count, 14,000/mL (14.0  109/L)
• Absolute neutrophil count, 5,100/mL (5.1  109/L)
• Absolute lymphocyte count, 7,900/mL (7.9  109/L)
• Blood smear revealing occasional anisocytosis
• Hemoglobin, 11.1 g/dL (111 g/L)
• Platelet count, 329  103/mL (329  109/L)
• Aspartate aminotransferase, 47 U/L (0.78 mkat/L) (reference range 0-37 U/L
[0-0.62 mkat/L])
• Alanine aminotransferase, 31 U/L (0.52 mkat/L) (reference range 0-78 U/L
[0-1.30 mkat/L])
• Lactate dehydrogenase, 727 U/L (reference range 100-240 U/L)
• Uric acid, 4.2 mg/dL (249.84 mmol/L) (reference range 3.5-7.2 mg/dL
[208.20-428.29 mmol/L])
• Urea nitrogen, 18 mg/dL (6.3 mmol/L)
• Creatinine, 0.28 mg/dL (24.8 mmol/L)
The remainder of the basic electrolyte panel, the coagulation profile, and
urinalysis are interpreted as normal. The patient is hospitalized and additional
evaluation leads to the diagnosis.

The Case Discussion and Suggested Readings appear with the online version of this
article at http://pedsinreview.aappublications.org/content/38/1/49.

Vol. 38 No. 1 JANUARY 2017 49


DISCUSSION cartilage, and adipose tissue. Rhabdomyosarcoma (RMS) is
the most common soft-tissue sarcoma, accounting for 3%
Abdominal ultrasonography revealed a large heterogeneous
of all pediatric tumors. Given its origin in the embryonal
mass suspected to originate from the liver. Abdominal
mesenchyme, this disease has the potential to arise anywhere
magnetic resonance imaging (MRI) further defined the
in the body. Most cases are sporadic. Several environmental
mass to be largely involving the left hepatic lobe (Fig).
factors have been implicated with an increased risk for RMS,
Additional laboratory results included ferritin measuring
including cigarette smoking, advanced maternal age, radia-
38 ng/mL (85.39 pmol/Lng/mL) (reference range 22-322
tion exposure in utero, and antibiotic use. Inherited familial
ng/mL [49.43-723.53 pmol/Lng/mL]) and negative urine
syndromes, such as neurofibromatosis and Li-Fraumeni syn-
vanillylmandelic acid and homovanillic acid. Biopsy of the
drome, also have some association with RMS.
mass established the diagnosis of an embryonal rhabdo-
The presentation depends on the site of the tumor and
myosarcoma. The patient underwent a gross total resection
whether it involves any metastasis. The head and neck region
of the mass and had multiple lymph node biopsies and a
is the most common site, accounting for about one-third of
bone marrow biopsy that were negative. He received 22
weeks of chemotherapy with vincristine, dactinomycin, and total cases, followed by the genitourinary (GU) tract (25%)
cyclophosphamide. He has been off therapy for 12 months and extremities (20%). Sites that are involved within the
and is doing well. head and neck include the orbit and parameningeal sites
(middle ear, nasopharynx, paranasal sinuses) and can present
Differential Diagnosis with proptosis or obstruction of the parameningeal region.
A palpable abdominal mass is 1 of the more common GU tract tumors can involve the bladder, prostate, or vagina
findings of malignant solid tumors in children. Neuro- and present with hematuria, outflow obstruction, or urinary
blastoma and Wilms tumor are the 2 most common intra- frequency. Extremity involvement typically presents with
abdominal tumors. For children younger than age 2 years, obvious swelling and has a higher likelihood of spreading
neuroblastoma is the most common extracranial malignant to regional lymph nodes.
tumor. Wilms tumor is the most common renal malignancy Initial evaluation involves plain radiographs of the area
and two-thirds of cases are diagnosed before age 5 years. surrounding the mass, followed by computed tomography
Leukemia, lymphoma, hepatoblastoma, and soft-tissue (CT) scan and/or MRI. Laboratory evaluation includes com-
sarcomas are more prevalent in older children. plete blood cell count with smear, electrolytes, liver function
tests, uric acid, and calcium. Definitive diagnosis requires
The Condition biopsy and special staining performed by a pediatric pathol-
Sarcoma is a relatively rare malignant tumor that arises ogist. If there is suspicion of metastasis based on the site of
from embryonic mesenchymal cells and has the ability to the primary tumor, further imaging should be performed
differentiate into striated skeletal and smooth muscle, bone, with CT scan or radionuclide bone scan.

Figure. Abdominal magnetic resonance imaging shows a mass largely involving the left lobe of the liver.
Management Lessons for the Clinician
Current treatment is based on the recommendations of • Rhabdomyosarcoma should be in the differential diagnosis
the Intergroup Rhabdomyosarcoma Study Group (IRSG), for children with enlarging masses anywhere in the body.
which began in 1972. The most current protocol is the IRSG- • Plain films are the appropriate initial imaging study, fol-
V, which achieves a cure rate of 70% in children with lowed by computed tomography scan and/or magnetic
localized disease. Complete surgical excision is recom- resonance imaging.
mended for local disease if it is possible to achieve appro- • Biopsy is required to obtain the definitive diagnosis.
priate functional and cosmetic results. Tumors that are in • Prompt therapy should be initiated with a combination
areas such as the orbit, vagina, or bladder may require of surgical excision, chemotherapy, and radiation therapy,
induction chemotherapy to decrease the tumor burden to depending on the tumor size and affected area.
allow for acceptable excision. Following surgical excision,
radiation therapy enhances the treatment of residual micro-
Suggested Readings
scopic disease. All patients with RMS should subsequently McCarville MB, Spunt SL, Pappo AS. Rhabdomyosarcoma in pediatric
patients: the good, the bad, and the unusual. AJR Am J Roentgenol.
undergo chemotherapy with the gold standard regimen of
2001;176(6):1563–1569
vincristine, dactinomycin, and cyclophosphamide. Patients
Perez EA, Kassira N, Cheung MC, Koniaris LG, Neville HL, Sola JE.
who relapse after the initial treatment have a poor prognos- Rhabdomyosarcoma in children: a SEER population based study.
tic outcome. J Surg Res. 2011;170(2):e243–e251
Fatigue, Polydipsia, and Nocturia in a 17-year-
5 old Boy

S. Amara Ogbonnaya, MD*


*Internal Medicine-Pediatrics, University of California-Los Angeles Medical Center, Santa Monica, CA.

PRESENTATION

A 17-year-old boy presents with a 4-month history of increasing fatigue, increased


AUTHOR DISCLOSURE Dr Ogbonnaya has
disclosed no financial relationships relevant to thirst, and nocturia 3 times per night. He complains of painful masses on his
this article. This commentary does not contain clavicle and scalp that have gradually increased in size for 1 year. In addition, he
a discussion of an unapproved/investigative
has been unsuccessfully treated with topical antifungal medications for a rash on
use of a commercial product/device.
his lower back. He denies sexual activity and discloses a loss of erections for
several months. He has no pertinent findings on his past medical history and is
not taking medications.
On physical examination, the adolescent’s height, weight, and body mass
index are at the 25th, 90th, and greater than the 95th percentile, respectively.
He is afebrile and his blood pressure is 100/70 mm Hg, heart rate is 83 beats/
min, and respiratory rate is 14 breaths/min. Tender, nonerythematous,
poorly circumscribed swellings are visible on the proximal left clavicle and
right parietal areas, measuring 4 and 2 cm, respectively. The genital exam-
ination reveals normal findings with Sexual Maturity Rating 5 pubic hair.
He has a hyperpigmented 10-cm patch with ulceration in the gluteal cleft
(Fig 1).

Figure 1. Hyperpigmented lesion with ulceration in the gluteal cleft.

50 Pediatrics in Review
Laboratory tests document the following:
• White blood cell count 3,700/mL (3.7  109/L), with
77.6% neutrophils, 9.7% lymphocytes, 8.3% mono-
cytes, 3.3% eosinophils, and 1.1% basophils
• Hemoglobin 10.6 g/dL (106 g/L)
• Mean corpuscular volume 91.8 mm3 (91.8 fL)
• Red cell distribution width 16%
• Platelet count 128  103/mL (128  109/L)
• Sodium 145 mEq/L (145 mmol/L)
• Potassium 4.3 mEq/L (4.3 mmol/L)
• Chloride 105 mEq/L (105 mmol/L)
• Carbon dioxide 29 mEq/L (29 mmol/L)
• Blood urea nitrogen 5 mg/dL (1.78 mmol/L)
• Creatinine 0.67 mg/dL (59.23 mmol/L)
• Calcium 8.6 mg/dL (2.15 mmol/L)
Computed tomography scan shows lytic lesions of the
skull (Fig 2) and left clavicle with associated soft-tissue
swelling.
Because of the patient’s substantial fatigue and obesity, Figure 2. Computed tomography scan showing lytic lesions of the skull.
thyroid studies are ordered and show thyrotropin of 0.79
mIU/L (normal range, 0.50-3.50 mIU/L) and free thyroxine
of 0.85 ng/dL (10.94 pmol/L) (normal range, 0.93-1.70
ng/dL [11.97-21.88 pmol/L]). The following day, tests are The Case Discussion and Suggested Readings appear with the
ordered to evaluate the patient’s loss of erections. Biopsies of online version of this article at http://pedsinreview.aappublica-
the left clavicle and skin lesion confirm the diagnosis. tions.org/content/38/1/50.

Vol. 38 No. 1 JANUARY 2017 51


DISCUSSION Clinical Presentation
LCH is classified as a single-system or multisystem disease.
The biopsy results are consistent with Langerhans cell
In children, the disease most frequently presents as an
histiocytosis (LCH). Additional laboratory results include
incidentally identified solitary lytic lesion of the skull. How-
total testosterone measuring less than 12 ng/dL (0.42 nmol/L)
ever, infants often present with multisystem disease involv-
(normal range, 249-835 ng/dL [8.64-28.97 nmol/L]), luteinizing
ing the bone marrow, liver, spleen, and skin, which requires
hormone of less than 0.1 mIU/mL (0.1 IU/L) (normal range,
aggressive treatment.
1.7-8.6 mIU/mL [1.7-8.6 IU/L]), and follicle-stimulating
The skin lesions can vary widely in appearance from
hormone of 0.6 mIU/mL (0.6 IU/L) (normal range,
papular to nodular. Most often children have a maculopap-
1.5-12.4 mIU/mL [1.5-12.4 IU/L]) are consistent with con-
ular rash that is mistaken for eczema. Skin findings suspi-
comitant panhypopituitarism. Subsequently, magnetic
resonance imaging, obtained because of the abnormal cious for atopic dermatitis or seborrheic dermatitis that are
laboratory results, shows a 1-cm enhancing mass of the unresponsive to standard therapies should prompt further
pituitary stalk. evaluation, including biopsy and referral to dermatology.
Adolescents and adults may have a rash with or without
ulceration in intertriginous areas that is often mistaken for
The Condition
a fungal infection.
LCH, previously called histiocytosis X or Hand-Schüller-
The skeletal LCH lesions are typically lytic and may have
Christian disease, is a disorder of histiocytes that can have
a surrounding soft-tissue mass. The skull is most commonly
highly variable presentations. The cause is unknown, but
the disease is marked by clonal proliferation of immature involved, followed by the pelvis and femurs. Lytic lesions
Langerhans cells. The prevalence is approximately 1 in compromise bone integrity, increasing the risk of fractures.
100,000 people, with 1,200 new cases diagnosed in the Patients with osseous involvement of the face and skull
United States annually. It occurs more commonly in chil- bones are more likely to have disease in the central nervous
dren younger than age 3 years but can affect any age group. system (CNS).
LCH can affect all organs, which can make the diagnosis For unknown reasons, LCH has a predilection for the
challenging. The areas most frequently involved are bone pituitary gland. Diabetes insipidus (DI) is the most common
(80% of patients), skin (33% of patients), and the pituitary manifestation of LCH of the pituitary gland. Anterior pitu-
gland (25% of patients). Because involvement of the bone itary dysfunction can occur and usually results in growth
marrow, liver, and spleen portend a poor prognosis, these hormone deficiency. Less commonly, patients may present
are considered “risk organs.” with panhypopituitarism, as in this case. Pituitary involve-
ment in LCH is noteworthy because it can develop years
Differential Diagnosis before other manifestations of LCH, can occur simulta-
Because of the myriad presentations, the differential diag- neously with other signs, or can develop years after the
nosis for LCH is broad and depends greatly on organ sys- initial diagnosis. Thus, patients with LCH or a history of
tems involved. When the disease is isolated to the skeletal LCH should have growth and pubertal progression as-
system, the differential diagnosis may include osteomyelitis, sessed regularly to identify development of endocrinopa-
primary bone malignancies, benign bone cysts, or metastatic thies. Clinicians should also consider LCH in patients
cancer. For a patient presenting with lymphadenopathy and presenting with pituitary disease without other symp-
splenomegaly and/or cytopenias, lymphoma or leukemia toms. Once present, pituitary dysfunction persists despite
may be considered initially. treatment. Extrapituitary CNS disease is less common and
can lead to neurodegenerative manifestations, including
Diagnosis learning difficulties, memory problems, and cerebellar
The diagnosis is made by tissue biopsy of bone or skin ataxia.
lesions, as in this patient. Positive immunohistochemical In addition to endocrinologic and CNS complications,
staining for the proteins CD1a and CD207, also called other long-term complications may occur, depending on the
Langerin, confirm the diagnosis. Electron microscopy organs involved. Patients with lesions of the temporal,
shows intracellular Birbeck granules, which are also confir- vertebral, or jawbones may develop hearing loss, scoliosis,
matory. No laboratory findings are specific for the diagnosis or dental deformities, respectively. Patients with LCH are at
of LCH, but results may aid in identification of involved increased risk for developing leukemia and lymphoma. In
organs. addition, secondary malignancies may occur after treatment.
Close follow-up evaluation is needed to monitor for these polydipsia. He was started on testosterone, desmopressin,
complications. hydrocortisone, and levothyroxine for panhypopituitarism.
He is being treated with cytarabine for LCH. He reports
Treatment improved energy, resolution of nocturia, and a decrease in
Treatment varies substantially based on the extent of dis- the size of the clavicular and scalp lesions since starting
ease. Rarely, when only skin is involved, lesions may spon- therapy.
taneously regress. Patients with single-system disease and
a solitary bone lesion may be treated with curettage only.
Lessons for the Clinician
However, systemic chemotherapy is indicated for patients
• Langerhans cell histiocytosis (LCH) is a difficult disease to
with single-system disease and diffuse lesions. Generally,
diagnose because of its variable presentation, particularly
vinblastine with prednisolone is the standard initial treat-
in multisystem involvement.
ment. Patients with single-system disease usually have good
• LCH should be considered in a patient presenting with
prognoses.
bone and/or skin lesions and symptoms concerning for
Multisystem disease always requires systemic therapy.
diabetes insipidus.
Involvement of risk organs portends a high mortality and
• Biopsies should be performed of any suspicious lesions
morbidity, and patients are more likely to have reactivation
or lesions that do not respond as expected to therapy.
of disease after treatment. LCH of the CNS is treated with
• Patients with LCH should be monitored for the devel-
systemic therapies that cross the blood brain barrier, includ-
opment of endocrinologic abnormalities.
ing cytarabine or cladribine. However, the efficacy in pre-
venting neurodegenerative complications is unclear.
Suggested Readings
Patient Course Haupt R, Minkov M, Astigarraga I, et al; Euro Histio Network.
Langerhans cell histiocytosis (LCH): guidelines for diagnosis,
Our patient was diagnosed with multisystem LCH with
clinical work-up, and treatment for patients till the age of 18 years.
pituitary involvement and was admitted for oncologic and Pediatr Blood Cancer. 2013;60(2):175–184
endocrinologic evaluation and treatment. Further evalua- Weitzman S, Egeler RM. Langerhans cell histiocytosis: update for the
tion also revealed DI, which explained his nocturia and pediatrician. Curr Opin Pediatr. 2008;20(1):23–29
6 Episodic Stiffness in a 30-month-old Girl

Lisa Mitchell, DO,* William Adams, MD,* Francois Aspesberro, MD†


*Madigan Army Medical Center, Tacoma WA.

Seattle Children’s Hospital, Seattle, WA.

PRESENTATION

A previously healthy 30-month-old girl presents a third time to the emergency


AUTHOR DISCLOSURE Drs Mitchell, Adams,
and Aspesberro have disclosed no financial department (ED) with a 3-day history of cough, increased fussiness, neck stiffness,
relationships relevant to this article. This clumsiness, and unwillingness to open her mouth. She continues to drink well
commentary does not contain a discussion of
but refuses to eat. She has had several brief episodes of screaming followed by
an unapproved/investigative use of a
commercial product/device. paroxysmal stiffness, during which she has difficulty drawing breath and has
roving eye movements, but she seems awake and alert. A normal breathing
pattern resumes quickly after the episodes resolve. Episodes have been worsening
in frequency, duration, and apparent intensity of pain, and today she had an
episode at home that lasted 60 seconds and was accompanied by perioral
cyanosis. She takes no medications, has had no immunizations, and has no
travel history or sick contacts, and her parents deny known ingestions or trauma.
Two days ago, the patient developed a cough, listlessness, and vocal raspiness.
Her parents noticed that she was drinking liquids well but had difficulty chew-
ing and opening her mouth, and her neck seemed stiff. On the second day of ill-
ness, she presented to another ED after 2 brief episodes of stiffening during which
she had noisy, shallow breathing, followed by a gasp, and then resumption of
a normal breathing pattern. Apart from erythematous tympanic membranes,
her examination results were normal, and she was diagnosed with acute otitis
media and sent home with ibuprofen and azithromycin. Early the next morning,
she had a more prolonged episode in which she cried out in pain, stiffened, and
then tried to breathe but seemed unable to do so. This lasted longer than before
and was accompanied by cyanosis but no loss of consciousness. She, therefore,
returned to the same ED on the third day of illness, where she was observed
for 2 hours with no return of symptoms and was sent home with analgesic otic
drops.
After discharge, she had multiple episodes of crying, stiffening, and airway
obstruction that progressively worsened in intensity, pain, and duration as well as
periods of inconsolable crying and screaming, so her parents brought her to our
institution for further evaluation. In the ED, her vital signs and tympanic membranes
are normal. Physical examination reveals an alert, afebrile, irritable child with con-
gestion, hoarse voice, and neck stiffness who is holding her head tilted right with
limited range of motion in flexion. There is no tenderness or swelling over her mas-
toids and no drooling. She cannot open her mouth wide and actively resists attempts to
pry it open with a tongue depressor. Her abdomen is soft and lungs are clear to
auscultation bilaterally. There is no evidence of phlegmon, abscess, trauma, or airway

52 Pediatrics in Review
compromise on soft-tissue lateral neck films, chest radiogra- and urine; and viral polymerase chain reaction assays on the
phy, or computed tomography scans of her head and neck. CSF for Ebstein-Barr virus, cytomegalovirus, varicella, her-
She is hospitalized initially in the general medicine ser- pes simplex, and enterovirus, all of which are negative.
vice for observation because of concern for seizures but is Ionized calcium and serum electrolytes, CSF analysis, and
transferred to the pediatric intensive care unit after another C-reactive protein results are normal.
stiffening and obstructing episode that resolves after 2 doses She has had no fever or mental status changes. Clinicians
of lorazepam. In the unit, she is noted to be hyperreflexive note that her jaw did not relax, even under propofol sedation
throughout, with diffuse clonus in both ankles, upgoing for lumbar puncture. Despite extensive laboratory and ra-
Babinski sign, and intermittent spasms in the lower ex- diologic evaluation, her diagnosis is established based on
tremities bilaterally. She is also observed to arch backwards her clinical presentation.
intermittently. Her oxygen saturations drop to 70% because
of spasms and airway obstructions and she is intubated. The Case Discussion and Suggested Readings appear with the
Laboratory evaluation includes an extended urine toxi- online version of this article at http://pedsinreview.aappublica-
cology screen; cultures of cerebrospinal fluid (CSF), blood, tions.org/content/38/1/52.

Vol. 38 No. 1 JANUARY 2017 53


DISCUSSION in more developed nations due to widespread vaccination.
The number of cases in the United States has declined 95%
Tetanus was diagnosed on the basis of her classic constella-
since it first became a reportable disease in 1947, and the
tion of symptoms coupled with a lack of immunizations and
Centers for Disease Control and Prevention reported only
absence of historical or laboratory data supporting other diag-
20 cases in 2014.
noses. The entry point for inoculation was never identified.
The patient received intramuscular human tetanus
Diagnosis
immune globulin (3,000 units) and a 14-day course of
The diagnosis of tetanus is clinical, but several other causes
metronidazole (14 mg/kg per day divided every 6 hours).
of symptoms should be considered. The differential diag-
Tracheostomy was performed early in her course to ensure
nosis for such symptoms as tetany, trismus, and muscular
airway patency during spasms. She initially showed signs of
rigidity includes strychnine poisoning (rat poison inges-
autonomic instability, including sinus tachycardia, widened
tion), encephalopathy, meningitis, stiff person syndrome (a
pulse pressure, sweating, and a fluctuating body tempera-
rare autoimmune disorder characterized by progressive trun-
ture, so a magnesium sulfate drip was started with a target
cal and proximal limb rigidity and stiffness that worsens with
serum level of 4 mg/dL. She also required norepinephrine
painful spasms), drug ingestion (leading to drug-induced
for arterial hypotension. Her sedation package included
dystonia or malignant neuroleptic syndrome), tetanic spasms,
midazolam, morphine, and vecuronium drips initially, with
and retropharyngeal abscess or trismus from dental infec-
dexmedetomidine hydrochloride added later. She was main-
tion. There is no definitive laboratory test to diagnose
tained in as low a stimulation environment as possible to
tetanus, and the bacterium can only be isolated from an
avoid provoking further spasms. She was weaned off ven-
infected wound in a minority of cases. The serum tetanus
tilation 15 days after intubation, and the tracheostomy was
immunoglobulin (Ig)G antibody assay is approved only to
capped 13 days after it was placed. She was transferred to the
confirm vaccine response in immunized individuals and was
general medicine service for continued rehabilitation.
unlikely to be helpful in this case because the initial response
is IgM-mediated. There is currently no tetanus IgM assay
Condition available. In addition, the IgG response to primary tetanus
Tetanus is caused by Clostridium tetani, a ubiquitous obligate infection is weak, which explains why infection with tetanus
anaerobe found in soil and the mammalian gastrointestinal does not confer adequate immunity and infected individuals
tract. Humans become inoculated through tissue damage, still require vaccination after infection resolution.
classically through penetrating injury (eg, stepping on a
rusty nail). In most of those infected, an antecedent injury is Management
identifiable, but in a small percentage of cases the diagnosis Tetanus toxin binding within the central nervous system is
must be based solely on clinical presentation. irreversible, so treatment is primarily supportive and tar-
After entering the body, the spore transforms into a geted at preventing airway compromise and dysautonomia
vegetative state and produces tetanospasmin. This neuro- until new axonal nerve terminals can grow. Patients require
toxin is taken up from peripheral nerve terminals into the intensive support for several weeks before complete recov-
central nervous system and binds to gangliosides at the ery can be achieved. If a wound can be found, treatment
presynaptic inhibitory motor nerve endings. Here it blocks starts with wound debridement. Patients should receive
the release of inhibitory neurotransmitters; the classic gen- metronidazole for 10 to 14 days to stop bacterial proliferation
eralized muscular spasms of tetanus result from unchecked at the wound site. Intramuscular injection of human tetanus
motor nerve impulses. This phenomenon also impairs immune globulin (3,000-6,000 units), preferably infiltrated
neural control over the adrenal cortex, leading to a hyper- near the wound, is recommended to neutralize any as yet
sympathetic state due to catecholamine release, with the unbound toxin. We initially used a magnesium sulfate drip
potential for widespread autonomic instability. The classic for autonomic instability in our patient, as discussed in
presentation includes trismus (in >50% of cases), stiff neck, some international studies. Other options include cloni-
opisthotonus, risus sardonicus, a boardlike rigid abdomen, dine, morphine, and labetalol. Muscle spasms have been
periods of upper airway obstruction due to thoracic muscle successfully managed with benzodiazepine drips, intra-
contraction or glottic/pharyngeal contractions, and dyspha- thecal baclofen, dantrolene, and neuromuscular blocking
gia. Untreated patients frequently die of sequelae of airway agents such as vecuronium. Control of stimuli such as light
obstruction and dysautonomia. Although common in many and noise around these patients is helpful to avoid provoking
developing parts of the world, tetanus is now extremely rare spasms.
Lessons for the Clinician diagnosis is crucial because most patients recover with
• Tetanus should be considered in unimmunized patients with access to modern supportive care.
intermittent airway obstruction, trismus, and/or muscle ri-
gidity, even without antecedent history or evidence of injury.
The vaccine is highly effective, so the likelihood of infection in Suggested Readings
a properly immunized individual is extremely small. American Academy of Pediatrics. Tetanus (lockjaw). In: Kimberlin DW,
• Tetanus occurs frequently in the developing world but Long SS, Brady MT, Jackson MA, eds. Red Book: 2015 Report of the
Committee on Infectious Diseases. 30th ed. Elk Grove Village, IL:
may still be seen in underimmunized populations in
American Academy of Pediatrics; 2015:773–778
more developed nations. Because few clinicians in the
Govindaraj GM, Riyaz A. Current practice in the management of
more developed world have ever seen this disease, the tetanus. Crit Care. 2014;18(3):145
diagnosis may not be considered until clinical manifes- Rodrigo C, Fernando D, Rajapakse S. Pharmacological management of
tations are obvious, thus delaying treatment. Correct tetanus: an evidence-based review. Crit Care. 2014;18(2):217
Brief
in

Medication Reconciliation
Jenna Merandi, PharmD, MS,* Matthew Sapko, PharmD, MS,* Charline Catt, RN, MS,* Jeffrey M Hoffman, MD*
*Nationwide Children’s Hospital, Columbus, OH

AUTHOR DISCLOSURE Drs Merandi, Sapko, A young boy presents to the emergency department (ED) for a leg laceration
and Hoffman and Ms Catt have disclosed no
suffered during a skateboarding accident. The fast-paced ED triages the patient,
financial relationships relevant to this article.
This commentary does not contain a sutures the wound, and discharges him. Almost immediately, the boy returns to the
discussion of an unapproved/investigative ED after having a seizure in the parking garage and sustaining a head injury that
use of a commercial product/device.
requires an admission. Why? It turns out that his skateboarding accident was related
to adherence issues with his antiepileptic medication. Unfortunately, his complete
medication history was not available to the ED staff. This type of event happens all
too frequently across our health systems, but it is one that we must be committed
to eliminating through improvements in our medication reconciliation process.
Medication reconciliation is a complex, continuous process of creating and
maintaining an accurate list of new and current medications for a patient. The
ultimate goal of the medication reconciliation process is to prevent adverse drug
events, provide education to patients and caregivers, and prevent hospital admis-
sions or readmissions.
According to Joint Commission guidelines, medication reconciliation in the
hospital setting must occur at admission, each transition of care, and discharge.
Thus, all members of an interdisciplinary team must participate in the medica-
tion reconciliation process to achieve success. It is essential that documentation
is correct and communication is clear between the health care team and patients
or caregivers during these transitions. Gathering information such as medica-
tion name, dosage form, route, and frequency and evaluating for appropriate-
ness of the therapy are all components of the medication reconciliation process.
Nurses, physicians, pharmacists, and other allied health care personnel all play
National Patient Safety Goals Effective
January 1, 2015. Oak Brook, IL: Hospital integral roles in keeping patients safe and preventing medication errors. A com-
Accreditation Program. The Joint Commission; parable process is appropriate in all health care environments.
2015. Available at: http://www.jointcommission. As part of every health care system’s goal to eliminate all preventable patient
org/assets/1/6/2015_NPSG_HAP.pdf. Accessed
January 27, 2015 harm, medication reconciliation should be targeted as a focus area for improve-
ment. To achieve that goal, many interventions can be implemented related to medi-
Establishment and Evaluation of Pharmacist-
Managed Admission Medication History and
cation reconciliation, including changes to the electronic medical record (EMR),
Reconciliation Process for Pediatric Patients. increased involvement from pharmacists, and improved patient instructions.
Provine AD, Simmons EM, Bhagat PH. J Pediatr The EMR is one of the first areas of focus that can improve medication
Pharmacol Ther. 2014;19(2):98–102
reconciliation. To standardize the medication reconciliation process, a workflow
Value of the Student Pharmacist to can be created that allows the clinician to view all medications at once, both
Experiential Practice Sites: A Review of the
recently ordered and those used long-term. Those that have not been reviewed
Literature. Mersfelder TL, Bouthiller MJ. Ann
Pharmacother. 2012;46(4):541–548 should be highlighted for increased visibility, and only after all medications have
been evaluated should clinicians be permitted to move onto the next step in the
How to Implement EHRs: Step 5: Achieve
Meaningful Use Stage 2: Medication medical encounter process.
Reconciliation. HealthIT.gov.2015. Available Other visual indicators can serve as a safety net, such as a red banner that re-
at: https://www.healthit.gov/providers- minds clinicians that 1 or more medications still need reconciliation; the banner
professionals/achieve-meaningful-use/core-
measures-2/medication-reconciliation. turns green only after review, helping guarantee that all medications have been
Accessed October 14, 2016 reconciled. Clinicians should then be instructed to verify all medications to be

54 Pediatrics in Review
prescribed, check for duplicates, and preview the patient prescribed medicines as well as to review the medication
instructions to ensure that the full list of medications, reconciliation details contained on the discharge instructions.
including their instructions for use, appears correctly. Another important role for pharmacists (or student phar-
As part of the reconciliation process, random audits can be macists, if available) is to complete medication histories for
conducted on discharge instructions at the end of a hospi- patients admitted from the ED. They can review current and
tal admission or any medical encounter. Typical problems past medications to ensure that correct information is present
that may be identified include duplicate medications and in each patient’s profile and enter notes and suggestions for the
conflicting usage information, often resulting from medica- inpatient care team about perceived issues and follow-up needs.
tion information being entered into the free text portions Of course, most of a patient’s medication behaviors occur
of discharge instructions. By avoiding free text fields and at home. Leveraging relationships within the community
documenting medication instructions only in the medication for enhanced sharing and collaboration among outpatient
section of the discharge summary, clinicians give patients practices, community pharmacies, and where relevant,
and their caregivers information that is both clearer and less hospital-based services should be a focus. The goal is to have
likely to lead to errors. more complete and up-to-date information readily available
Increased auditing as part of the medication reconcilia- by strengthening communication throughout the network of
tion process also allows for direct feedback to clinicians health care providers for each patient. In addition, patients
when errors are identified. This enables prescribers to and their families should be empowered to share information
correct the errors before a patient is discharged either from about their medications to improve medication reconciliation
an admission or an outpatient encounter. efforts further.
Pharmacists can be crucial resources in the process of The time invested today in medication reconciliation
medication reconciliation. As medication experts, they are pays dividends far into the future. By establishing effective
integral team members involved not only in direct patient medication reconciliation workflows throughout each patient’s
care but also during the process of admission, unit-to-unit network of care, from the community to the hospital setting,
transfers, and at the time of discharge. By providing custom- and promoting collaboration among physicians, nurses, and
ized education for patients and caregivers, pharmacists can pharmacists, we give our patients the best chance to be cared
help to substantially reduce medication errors and im- for safely.
prove patient satisfaction. Hospital pharmacists should rou- COMMENT: Why medication reconciliation? In the United
tinely review every patient’s medication list to determine States today, preventable medical errors are responsible for
the accuracy of all prescribed medications. As part of the about 400,000 deaths per year, third only to heart disease and
medical team, a pharmacist should be available to counsel cancer, as well as for something like 10 times as many serious
every patient or caregiver, providing another opportunity for complications not resulting in death. The cost of preventable
patients and their families to learn about their medications. errors in terms of human suffering, not to mention dollars, is
Another helpful intervention in the hospital setting is to enormous. Of course, not all preventable errors are adverse
have prescriptions/discharge medications delivered directly drug events, but enough are to make this an issue we need to
to patients at the bedside before discharge from inpatient take very seriously in our offices as well as in our hospitals.
units, perioperative areas, and the ED. This gives pharmacists – Henry M. Adam, MD
the opportunity to provide counseling and education about Associate Editor, In Brief

Vol. 38 No. 1 JANUARY 2017 55


Brief
in

CHARGE Syndrome
Alexandra Hudson, HBSc,* Carrie-Lee Trider, MD,† Kim Blake, MB, FRCPC*‡
*Dalhousie University School of Medicine, Halifax, Nova Scotia, Canada.

Queen’s University School of Medicine, Kingston, Ontario, Canada.

IWK Health Centre, Nova Scotia, Canada.

AUTHOR DISCLOSURE Ms Hudson, Dr Trider, CHARGE syndrome (CS) is an autosomal dominant genetic condition caused
and Dr Blake have disclosed no financial
by a mutation in the CHD7 gene. The incidence is approximately 1 in 10,000 to
relationships relevant to this article. This
commentary does not contain a discussion 15,000 live births. Most cases result from de novo mutations on the q12 arm of
of an unapproved/investigative use of a chromosome 8, which interfere with neural crest cell migration and embryo-
commercial product/device.
genesis. The parent of a child with a de novo mutation causing CS has a
recurrence risk of approximately 1% to 3% for future pregnancies. However, if
the disease-causing mutation of the proband is found in a parent, depending upon
whether in a mosaic or nonmosaic form, the recurrence risk may be increased up
to 50%, and subsequent pregnancies can be screened genetically and with fetal
ultrasonography and/or magnetic resonance imaging.
CS remains a predominantly clinical diagnosis that can be confirmed with
genetic testing. Physical features are highly variable and include those repre-
sented by the CHARGE mnemonic: Coloboma ocular, Heart defects, Atresia or
stenosis of the choanae, Retardation of growth and/or development, Genito-
urinary anomalies, and Ear abnormalities. However, the mnemonic does not
include all of the major diagnostic criteria. Central nervous system find-
ings are important clues to the diagnosis, with more than 90% of affected
individuals having cranial nerve (CN) dysfunction. Many individuals have
more than 1 dysfunctional cranial nerve, manifesting as an absent or reduced
sense of smell (CN I), weak chewing/swallowing (CN V), facial palsy (CN VII)
(Fig 1), sensorineural hearing loss (CN VIII), balance vestibular problems (CN
VIII), and swallowing problems (CN IX, X). Visual and hearing impairments
as well as intellectual disabilities are prevalent and can range from mild to
Clinical Utility Gene Card for CHARGE severe.
Syndrome - Update 2015. van Ravenswaaij- Because CS is the most common syndrome associated with choanal atresia or
Arts CMA, Blake K, Hoefsloot L, Verloes A. Eur
J Hum Genet. 2015;23(11) stenosis, it should be considered in the differential diagnosis of any infant
presenting with this defect. The characteristic outer ear malformation seen in
CHARGE Syndrome: A Review. Hsu P, Ma A,
Wilson M, et al. J Paediatr Child Health. 2014;50
CS is another important diagnostic clue (Fig 2). The diagnosis is also informed by
(7):504–511 imaging of the inner ears (for cochlear/vestibular anomalies and absent olfac-
CHARGE Syndrome. Blake KD, Prasad C.
tory bulbs). A thorough clinical evaluation of an individual suspected to have CS
Orphanet J Rare Dis. 2006;7(1):34 should include echocardiography, renal ultrasonography, cranial computed
Postoperative Airway Events of Individuals
tomography scan, cerebral magnetic resonance imaging, audiometry testing,
with CHARGE Syndrome. Blake K, MacCuspie CN testing, a swallowing study, nasal endoscopy, and funduscopy.
J, Hartshorne TS, Roy M, Davenport SL, An early diagnosis is important to establish a multidisciplinary care team to
Corsten G. Int J Pediatr Otorhinolaryngol.
manage developmental concerns. CS is one of the most common causes of
2009;73(2):219–226
combined deafness-blindness and should be considered in all infants who have
Quality of Life in Adolescents and Adults
both of these sensory deficits.
with CHARGE Syndrome. Hartshorne N,
Hudson A, MacCuspie J, et al. Am J Med Genet Communication can be challenging for individuals with CS. They are best
A. 2016;170(8):2012–2021 supported using multiple methods, including verbal, sign language, picture

56 Pediatrics in Review
has been developed to better evaluate pain. Aside from
hearing and vision impairment, the senses of touch, body
position, balance, smell, and taste are often dysfunctional.
Balance and vestibular problems can cause difficulties with
mobility, creating the need for a walker early in life to allow
the child to walk in an upright position. Affected children
should undergo a mobility and sensory integration assess-
ment by an occupational and physical therapist so that
adaptations for sensory impairments can be provided early
in life and into school.
Long-term and complex feeding issues can contribute
to morbidity and mortality. Feeding difficulties are highly
prevalent, mostly present from birth, but can start at any
Figure 1. Facial palsy is a common manifestation of cranial nerve
dysfunction in CHARGE syndrome.
point across the lifespan. Gastrostomy/jejunal tube feed-
ing may be necessary and can continue for many years. CN
dysfunction is hypothesized as the primary clinical anomaly
exchange, and gestures. Cochlear implants are an option if adversely affecting feeding development. The most com-
there is an intact vestibular nerve (Fig 3). Bone-anchored mon upper gastrointestinal tract problems are weak suck-
hearing aids are also an option for hearing impairment. Pain ing/chewing, dysphagia, severe gastroesophageal reflux
can be an underdetected symptom in genetic disorders (GER), and aspiration. Problematic feeding behaviors, such
in which individuals have difficulty communicating. For as overstuffing of the mouth or pocketing of food in the
children who are nonverbal, the “Multidimensional Pain cheeks, are also encountered. Constipation and bowel dys-
Assessment for Individuals with CHARGE Syndrome” regulation are common, believed to result from vagus nerve
dysfunction affecting gut motility. Every patient who has CS
and feeding difficulties should be assessed by a multidisci-
plinary team, including an occupational therapist, physical
therapist, and speech-language pathologist to evaluate swal-
lowing and CN function. A scale specifically designed for CS
is currently being developed to help parents and therapists
assess and monitor feeding difficulties over time.
Patients with CS often require numerous surgeries and
face a substantial risk of postoperative airway events with
anesthesia. Where possible, multiple surgical procedures
should be combined under one anesthetic to reduce the risk
of intubation and postoperative complications. In child-
hood, sleep apnea may warrant the removal of tonsils and
adenoids and should be investigated by an otolaryngologist.
Excessive salivary secretions, which can lead to aspiration
and related complications, are also a common problem;
injections of onabotulinumtoxinA into the salivary glands
can be effective and may avert the need for a tracheostomy.
Although infant mortality is high in CS, life expectancy
has improved for patients who survive beyond the first
postnatal year. Death in the neonatal period is associated
with combinations of major cardiovascular malforma-
Figure 2. The characteristic outer ear appearance in CHARGE syndrome
tions, bilateral choanal atresia, esophageal atresia, severe
is a short, wide, cup-shaped ear with decreased cartilage and a triangular
concha. The ears are often asymmetric. T-cell deficiencies, and central nervous system anomalies.

Vol. 38 No. 1 JANUARY 2017 57


an endocrinologist, consultation with a nutritional therapist
can be helpful.
New issues that appear in adolescence/adulthood include
scoliosis, sleep apnea, gallstones, seizures, ophthalmo-
logic complications (eg, retinal detachment), and the onset
of problematic feeding behaviors (eg, food pocketing in
cheeks). Parents should be advised that acute behavioral
changes warrant prompt clinical evaluation. Adolescents
and adults with CS continue to need care in a variety of
subspecialties (ophthalmology, otolaryngology, endocrinol-
ogy, and cardiology) as well as ongoing coordination of their
care by their primary care physician.
A newly developed and easy-to-use comprehensive clin-
ical checklist is available to clinicians and parents to facilitate
coordinating the complex medical needs faced by patients
with CS across their lifespans. The checklist tracks all the
medical specialties involved in the care of the particular
individual and alerts caregivers to the timing of common
medical issues that may affect the person with CS.
A book titled CHARGE Syndrome by Hartshorne et al was
published in 2010, describing the sensory, physical, and
behavioral findings in CS. Individuals and parents can also
learn more about CS through www.chargesyndrome.org, a
website created by The International CHARGE Syndrome
Foundation. SENSE, a national charity in the United King-
Figure 3. A cochlear implant used to ameliorate hearing loss from dom for people with deafness and blindness, has created
common middle and inner ear malformations in CHARGE syndrome.
a “CHARGE Information Pack for Practitioners” consisting
of 28 factsheets summarizing all aspects (medical and
Mortality in the postnatal period is attributed primarily to educational) of living with CS.
CN dysfunction, specifically to difficulties with swallowing, Lastly, individuals and their parents with CS can join the
GER disease, respiratory aspiration, and postoperative air- International CHARGE Syndrome Foundation to become
way events. Early management of CS often involves repair connected to the larger global community affected by this
of the malformations (cardiac, choanal atresia, esophageal genetic disorder. The International Foundation, the United
atresia) and treatment of feeding difficulties and GER (tube Kingdom, and Australasia (Australia and New Zealand) all have
feeding and antireflux and motility medications). well-established Facebook groups where parents can post ques-
An endocrinologist should follow affected children as tions to other parents of children with CS. Family support groups
they move into preadolescence because of the risk of delayed that exist around the world are the CHARGE Family Support
puberty. Hormone replacement therapy is often indicated, Group UK, Texas CHARGE Group, CHARGE Syndrome Asso-
especially when there were signs of hypogonadotropic hy- ciation of Australasia, Association CHARGE (France), Dutch
pogonadism at birth. Growth hormone deficiency may also CHARGE Syndrome Network, Belgian CHARGE Syndrome,
be a problem, although it is less prevalent. Poor bone health CHARGE Syndrome Canada, CHARGE Syndrome German,
is common with CS. Contributing risk factors include Asociacion Espanona Sindrome de CHARGE (Spain), and
dietary restrictions, inactivity, and hypogonadism. Patients National Family Association for Deaf-Blind (USA). This is
often fail to meet the recommended nutritional intake of not an exhaustive list and does not list all of the available
calcium and vitamin D and are at risk for osteoporosis. family support groups for CS around the world.
Failure to thrive is common in childhood, but obesity COMMENT: First described in 1979 in a report of 17
becomes prevalent later in life. In addition to input from children with choanal atresia, CHARGE was coined as an

58 Pediatrics in Review
acronym in 1981. Initially it was termed the CHARGE as- other acronymic associations have had their underlying
sociation, not syndrome. A syndrome in genetics denotes causes identified (WAGR and LEOPARD are examples),
a set of signs and/or symptoms appearing together that re- but some (like VACTERL) have not yet graduated from
sults from an identified cause. In 1981, the CHD7 gene on association to syndrome. If by chance you don’t remem-
chromosome 8 was not known to be the site of mutation(s) ber what these acronyms represent, have fun looking
underlying the condition, so all that could be claimed was them up. I do almost every time I come across one of
that an association had been recognized, indicating that them!
this group of anomalies appeared together more often
than would happen purely by chance. With dramatic – Henry M. Adam, MD
advances in genetic technology over the past few decades, Associate Editor, In Brief

Vol. 38 No. 1 JANUARY 2017 59


A Child with Presumed Reactive Airway
Disease, Pectus Carinatum, and Aortic
Root Dilation
Aamir Jeewa, MD,* Shaine A. Morris, MD, MPH,* William J. Dreyer, MD,*
Iki Adachi, MD,† Susan W. Denfield, MD,* E. Dean McKenzie, MD†
*Lillie Frank Abercrombie Section of Cardiology, Department of Pediatrics;

Michael E. DeBakey Department of Surgery, Division of Congenital Heart Surgery, Baylor College of
Medicine, Houston, TX.

PRESENTATION

A 6-year-old girl of South Asian descent has a history of reactive airway disease
that has been treated with bronchodilators from age 3 years. She is referred to
a pulmonologist for refractory wheezing after multiple antibiotic courses and
treatments with levalbuterol and corticosteroids, both oral and inhaled. Her
parents had noted an increase in wheezing and upper respiratory tract infections
during the winter months. She has no history of recurrent pneumonias or
other hospitalizations. Prior chest radiographs had not shown any signifi-
cant pulmonary pathology but incidentally did show evidence of scoliosis.
The patient’s parents had noticed a chest deformity since the age of 4 years.
Allergy testing shows she is allergic to mold, eggs, grass, and peanuts. The
patient’s perinatal history is unremarkable. The family history is notable for
a maternal grandfather who died suddenly after developing chest pain at 45
years of age.

AUTHOR DISCLOSURE Drs Jeewa, Morris,


Dreyer, Denfield, and McKenzie have
disclosed no financial relationships relevant to
this article. Dr Adachi has disclosed that he is a
consultant for New England Research
Institute’s (NERI) PumpKin trial with the Jarvik
ventricular assist device, for which his
employing institution receives salary support;
he also serves as a consultant for Heartware,
Inc, and Sony-Olympus, Inc. This commentary
does not contain a discussion of an
unapproved/investigative use of a Figure 1. Preoperative computed tomography angiography (sagittal [A] and short axis [B] views)
commercial product/device. showing the dilated aortic root.

Vol. 38 No. 1 JANUARY 2017 e1


TABLE. Major Systems Involved in Loeys-Dietz
Syndrome
Vascular
• Aortic complications include dissection or dilation; aortic root
dilation is common and seen in >95% of probands
• Other arterial aneurysms and tortuosity are common, with 50% of
the aneurysms detected away from the aortic root
• Atrial septal defect and patent ductus arteriosus are frequently
seen
• Bicuspid aortic valve may be associated
• Mitral valve prolapse is common

Figure 2. Transesophageal echocardiography with color Doppler Musculoskeletal


demonstrating severe aortic insufficiency (arrows).
• Pectus deformities: either excavatum or carinatum
• Spinal curvature disorder: scoliosis
The patient’s initial physical examination reveals a Sex- • Joint hypermobility (most often involving the fingers)
ual Maturity Rating 1 girl with a thin build (84th percentile
• Arachnodactyly
for height and 27th percentile for weight), frontal bossing,
mild bilateral tonsillar hypertrophy, pectus carinatum, and • “Club foot” or talipes equinovarus

a hyperdynamic precordium. Further evaluation demon- • Craniosynostosis: most commonly dolichocephaly, brachycephaly,
and/or trigonocephaly
strates a wide uvula with a prominent raphe, hypertelorism,
downward-slanting palpebral fissures, mild thoracic sco- Craniofacial
liosis, and a grade II/VI systolic ejection murmur that is • Widely spaced eyes or hypertelorism
• Presence of a cleft palate or a bifid/wide uvula
• Premature fusion

Cutaneous
• Translucent skin
• Easy bruising
• Dystrophic scars

Other (by systems)


• Allergy and Immunology: Environmental and food allergies
• Gastrointestinal: Malabsorption, chronic diarrhea, abdominal pain,
and/or gastrointestinal bleeding and inflammation; possible
rupture of the spleen or bowel
• Genitourinary: Potential for rupture of the uterus during pregnancy

loudest at the right upper sternal border with continuation


into diastole.

DIAGNOSIS

Based on the abnormal cardiac findings and the long-


standing history of reactive airway disease, computed
Figure 3. Intraoperative photograph of the massively dilated aortic root
and ascending aorta. The root is rotated such that the noncoronary sinus tomography (CT) angiography of the chest was performed
lies more anterior than usual and the right coronary artery origin is
displaced leftward, with the right coronary artery stretched over the
that showed massive dilation of the aortic root (6.5 cm),
dilated root. rt¼right. moderate dilation of the ascending aorta and transverse

e2 Pediatrics in Review
tissue disorder (CTD). During intraoperative inspection
for aortic root replacement, a massively dilated aortic root
measuring 6.5 cm was noted (Fig 3).

Discussion
Loeys-Dietz syndrome (LDS) belongs in the category of
CTDs and is primarily due to mutations in TGFBR1
or TGFBR2. (1) This syndrome is characterized by involve-
ment in four major systems (Table) (2). A key distinction
compared to Marfan syndrome is that patients with LDS
appear to have a higher overall risk of dissection and of
early mortality, even though they have similar incidences
of aortic dilation.
The inheritance pattern is primarily autosomal dominant
with most cases (w75%) due to a de novo mutation. In this
case, genetic testing documented c.797 A>G transition in
exon 4 of the TGFBR1 gene that results in conversion of a
codon for aspartic acid to a codon for glycine in the serine-
threonine kinase domain of TGFBR1. Outcome data for the
rare patient with CTD requiring orthotopic heart transplan-
Figure 4. Three-dimensional reconstruction from computed tation (OHT) are limited and conflicting. (3)(4) For LDS in
tomography angiography performed after valve-sparing aortic root particular, a single case report exists of a 44-year-old man
replacement with graft replacement of the aortic arch and innominate
artery. with systolic failure who underwent emergent OHT before
obtaining the genetic diagnosis. (5)

arch, a patent ductus arteriosus, and significant left Patient Course


ventricular dilation (Fig 1). Transthoracic echocardiogra- The patient underwent aortic root replacement with a 21-
phy revealed severe aortic insufficiency and moderately mm HP St. Jude mechanical composite graft, graft replace-
depressed left ventricular function (Fig 2). Genetic test- ment of the entire aortic arch and innominate artery, and
ing confirmed the suspected diagnosis of a connective ligation and division of the patent ductus arteriosus (Fig 4).

Figure 5. A. Three-dimensional reconstruction from magnetic resonance angiography 6 months after cardiac transplantation with replacement of
entire arch with donor arch except for the left carotid and subclavian artery origins, which were incorporated using a beveled anastomosis with the
donor aorta. B: Magnetic resonance angiography 1.5 years after transplantation documenting progressive dilation localized to the native tissue around
the left carotid and subclavian arteries.

Vol. 38 No. 1 JANUARY 2017 e3


the donor arch was used to replace the entire arch graft. A
beveled anastomosis was created to incorporate the left
carotid and subclavian arteries. After lengthy rehabilita-
tion, she was discharged 1.5 months later and is now more
than 3 years post-OHT.
She is currently followed by the pediatric cardiac trans-
plant team and the cardiovascular genetics team. In addition
to her immunosuppression regimen, she is receiving com-
bination therapy of a b-blocker and angiotensin receptor
blocker and undergoes serial echocardiography and mag-
netic resonance angiography. Although her cardiac function
has been normal, the aortic arch has progressively dilated.
This increase in diameter appears to be limited to the area of
native aortic tissue remaining in the arch surrounding the
origins of the head and neck vessels (Fig 5).
Of note, the arterial tortuosity in this patient, as mea-
sured by the vertebral artery tortuosity index (VTI) on
magnetic resonance angiography, is severely increased,
with a measurement of 179 (Fig 6). In a study by Morris
et al, (6) patients with a VTI greater than 50 (defining severe,
with normal measured as <10) had a median age at first sur-
gery of 11.2 years, had an average of 0.8 surgeries per decade,
and were at the highest risk of experiencing an aortic dis-
section. In this study, 6 patients had VTIs equal to or greater
than 100, with 4 dying at ages ranging from 5 months to 26
years and 2 surviving at ages 11 and 21 years.

Summary
• Vascular, skeletal, craniofacial, and cutaneous findings on
physical examination should raise suspicion for the diagnosis of a
connective tissue disorder (CTD).
Figure 6. Three-dimensional reconstruction of computed tomography • Multimodality imaging techniques (such as chest radiography,
angiography documenting significant arterial tortuosity, most evident in echocardiography, computed tomography scan, and magnetic
the vertebral arteries (see arrows). Vertebral artery tortuosity index is
resonance angiography) are often required for complete
severely elevated at 179.
assessment of the cardiac and vascular abnormalities that can
occur in children with Loeys-Dietz syndrome (LDS).
• To our knowledge, this is the first successful case of heart
Pathology of the aorta at the time of repair showed a vari- transplantation in a pediatric patient with LDS.
ably thickened aortic wall with diffusely increased acid • Close long-term follow-up evaluation is crucial in this very high-
mucopolysaccharides and markedly decreased elastic tissue risk patient to try to prevent the occurrence of CTD-related
complications.
and smooth muscle fibers. Postoperatively, the girl devel-
oped severely depressed left ventricular function. She was
ultimately discharged but returned to the hospital later with
seizures and chorealike movements. Neuroimaging at the
time ruled out any evidence of a stroke. Her heart failure
References
worsened such that she required mechanical ventilation and
1. Loeys BL, Chen J, Neptune ER, et al. A syndrome of altered
inotropic support. After extensive evaluation and literature
cardiovascular, craniofacial, neurocognitive and skeletal development
review for OHT in CTD, she was listed for cardiac trans- caused by mutations in TGFBR1 or TGFBR2. Nat Genet. 2005;
plant. She received the transplant 17 days after listing, and 37(3):275–281

e4 Pediatrics in Review
2. Loeys BL, Dietz HC. Loeys-Dietz syndrome. GeneReviews. Seattle, 5. Eckman PM, Hsich E, Rodriguez ER, Gonzalez-Stawinski GV,
WA: 2013. Available at: https://www.ncbi.nlm.nih.gov/books/ Moran R, Taylor DO. Impaired systolic function in Loeys-Dietz
NBK1133/. Accessed October 24, 2016 syndrome: a novel cardiomyopathy? Circ Heart Fail. 2009;
3. Kesler KA, Hanosh JJ, O’Donnell J, et al. Heart transplantation in 2(6):707–708
patients with Marfan’s syndrome: a survey of attitudes and results. 6. Morris SA, Orbach DB, Geva T, Singh MN, Gauvreau K,
J Heart Lung Transplant. 1994;13(5):899–904 Lacro RV. Increased vertebral artery tortuosity index is associated
4. Knosalla C, Weng YG, Hammerschmidt R, et al. Orthotopic heart with adverse outcomes in children and young adults with
transplantation in patients with Marfan syndrome. Ann Thorac Surg. connective tissue disorders. Circulation. 2011;124(4):
2007;83(5):1691–1695 388–396

Vol. 38 No. 1 JANUARY 2017 e5

You might also like